Sie sind auf Seite 1von 217

Item: 1 of 33 ~ 1 • M k -<:J 1>- Jil ~· !

:';-~
QIO: 5114 ..L ar Pre v ious Next Labfli!llues Not es Calcula t o r

•1 & &

A mother brings her 12-year-old son to the pediatrician's office because he is complaining of leg pa in. The
•2
pain is located at the proximal right tibia and has persisted over the past 2 weeks. The mother also notes
•3 that the child has had intermittent fevers during th is time . On examination/ the site is erythematous and
·4 swollen . A plain-film X-ray is taken/ the results of wh ich are shown in the image. Biopsy of the site shows sheets of
•5
many un iform cells with scant/ clear cytoplasm and no evidence of norma l bony matrix.
•6
Wh ich of the following is the most likely diagnosis?
•7

·8 :
A. Ewing sarcoma
.9
• 10 B. Giant cel l tumor
• 11 C. Osteochondroma
• 12
D. Osteomyelitis
• 13
• 14
E. Osteosarcoma
• 15
• 16
• 17
• 18
• 19
• 20
• 21

a
Lock
s
Suspend
8
End Bl ock
Item: 1 of 33 ~ 1 • M k -<:J 1>- Jil ~· !:';-~
QIO: 5114 ..L ar Pre v ious Next Labfli!llues Not es Calcula t o r
& &
1
•2 The correct answer is A. 41°/o chose this.
•3 The most likely diagnosis is Ewing sarcoma, which most common ly occurs on
the metaphysis of the long bones, pelvis, scapula, or r ibs. Made up of
·4 anaplastic, small blue cells, Ewing sarcoma is classified as one of the
•5 primitive neuroectodermal tumors. It is more frequently seen in patients who
•6 are young ( <15 or 20 years old), male, and Caucasian. This highly aggressive
•7
malignancy is associated with a t(11;22) translocation.

·8 X-ray films of Ewing sarcoma classically reveal a lytic lesion with "onion
skinning" of the periosteum, indicated by the arrow. This reactive process
.9
occurs as the tumor arises out of the medullary cavity and new layers of bone
• 10 are deposited around it by the periosteum. Microscopic analysis of biopsy
• 11 specimens reveals sheets of uniform, small round cells that are slightly larger
• 12
than lymphocytes. Rosette formations may be seen as cells arrange
themselves around a central fibrous space. Otherwise, there is very little
• 13 stromal space and no normal bony matrix material.
• 14
Periosteum Medullary cavity Metaphysis Ewing's sarcoma Scapula Sarcoma Malignancy Biopsy Cancer Pelvis
• 15
Neoplasm Bone tumor Lesion Lymphocyte Long bone Lytic cycle X-ray Anaplasia Lysis Stroma (animal tissue)
• 16
Bone
• 17
Image courtesy of Michael
• 18 Richardson, MD
• 19
• 20 B is not correct. 6°/o chose this .
• 21 Giant cell tumors differ in their most common presentation, peaking in

a
Lock
s
Suspend
8
End Bl ock
Item: 1 of 33 ~ 1 • M k -<:J 1>- Jil ~· !:';-~
QIO: 5114 ..L ar Pre v ious Next Labfli!llues Not es Calcula t o r
& &
1
•2 B is not correct. 6°/o chose this.
•3 Giant cell tumors differ in their most common presentation, peaking in
·4 incidence between the ages of 20 and 40 years, and arising from the
epiphyseal end of long bones. Plain-film X-ray classically shows a "soap
•5
bubble" appearance (as shown by the arrows), and histology shows
•6 spindle-shaped cells with multinucleated giant cells interspersed
•7 between them.
Histology ay Epiphysis Epiphyseal plate Giant cell
·8
.9
• 10
• 11
Image courtesy of Wikimedia
• 12
Commons
• 13
• 14 C is not correct. 18°/o chose this .
• 15 Osteochondroma, the most common benign tumor of bone, appears to extend
from the bone as a cartilaginous cap on a mature bony stalk ( indicated
• 16
by the white dots). Also known as exostosis, it very rarely converts to a
• 17 malignant neoplastic process.
• 18 Osteochondroma Exostosis Neoplasm Cartilage Malignancy Benign tumor Cancer Benignity

• 19
• 20
• 21

a
Lock
s
Suspend
8
End Bl ock
Item: 1 of 33 ~ 1 • M k -<:J 1>- Jil ~· !:';-~
QIO: 5114 ..L ar Pre vious Next Labfli!llues Notes Calculator
-·- •• -r. • .,, , - ...
1
Commons
•2
•3 C is not correct. 18°/o chose this.
·4 Osteochondroma, the most common benign tumor of bone, appears to extend
•5 from the bone as a cartilaginous cap on a mature bony stalk (indicated
by the white dots). Also known as exostosis, it very rarely converts to a
•6
malignant neoplastic process.
•7 Osteochonc oma E ostosis Neoplasm cartilage Malignancy Benign tumor Cancer Benignity

·8
.9
• 10
• 11
• 12
• 13
• 14
• 15
• 16
• 17
• 18 Image courtesy of Lucien Monfils
• 19
D is not correct. 12°/o chose this .
• 20
The child's presentation w ith gradually progressing leg pain, fevers, and
• 21 erythema does initially seem to point to osteomyelitis. However, biopsy

a
Lock
s
Suspend
8
End Block
Item: 1 of 33 ~ 1 • M k -<:J 1>- Jil ~· !:';-~
QIO: 5114 ..L ar Pre v ious Next Labfli!llues Not es Calcula t o r
& &
1
•2
D is not correct. 12°/o chose this.
The child's presentation w ith gradually progressing leg pain, fevers, and
•3
erythema does initially seem to point to osteomyelitis. However, biopsy
·4 resu lts suggest a ma lignant process, rather than the necrotic picture one
•5 would expect in osteomyelitis. Additionally, X-ray in osteomyelitis should
•6 show destruction of cortical bone and osteopenia rather than the lytic lesions
seen in the vignette.
•7
Osteomyeli •s Osteopema Erythema Cortical bone Biopsy Lysos X-ray Cancer Lytic cycle Necrosis Malignancy
·8 Bone
.9
• 10
• 11
• 12
• 13
• 14
• 15
• 16
• 17
Image courtesy of Wikimedia
• 18 Commons
• 19
• 20
E is not correct. 23°/o chose this .
Osteosarcoma is the most common primary malignant bone tumor in ch ildren .
• 21
• I ikP Fwinn c:;r~rrnmr~. it nP.r~kc:; in nr:r:1 1rrP.nc:P. hP.tWPPn thP .::~npc; nf 10 .::~nrl /0

a
Lock
s
Suspend
8
End Bl ock
Item: 1 of 33 ~ 1 • M k -<:J 1>- Jil ~· !:';-~
QIO: 5114 ..L ar Prev ious Next Labfli!ll ues Not es Calculat o r

& &
1
E is not correct. 23°/o chose this.
•2
Osteosarcoma is the most common primary ma lignant bone tumor in ch ildren . .-----...-.
•3
Like Ewing sarcoma, it peaks in occurrence between the ages of 10 and 20
·4 years but arises more often in the diaphysis of long bones.
•5 However, the key difference is in the results of the imaging and biopsy
•6 studies. Classically, osteosarcomas show a Codman triangle or "sunburst"
•7 pattern on plain-film X-ray. Histologically, cellular patterning is highly
variable, with a number of subtypes. The most characteristic feature,
·8
however, is the formation of bony matrix material (indicated by the
.9 arrow) by the tumor cells, which does not occur in Ewing sarcoma.
• 10 Osteosarcoma Diaphysis Ewing's sarcoma Codman triangle Bone tumor Biopsy Sarcoma Malignancy Histology
• 11 Neoplasm Cancer X-ray Bone Long bone
• 12
• 13
• 14
• 15
• 16
• 17
• 18
• 19
• 20
• 21 Image copyright © 2013
• T'.......... -&.............. ~ -1 • /: .......... -.- ......... o: ..... AA ..... -1

a
Lock
s
Suspend
8
End Bl ock
•2 Osteosarcoma Diaphysis Ewing' s sarcoma Cadman triangle Bone tumor Biopsy Sarcoma Malignancy Histology

•3 Neoplasm Cancer X-ray Bone long bone

•4
•5
•6
•7
•8
•9
• 10

· 11
• 12
Image copyright © 2013
• 13
Toepfer eta/.; licensee BioMed
• 14 Central Ltd.
• 15
• 16
• 17 Bottom Line:
• 18 Ewing sarcoma is one of the smal l blue cel l, or primitive neuroectodermal, tumors. It occurs most commonly in
• 19
wh ite males between the ages of 10 and 20 years. It is associated with a t(11;22) translocation .
Ewing' s sarcoma Sarcoma Chromosomal translocation
• 20
• 21

6
lock
s
Suspend
0
End Block
Item: lof33 ~. , . M k <:] t> al ~· ~
QIO: 5114 .l. ar Previous Next lab 'lifllues Notes Calculator

1 • •
FA17 p438.1
•2
Primary bone tumors
•3 TUMOR TYPE EPIDEMIOLOGY/LOCATION CHARACTERISTICS
•4 Benign tumors
•5 Osteochondroma Most common benign bone tumor. Bony exostosis with cartilaginous (chondroid)
•6 Males < 25 years old. cap a.
•7 Rarely transforms to chondrosarcoma.
•8 Giant cell tumor 20-40 years old. Locally aggressive benign tumor.
•9 Epiphysis of long bones (often in knee region). "Soap bubble" appearance on x-ray : .
• 10
rises most commonly at distal femur and lultinucleated giant cells that express RANKL.
proximal tibia.
· 11
"Osteoclastoma."
• 12
Malignant tumors
• 13
Osteosarcoma One of the most common malignant bone Cod man triangle (from elevation of periosteum)
• 14
(osteogenic sarcoma) tumors. or sunburst pattern on x-ray.
• 15 Bimodal distribution: 10- 20 years old 00), > 65 Aggressive. Treat with surgical en bloc resection
• 16 (20). (with limb salvage) and chemotherapy.
• 17 Predisposing factors: Paget disease of bone, bone
infarcts, radiation, familial retinoblastoma,
• 18
Li-Fraumeni syndrome (germline p53
• 19
mutation).
• 20 tvletaphysis of long bones, often around knee ~.
• 21 Ewing sarcoma Boys< 15 years old. Anaplastic small blue cell malignant tumor [!]. •

6
lock
s
Suspend
0
End Block
Item: 1 of 33 ~ 1 • M k -<:J 1>- Jil ~· !:';-~
QIO: 5114 ..L ar Pre v ious Next Labfli!llues Not es Calcula t o r
& &
1 Ewing sarcoma Boys< 15 years old. Anaplastic small blue cell malignant tumor [!].
.2 Commonly appears in diaphysis of long bones, Extremely aggressive with early metastases, but
•3 pelvis, scapula, ribs. responsi,·e to chemotherapy.
"Onion skin" periosteal reaction in bone.
·4
Associated with t(l l ;22) translocation causing
•5
fusion protein E\VS-F'LI I.
•6 11 + 22 = 33 (Patrick E" ing's jerse~ number).
.7 ----------~~~~~

·8
.9
• 10 Round cell lesions---;-...
Ewing sarcoma
• 11 Myeloma
~ Fibrous dysplasia
• 12
• 13 Osteoid osteoma
lr tJetnw r- etnlf nd.
• 14
Simple bone cyst
• 15
• 16
• 17
. 18
• 19
c
-- -
Osteochondroma

Physis
1
• 20 l{
·a.
....
G~ant cell tumor
1:!1
• 21

a
Lock
s
Suspend
8
End Bl ock
Item: 2 of 33 ~ 1 • M k -<:J 1>- Jil ~· !:';-~
QIO: 5 0 93 ..L ar Pre v ious Next Lab fli!ltues Not es Calcula t o r

IAA]
& &
1
A 35-year-old man presents to the emergency departm ent with mu ltiple rib fractures after a low-speed
•2
motor vehicle collision. No one else involved in the crash was injured . Laboratory work-up revealed calcium
•3 of 15.3 mg/dl, phosphate of 1.8 mg/dl, and alkaline phosphatase of 172 IU/L. A representative radiograph
·4 of the patient's femu r is shown in the image.
•5
•6
•7

·8
.9
• 10
• 11
• 12
• 13
• 14
• 15
• 16
• 17
• 18
• 19
• 20
• 21

a
Lock
s
Suspend
8
End Bl ock
Item: 2 of 33 ~ 1 • M k -<:J 1>- Jil ~· !:';-~
QIO: 5 0 93 ..L ar Pre v ious Next Lab fli!ltues Not es Calcula t o r
&
------ -- - --- - - --- - - - - - - - - - --- - - - --- ----- -
&
1
of the patient's femur is shown in t he image.
.2
•3

·4
•5
•6
.7
·8
.9
• 10
• 11

• 12
• 13
• 14
• 15
• 16
• 17
. 18
• 19
• 20
• 21

a
Lock
s
Suspend
8
End Bl ock
•2
•3

·4
•5
•6
•7

·8
.9
• 10
• 11
• 12
What is the most likely cause of this patient's laboratory and imaging findings?
• 13
:
• 14
A. Osteitis fib rosa cystica
• 15
• 16
B. Osteoma lacia

• 17 C. Osteopetrosis
• 18 D. Osteoporosis
• 19
E. Paget's disease
• 20
• 21

a
Lock
s
Suspend
8
End Block
Item: 2 of 33 ~. I • M k <:] t> al ~· ~
QIO: 5093 .l. ar Previous Next lab 'lifllues Notes Calculator

1 •

2 The correct answer is A. 46°/o c hose this.


•3 The most like ly cause of this patient's cl inica l presentation is osteitis f ibrosa cystica, or von Recklinghausen's
disease of bone. It is a consequence of untreated hyperparathyroidism, lead ing to overactiv ity of the osteoclasts
•4
in bone resorption . With the increased rate of turnover of bone, ser um levels of ca lcium and alkal ine
•5 phosphatase are increased . At the same t ime, ser um phosphorus leve ls are decreased secondary to increased
•6 excretion by the kidney. The cystic changes in the skeleton pred ispose the bone to fractures and weight-bea ring
•7
defor mit ies. Recurrent microfractures and the resu ltant hemor rhage and influx of inflammatory med iators may
resu lt in the patholog ic f ind ing of a "brown tumor" in the bone .
•8 Osteitis fibrosa cystica Alkaline phosphatase Hyperparathyroidism Bone resorption Osteoclast Phosphorus Kidney Inflammation Neoplasm
•9 Neurofibromatosis type I Calcium Skeleton Bone Bleeding
• 10
B is not correct. 15% chose this .
· 11
Osteoma lacia, the consequence of v itamin D deficiency in adults, may mimic an osteoporotic lesion
• 12
rad iograph ica lly . However, serum leve ls of ca lcium are either low or norma l and alkaline phosphatase is not
• 13 markedly elevated .
Osteomalacia Alkaline phosphatase Vitamin 0 Osteoporosis Vitamin Blood plasma X-ray Calcium Hypovitaminosis 0 lesion Alkalinity Radiology Radiography
• 14
• 15 C is not correct . 14% chose this .
• 16 Osteopetrosis is a consequence of ma lfunctioning osteoclasts . Osteoblasts lay down bony matr ix unabated .
• 17 Though bone density is increased, the bone is brittl e and predisposed to fracture . Rad iog raph ica lly, the
rad iopacity of the bone is increased, with loss of the medu llary space .
• 18
Osteopetrosis Osteoclast Osteoblast X-ray Bone density Radiography Radiology Radiodensity Dental radiography Bone Bone fracture
• 19
• 20
D is not correct. 7°/o chose this .
Though resembling osteoporosis radiog raphica lly, th is patient is both young and male, making it un likely that he
• 21
• wmJ irl hr~vP. <;tJr:h nrofotmrl nr~tho l onir: r:hr~nnP.c; from oc;tP.onoroc;ic; r~ l onP. . Tn r~rlrlit i on. thP. <;P. rt Jm IP.vP.Ic; of r:r~ l dtJm .

6
lock
s
Suspend
0
End Block
Item: 2 of 33 ~. I • M k <:] t> al ~· ~
QIO: 5093 .l. ar Previous Next lab 'lifllues Notes Calculator

1 •
Osteomalacia Alkaline phosphatase Vitamin D Osteoporosis Vitamin Blood plasma X-ray Calcium Hypovitaminosis D lesion Alkalinity Radiology Radiography

2
C is not correct. 14% chose this.
•3
Osteopetrosis is a consequence of malfunctioning osteoclasts. Osteoblasts lay down bony matr ix unabated.
•4 Though bone density is increased, the bone is brittle and predisposed to fracture . Rad iog raph ically, the
•5 radiopacity of the bone is increased, with loss of the medu llary space.
Osteopetrosis Osteoclast Osteoblast X-ray Bone density Radiography Radiology Radiodensity Dental radiography Bone Bone fracture
•6
•7 D is not correct. 7°/o chose this.
•8 Though resembling osteoporosis radiog raphically, th is patient is both young and male, making it unlikely that he
•9 wou ld have such profound pathologic changes from osteoporosis alone . I n addition, the serum levels of ca lcium,
phosphor us, and alkaline phosphatase are typical ly normal with osteoporosis .
• 10
Osteoporosis Alkaline phosphatase Radiography Phosphorus X-ray Phosphatase Radiology Alkalinity Calcium Blood plasma Dental radiography Serum (blood)
· 11
E is not correct. 18% chose this .
• 12
Paget's disease of bone, resu lting from a concordant increase in both osteoblastic and osteoclastic activity,
• 13
manifests as a sol itary elevation of al kal ine phosphatase, with nor mal levels of calcium and phosphorus. Both
• 14 sclerotic and lytic lesions may be viewed on radiography .
• 15 Paget' s disease of bone Alkaline phosphatase Phosphorus Alkalinity Osteoblast lytic cycle Radiography Calcium lysis Bone Sclerosis (medicine)

• 16
• 17
Bottom Line:
• 18
Osteitis fibrosa cystica may be diagnosed with hypercalcemia, elevated alkaline phosphatase,
• 19 hypophosphatemia, and osteoporotic lesions on x-ray .
• 20 Osteitis fibrosa cystica Hypercalcaemia Alkaline phosphatase Osteoporosis Hypophosphatemia X-ray

• 21

6
lock
s
Suspend
0
End Block
Item: 2 of 33 ~ 1 • M k -<:J 1>- Jil ~· !:';-~
QIO: 5 0 93 ..L ar Pre v ious Next Lab fli!ltues Not es Calcula t o r
& &
1
FA17 p 332.1
2
Hyperparathyroidism
•3
Primary Usually due to parathyroid adenoma or Osteitis fibrosa cystica-cystic bone spaces
·4 hyperparathyroidism hyperplasia. Hyp ercalcemia, hypercalciuria filled with brown fibrous tissue · ('" brown
•5 {renal stones}, polyuria (throne~), tumor'' consisting of osteoclas ts and deposited
•6 hypophosphatemia, t PTI I, t ALP, t cA ~ l P in hemosiderin from hemorrhages; causes
.7 urine. \ lost often asymptomatic. \ lay present bone pain). Due to t PTH, classically
with weakness and constipation ("groan<,"), associated\\ ith 1° (but also seen\\ ith 2°)
·8
abdominaJ/flan k pain ( k idne~ stones, acute h) perparathyroidism).
.9
pancreatitis), depression ('"ps}ch iatric "Stones. thrones, bones, groans, and
• 10 O\ crtoncs"). p~yc hiatric O\Crtoncs."
• 11

• 12 Secondary zohyperplasia due to ! Ca2+ absorption Renal osteodystrophy- renal disease .... zoand
• 13
hyperparathyroidism and/o r t P04 3- , most often in chronic 3° hr perparathyroidism .... bone lesions.
renal disease (causes hypovitaminosis 0
• 14
and hyperphosphatem ia .... ! Ca 2+).
• 15 Hypocalcemia, hyperphosphatcmia in
• 16 chronic renal failure (vs hypophosphatcm ia
• 17 with most other causes), t ALP, t PTH .
. 18 Tertiary Refractory (autonomous) hyperparalh) roidism
• 19
hyperparathyroidism resulting from chronic renal disease. t t PTH ,
t C a 2+.
• 20
• 21
• ~6.17 "'A"t7 1

a
Lock Suspend
s 8
End Bl ock
Item: 2 of 33 ~. I • M k <:] t> al ~· ~
QIO: 5093 .l. ar Previous Next lab 'lifllues Notes Calculator

2
FA17 p437.1
•3
Lab values in bone disorders
•4
DISORDER SERUMCa 2+ PO l- ALP PTH COMMENTS
•5
Osteoporosis l bone mass
•6
Osteopetrosis - !l Dense, brittle bones. Ca 2+ l in severe,
•7
malignant disease
•8
Paget disease of bone t Abnormal "mosa ic" bone architecture
•9
Osteitis fibrosa cystica "Brown tumors" due to fibrous replacement of
• 10
bone, subperiosteal th inning
· 11 Primary t t t Idiopathic or parathyroid hyperplasia, adenoma,
• 12 hyperparathyroidism carcmoma
• 13 Secondary t t t Often as compensation for CKD (l P0 4 3-
• 14 hyperparathyroidism excretion and production of activated
• 15
vitamin D)
• 16 Osteomalacia/rickets t t Soft bones; vitamin D deficiency also causes zo
hyperparathyroidism
• 17

• 18
Hypervitaminosis D t t Caused by oversupplementation or
granu lomatous disease (eg, sarcoidosis)
• 19
• 20
• 21 •

6
lock
s
Suspend
0
End Block
Item: 3 of 33 ~ 1 • M k -<:J 1>- Jil ~· !:';-~
QIO: 3186 ..L ar Pre v ious Next Labfli!llues Notes Calcula t o r

IAA]
& &
1
An 81-year-old man complains of back pain that is du ll and constant, regardless of his activity level.
2
Laboratory tests reveal an elevated serum alkaline phosphatase level. The patient dies, and the image shows
•3 his back before his death.
·4
•5
•6
•7

·8
.9
• 10
• 11
• 12
• 13
• 14
• 15
• 16
• 17
• 18
• 19
• 20
• 21

a
Lock
s
Suspend
8
End Bl ock
2
•3

·4
•5
•6
•7

·8
.9
• 10
I m age courtesy of Radiopaedia/Drs. Henry Knipe and Frank Gaillar d
• 11
• 12
What was the most likely cause of th is patient's pa in?
• 13
:
• 14
A. Chondrosarcoma
• 15
• 16
B. Ewing sarcoma

• 17 C. Metastatic adenocarcinoma
• 18 D. Osteosarcoma
• 19
E. Paget disease of bone
• 20
• 21

a
Lock
s
Suspend
8
End Block
Item: 3 of 33 ~ 1 • M k -<:J 1>- Jil ~· !:';-~
QIO: 3186 ..L ar Pre v ious Next Labfli!llues Notes Calcula t o r
& &
1
The correct answer is C. 45°/o chose this.
2
Th is image shows several nodular sclerotic lesions (arrows), high ly suggestive
3 of metastases. Metastatic neoplasia shou ld be strongly considered in older
·4 adu lts with elevated alka line phosphatase levels, particu larly older men with
•5
•6
•7
back pain, often a sign of bone lesions due to prostate cancer. Metastases to
bone are far more common than primary bone tumors. The most common
cancers that metastasize to bone are breast, lung, thyroid, testes, kidney,
and prostate. They can cause lytic lesions (eg, in multiple myeloma, lung,
I
·8 thyroid and kidney cancers), blastic lesions (eg, in prostate and
.9 breast cancer), or mixed lesions (eg, in breast cancers). Given the fact that
the lesions in the image are sclerotic metastases in an older man, they are
• 10
almost certain to represent prostatic adenocarcinoma. When bone formation
• 11 predominates over bone destruction, the lesion is considered sclerotic, which
• 12 is the equivalent of a blastic lesion .
Multiple myeloma Alkaline phosphatase Metastasis Prostate cancer Adenocarcinoma Testicle Kidney Thyroid
• 13
Lysis Neoplasm Back pain Ossification Lytic cycle Lesion Lung Prostate Cancer Phosphatase Bone Alkalinity
• 14
Sclerosis (medicine)
• 15
• 16
• 17
• 18
• 19
Image courtesy of
• 20
Radiopaedia/Dr. Frank Gaillard
• 21

a
Lock
s
Suspend
8
End Bl ock
Item:3of33 ~. , . M k <:] t> al ~· ~
QIO: 3186 .l. ar Previous Next lab 'lifllues Notes Calculator

1 •

2
A is not correct. 7°/o chose this.
Chondrosa rcoma is a mal ignant cartilag inous tumor. It is most common in men 30-60 years old.
3
Chondrosa rcoma is rarely seen in the spine .
•4 Chondrosarcoma Cartilage Neoplasm Malignancy Chondrichthyes Cancer Vertebral column
•5
B is not correct. 5°/o chose this .
•6
Ewing sarcoma is an anaplastic blue cell malignant tumor most common in boys < 15 years of age. It is typica lly
•7 aggressive but highly responsive to chemotherapy. Most cases are the result of a t(11;22) chromosomal
•8 translocation .
Chromosomal translocation Chemotherapy Ewing' s sarcoma Malignancy Sarcoma Neoplasm Anaplasia Cancer Chromosome
•9
• 10 D is not correct. 15% chose this .
· 11 Osteosarcoma is the most common t r ue pr imary malignancy of bone in child ren and adolescents, with a pea k
• 12 incidence in teenage males (10-20 years old). Patients with a history of hereditary retinoblastoma are at an
increased risk and should be screened for this disease. It is most common ly found in the metaphysis of long
• 13
bones, most common ly in the lower extremities. It commonly man ifests with systemic symptoms and a palpable
• 14 soft tissue mass .
• 15 Osteosarcoma Metaphysis Retinoblastoma Malignancy Cancer Soft tissue Bone long bone

• 16 E is not correct. 28% chose this.


• 17 Paget disease of bone is characterized by accelerated bone turnover, leading to increased bone for mation and
• 18 resorption, resulting in its characteristic "mosaic" patterns. Infection with a vir us, especial ly paramyxovirus or
• 19
respiratory syncytial vi rus, may play a role in the etiology of the disease, but this is still controversial. There are
three phases to the disease : (1) it begins with the lytic phase-increased bone turnover by mor pholog ically
• 20 abnormal osteoclasts (2) followed by the mixed phase-increase in bone formation by osteoblasts, wh ich remain
• 21 morphologically normal but are increased in number and fina lly, (3) a blastic phase in wh ich bone for mation

6
lock
s
Suspend
0
End Block
Item:3of33 ~. , . M k <:] t> al ~· ~
QIO: 3186 .l. ar Previous Next lab 'lifllues Notes Calculator

1 • increased risk and should be screened for this disease. It is most common ly found in the metaphysis of long
2
bones, most common ly in the lower extremities. It common ly man ifests with systemic symptoms and a pa lpab le
soft tissue mass .
3 Osteosarcoma Metaphysis Retinoblastoma Malignancy Cancer Soft tissue Bone long bone
•4
E is not correct. 28% chose this.
•5
Paget disease of bone is characterized by accelerated bone turnover, leading to increased bone for mation and
•6
resorption, resulting in its characteristic "mosaic" patterns. Infection with a vir us, especial ly paramyxovirus or
•7 respiratory syncytial vi rus, may play a role in the etiology of the disease, but this is still controversial. There are
•8 three phases to the disease : (1) it begins with the lytic phase-increased bone turnover by mor pho log ical ly
abnorma l osteoclasts (2) fo llowed by the mixed phase-increase in bone formation by osteoblasts, wh ich remain
•9
morphologically normal but are increased in number and fina lly, (3) a blastic phase in wh ich bone for mation
• 10 dominates. It manifests most commonly with pa in, skeletal deformities, fractu res, and hearing loss due to bony
· 11 impingement of the cochlear nerve. Laboratory resu lts often show normal calcium and phosphate levels but an
• 12
elevated alkal ine phosphatase level (ALP) may occu r. It is associated with secondary osteosarcoma and
fibrosarcoma. It does not cause nodula r lesions like those in th is patient .
• 13 Paramyxoviridae Osteosarcoma Fibrosarcoma Paget' s disease of bone Alkaline phosphatase Human respiratory syncytial virus Osteoclast Osteoblast Bone
• 14
Syncytium Virus Hearing loss lytic cycle Etiology Phosphate Calcium Bone remodeling Ossification Alkalinity Morphology (biology) Bone resorption Infection
• 15
• 16
• 17 Bottom Line:
• 18 The primary cancers affecting the bone have varying morphologic appearances, and metastases to the bone
are fa r more common than primary cancers of bone .
• 19
Metastasis Morphology (biology) Bone
• 20
• 21

6
lock
s
Suspend
0
End Block
Item:3of33 ~. , . M k <:] t> al ~· ~
QIO: 3186 .l. ar Previous Next Lab 'lifllues Notes Calculator

1 •
FA17 p 221 .2
2
Common metastases Most sarcomas spread hcmatogcnously; most carcinomas spread via lymphatics. However, Four
3
Carcinomas Route Ilematogenously: Follicular thyroid carcinoma, Choriocarcinoma, Renal cell
•4
carcinoma, and Hepatocellular carcinoma.
•5 SITE OF METASTASIS 1• TUMOR NOTES
•6 Brain Lung> breast> melanoma, colon, kidney. 50% of brain tumors are from metastases t:i.1rn.
•7 Commonly seen as multiple well-circumscribed
•8 tumors at gray/white matter junction .
•9 Liver Colon >>stomach >pancreas. Liver l!l (!] and lung are the most common sites
• 10 of metastasis after the regional lymph nodes.
· 11 Bone Prostate, breast > lung, thyroid, kidney. Bone metastasis 0 D >> 1o bone tumors (eg,
multiple myeloma, lytic). Common mets to
• 12
bone: breast (m ixed), lung (lytic), thyroid
• 13
(lytic), kidney (lytic), prostate (blastic) .
• 14 Predilection for axia I skeleton (!!.
• 15
• 16
• 17

• 18
• 19
• 20
• 21

6
lock
s
Suspend
0
End Block
Item: 3 of 33 ~ 1 • Ma r k -<:J I> ~ £!1}>'
• !!":-~
QIO: 3186 ..L Prev ious Next Lab lues Notes Cal culat o r

& &
1

3
·4
•5
•6
.7
·8
.9
• 10
• 11

• 12
• 13
• 14
• 15
• 16
• 17
. 18
• 19
• 20
FA17p438.1
• 21 Primary bone tumors

a
Lock
s
Suspend
8
End Bl ock
Item:3of33 ~. , . M k <:] t> al ~· ~
QIO: 3186 .l. ar Previous Next Lab 'lifllues Notes Calculator

6
lock
s
Suspend
0
End Block
Item: 3 of 33 ~ 1 • M k -<:J 1>- Jil ~· !:';-~
QIO: 3186 ..L ar Pre v ious Next Labfli!llues Notes Calcula t o r
& &
1
Ewing sarcoma Boys< 15 years old. Anaplastic small blue cellmalignanl tumor 1!].
2
Commonly appears in diaphysis of long bones, Extremel}' aggressive with early metastases, but
3 pelvis, scapula, ribs. responsi,·e to chemotherapy.
·4 "Onion skin" periosteal reaction in bone.
•5 Associated with t(ll ;22) translocation causing
•6
fusion protein E\VS-F'LI I.
11 + 22 = 33 (Patrick E" ing's jerse~ number).
.7
·8
.9
• 10 Round cell lesions ___,_ _
Ewing sarcoma
• 11 Myeloma
• 12 / \ Fibrous dysplasia
• 13
Osteoid osteoma
• 14 (1"1' tllt.tnf p. n Cfflll nO.

• 15 Simple bone cyst


• 16
• 17
. 18 Osteochondroma
I
• 19 ...-=. _____ - - - - Physis
• 20
• 21
!{ ·a.
.....
1
a.....;___ Giant cell tumor
1:]

a
Lock
s
Suspend
8
End Bl ock
Item:3of33 ~. , . M k <:] t> al ~· ~
QIO: 3186 .l. ar Previous Next Lab 'lifllues Notes Calculator

1 • •

2 FA17 p217.1

3 Paraneoplastic syndromes
MANIFESTATION DESCRIPTION/MECHANISM MOST COMMONLY ASSOCIATED CANCER(S)
•4
•5
Cutaneous
•6 Acanthosis nigricans l lyperpigmented velvety plaques in axilla and Gastric adenocarci noma and other visceral
neck malignancies (but more commonly associated
•7
with obesity and insulin resistance)
•8
Sign of Leser-Trelat Sudden onset of multiple seborrheic keratoses C l adenocarcinomas and other visceral
•9
malignancies
• 10
Endocrine
· 11
Hypercalcemia PTHrP Squamous cell carcinomas of lung, head, and
• 12 neck; renal, bladder, breast, and ovarian
• 13 carcmomas
• 14 t 1,25-(0H)2 vitamin 0 3 (calcitriol) Lymphoma
• 15 Cushing syndrome t ACTI I
Small cell lung cancer
• 16 Hyponatremia (SIADH) t ADII
• 17 Hematologic
• 18 Polycythemia t Erythropoietin Renal cell carcinoma, hepatocellular
• 19 carcinoma, hemangioblastoma,
• 20 pheochromoC)torna, leiomyoma
• 21 Pure red cell aplasia Anemia with low reticulocytes •
• Tl-, , , ...,n"•,. ••

6
lock
s
Suspend
0
End Block
Item: 3 of 33 ~ 1 • M k -<:J 1>- Jil ~· !:';-~
QIO: 3186 ..L ar Pre v ious Next Labfli!llues Notes Calcula t o r

1 Good syndrome Tlypogammaglobulmem1a
2 Trousseau syndrome 1\ ligratory superficial th rombophlebit is
3 Nonbacterial Deposition of sterile platelet thrombi on heart Adenocarcinomas, especially pancreatic
·4 thrombotic (marantic) valves
•5 endocarditis
•6 Neuromuscular
•7 Anti-NMDA receptor Psychiatric disturbance, memOT) deficits, o ,·arian teratoma
·8 encephalitis seizures, drskinesias, autonomic instability,
.9 language dysfunction
• 10 Opsoclonus- "Dancing eyes, dancing feet" euroblastoma (children), smaJI eel I lung
myoclonus ataxia cancer (adults)
• 11
syndrome
• 12
Paraneoplastic Antibodies against antigens in Purkinje cells Small cell lung cancer (anti-Hu), gynecologic
• 13
cerebellar and breast cancers (anti-Yo), and Hodgkin
• 14 degeneration lymphoma (anti-Tr)
• 15
Paraneoplastic Antibodies against l-lu antigens in neurons Small cell lung cancer
• 16 encephalomyelitis
• 17 Lambert-Eat on Antibodies against presynaptic (P/Q -type) Ca 2+ Small cell hmg cancer
• 18 myasthenic syndrome channels at MJ
• 19 Myasthenia gravis Antibodies aga inst postsrnaptic ACh receptors Thymoma
• 20
• 21

at N:VIJ
I
a
Lock
s
Suspend
8
End Bl ock
Item: 4 of 33 ~ 1 • M k -<:J 1>- Jil ~· !:';-~
QIO: 3254 ..L ar Pre v ious Next Labfli!llues Not es Calcula t o r
& &
1
An 8-year-old boy is brought to his ped iatrician by his parents because of swelling and pain over his right
2
femur for the past 2-3 weeks. The ch ild and parents deny any history of trauma to the reg ion. The patient
3 reports the pain often is worse at night, and his mother states he has been having low-grade fevers of
•4 37.8°C-38.1 °C (100.0°F-100 .6°F) . There is no erythema of the region, but examination reveals a f irm and
•5
immobile mass that is tender to palpation over the proximal r ight lower extremity. Cytologic study with fine-needle
aspiration is performed, and the result is shown here .
•6
•7

·8
.9
• 10
• 11
• 12
• 13
• 14
• 15
• 16
• 17
• 18
• 19
• 20 Genetic evaluation of this patient is most likely to reveal which of the following?
• 21 :

a
Lock
s
Suspend
8
End Bl ock
2
3
•4
•5
•6
•7

·8
.9
• 10
• 11
• 12
Genetic evaluation of th is patient is most likely to reveal which of the following?
• 13
:
• 14
A. Dystrophin mutation
• 15
• 16
B. t(11;22) chromosomal translocation

• 17 C. t(12;2 1) chromosomal translocation


• 18 D. t(9; 22 ) chromosoma l translocation
• 19
E. XXV disorder
• 20
• 21

a
Lock
s
Suspend
8
End Block
Item:4of33 ~. , . M k <:] t> al ~· ~
QIO: 3254 .l. ar Previous Next lab 'lifllues Notes Calculator

1 •

2 The correct answer is B. 70°/o chose this.


3
The patient's presentation and histo logy are most consistent with Ewing sa rcoma, which often occurs in chi ldren
and adolescents and most often involves the pelvis and femur. On examination, one may discern a pa lpab le soft
4
tissue mass, which is often tender. Pain is common and is often worse at night and may be aggravated by
•5 exercise. Systemic find ings such as fever, fatigue, malaise and weight loss may be present in about a fifth of al l
•6 cases. On imaging, destructive lesions of the bone may occu r with a " moth -eaten" appea rance and the
character istic periosteal reaction may be present ("onion peel" appearance). Pathology often demonstrates
•7
smal l, round blue cel ls (l ike those shown in the image in the vignette), and a majority of cases will have
•8 t( 11; 22) t ranslocations.
•9 Histology Femur Ewing' s sarcoma Sarcoma Pelvis Pathology Periosteal reaction Soft tissue Weight loss Chromosomal translocation Fatigue (medical) Fever Bone

• 10
A is not correct. 4°/o chose this.
· 11 Dystrophin mutations resu lt in muscu lar dystrophy, wh ich manifests as progressive proximal muscle weakness.
• 12 Duchenne and Becker muscular dystrophies have this mutation. Duchenne patients usua lly are asymptomatic at
• 13
birth but develop weakness at 3-5 years of age. Patients usually die of cardiopulmonary pathologies in the
second and third decades of life. Becker muscular dystropy is similar clinica lly but presents later in chi ldhood,
• 14
and patients generally survive into adu lthood .
• 15 Muscular dystrophy Dystrophin Mutation Becker' s muscular dystrophy Muscle weakness Muscle Pathology Anatomical terms of location

• 16
C is not correct. 15% chose this .
• 17
Acute lymphoblastic leukemia (ALL) is associated with t( 12;21) translocation and is general ly not associated
• 18 with lower leg masses. ALL is the most common cancer in chi ldren, especial ly from ages 2-5 years. Patients
• 19 with ALL present with signs and symptoms of bone marrow failure ( like bleeding/bruising, fatigue, and
infections) .
• 20
Acute lymphoblastic leukemia leukemia Chromosomal translocation Bone marrow Aplastic anemia Cancer Bone Fatigue (medical)
• 21
. -. •• •

6
lock
s
Suspend
0
End Block
Item:4of33 ~. , . M k <:] t> al ~· ~
QIO: 3254 .l. ar Previous Next lab 'lifllues Notes Calculator

1 t( 11; 22) translocat ions.


Histology Femur Ewing's sarcoma Sarcoma Pelvis Pathology Periosteal reaction Soft tissue Weight loss Chromosomal translocation Fatigue (medical) Fever Bone
2

3 A is not correct. 4°/o chose this.


4 Dystrophin mutations resu lt in muscu lar dystrophy, wh ich manifests as progressive proximal muscle weakness.
•5 Duchenne and Becker muscular dystrophies have this mutation. Duchenne patients usua lly are asymptomatic at
birth but develop weakness at 3-5 years of age. Patients usually die of cardiopulmonary pathologies in the
•6
second and third decades of life. Becker muscular dystropy is similar clinica lly but presents later in chi ldhood,
.7 and patients generally survive into adu lthood .
•8 Muscular dystrophy Dystrophin Mutation Becker's muscular dystrophy Muscle weakness Muscle Pathology Anatomical terms of location

•9 C is not correct. 15% chose this .


• 10 Acute lymphoblastic leukemia (ALL) is associated with t( 12;21) translocation and is general ly not associated
· 11 with lower leg masses. ALL is the most common cancer in chi ldren, especial ly from ages 2-5 years. Patients
• 12
with ALL present with signs and symptoms of bone marrow failure ( like bleeding/bruising, fatigue, and
infections) .
• 13 Acute lymphoblastic leukemia leukemia Chromosomal translocation Bone marrow Aplastic anemia Cancer Bone Fatigue (medical)
• 14
D is not correct. 10% chose this .
• 15
The t(9;22) translocation is associated with chron ic myelogenous leukem ia (CML), which can cause
• 16 splenomegaly. However, the painful mass in the patient's thigh shou ld suggest Ewing sarcoma before CML.
• 17 Chronic myelogenous leukemia Splenomegaly leukemia Ewing's sarcoma Chromosomal translocation Sarcoma

• 18
E is not correct. 1°/o chose this .
• 19 XXY is Kl inefelter syndrome, a genetic disturbance of the sex chromosomes associated with testicu lar atrophy,
• 20 eunuchoid body shape, long extremities, and gynecomastia .
Gynecomastia Klinefelter syndrome Testicular atrophy Chromosome Atrophy Sex-determination system Eunuch Allosome
• 21

6
lock
s
Suspend
0
End Block
Item:4of33 ~. , . M k <:] t> al ~· ~
QIO: 3254 .l. ar Previous Next lab 'lifllues Notes Calculator
- e I • I I.- .1 - I - .- .. _ I I • l'f: liT • • 't: II I- - •- I Ill
1
and pat ients general ly su rv ive int o adu lt hood.
2 Muscular dystrophy Dystrophin Mutation Becker' s muscular dystrophy Muscle weakness Muscle Pathology Anatomical terms of location
3
C is not correct. 15% chose this.
4
Acute lymphobl astic leukemia (ALL) is associated with t ( 12;2 1) translocation and is general ly not associated
•5 wit h lower leg masses. ALL is the most common cancer in chi ldren, especial ly from ages 2- 5 years . Patients
•6 wit h ALL present with signs and symptoms of bone ma r row failure ( like bleeding/bru ising, fatigue, and
•7 infections) .
Acute lymphoblastic leukemia leukemia Chromosomal translocation Bone marrow Aplastic anemia Cancer Bone Fatigue (medical)
•8
•9 D is not correct. 10% chose this .
• 10 The t(9;22) t ranslocat ion is associated wit h chron ic myelogenous leukem ia (CML), which can cause
splenomega ly . However, the painfu l mass in t he patient 's thigh shou ld suggest Ewing sarcoma before CM L.
· 11
Chronic myelogenous leukemia Splenomegaly leukemia Ewing' s sarcoma Chromosomal translocation Sarcoma
• 12
E is not correct. 1°/o chose this .
• 13
XXY is Klinefelter syndrome, a genetic disturbance of the sex ch romosomes associated with testicu lar atrophy,
• 14
eunuchoid body shape, long extremities, and gynecomastia .
• 15 Gynecomastia Klinefelter syndrome Testicular atrophy Chromosome Atrophy Sex-determination system Eunuch Allosome

• 16
• 17
Bottom Line:
• 18
• 19
Ewing sarcoma is diagnosed histolog ica lly with blue anaplastic cells and is associated with t( 11;22)
t ranslocat ions .
• 20 Ewing' s sarcoma Histology Sarcoma Chromosomal translocation Anaplasia

• 21

6
lock
s
Suspend
0
End Block
Item:4of33 ~. , . M k <:] t> al ~· ~
QIO: 3254 .l. ar Previous Next Lab 'lifllues Notes Calculator

1 • •
FA17 p438.1
2 Primary bone tumors
3 TUMOR TYPE EPIDEMIOLOGY/LOCATION CHARACTERISTICS
4 Benign tumors
•5 Osteochondroma Most common ben ign bone tumor. Bony exostosis with cartilaginous (chondroid)
•6 Males < 25 years old. cap fl.
•7
Rarely transforms to chondrosarcoma.
•8 Giant cell tumor 20-40 years old. Locally aggressive benign tumor.
Epiphysis of long bones (often in knee region). "Soap bubble" appearance on x-ray (l].
•9
Arises most commonly at distal femur and Multinucleated giant cells that express RANKL.
• 10 proximal tibia.
· 11 "Osteoclastoma."
• 12 Malignant tumors
• 13 Osteosarcoma One of the most common malignant bone Cod man triangle (from elevation of periosteum)
• 14 (osteogenic sarcoma) tumors. or sunburst pattern on x-ray.
• 15 Bimodal distribution: 10-20 years old 00), > 65 Aggressive. Treat with surgical en bloc resection
• 16
(20) . (with limb sakage) and chemotherapy.
Predisposing factors: Paget disease of bone, bone
• 17
infarcts, radiation, fam ilial retinoblastoma,
• 18 Li-Fraumeni syndrome (germline p53
• 19 mutation).
• 20 Metaphysis of long bones, often around knee [!l
• 21 Ewing sarcoma Boys< 15 years old. Anaplastic small blue cell malignant tumor [!]. •

6
lock
s
Suspend
0
End Block
Item: 5 of 33 ~ 1 • M k -<:J 1>- Jil ~· !:';-~
QIO: 5 0 76 ..L ar Pre v ious Next Lab fli!ltues Not es Calcula t o r

IAA]
& &
1
A 78-year-old woman complains of worsening headaches on the r ight side of her head, as wel l as blurred
2
vision in the right eye. These symptoms started 6 weeks ago, when she first noticed blurriness whi le trying
3 to read a book at night. Since then she has noticed increasing blurriness and headache when reading the
4 newspaper or fine print. She also complains of general fatigue and stiffness in her neck and shoulders upon
•5
awakening in the morning for the past month . On physical examination she has a low-grade fever and appears
fatigued. Muscle strength in all areas is 5/5 .
•6
•7
What is the most appropriate treatment for this condition?
·8
:
.9
A. Corticosteroids
• 10
B. Methotrexate
• 11
• 12 C. Nonsteroidal anti-inflammatory drugs
• 13 D. Physical therapy
• 14
E. Vitamin A supplement s
• 15
• 16
• 17
• 18
• 19
• 20
• 21

a
Lock
s
Suspend
8
End Bl ock
Item:5of33 ~. , . M k <:] t> al ~· ~
QIO: 5076 .l. ar Previous Next lab 'lifllues Notes Calculator

1 •
The correct answer is A. 65°/o chose this.
2
This woman has symptoms of tempo ral (giant cell) arteritis ( GCA) and accompanying po lymya lgia rheumatica
3 (PMR). Tempo ral arteritis is a vascul itis of la rge and medium-sized vesse ls, and co mmon ly causes bl indness
4 un ilateral ly or bilateral ly if untreated (due to involvement of the poste rio r ci liary and ophtha lmic arte ries).
Patients often complain about fatigue, headache, and blurriness in the eyes. They may also present with low-
5
grade fevers and jaw/arm claudication . I n PMR, patients have symptoms of muscle stiffness for at least 1
•6 month. The best initial treatment for both conditions is a t r ial of corticosteroids. However, if GCA is suspected,
•7 treatment with high -dose ste roids shou ld be started promptly, even before the diag nosis is confirmed, due to
•8 the r isk of blindness . Urgency shou ld be even higher in patients already presenting with signs of v isua l
impa irment .
•9
Polymyalgia rheumatica Giant-cell arteritis Visual impairment Vasculitis Corticosteroid Claudication Headache Giant cell Artery Fatigue (medical) Arteritis Steroid
• 10
Muscle Ophthalmology
· 11
B is not correct. 8°/o chose this .
• 12
Methotrexate and other immunosuppressive dr ugs may be used to t reat po lymyalgia rheumatica (PMR);
• 13
however, they are secondary to corticosteroid t herapy , as the re are no clea r data suggesting that methotrexate
• 14 is bette r than corticosteroids. The fi rst-line therapy is corticosteroids, w hich would t reat PMR and gia nt cell
• 15 arter itis .
Corticosteroid Polymyalgia rheumatica Methotrexate Giant-cell arteritis Immunosuppression Immunosuppressive drug Giant cell
• 16
• 17 C is not correct. 15% chose this .
• 18 Nonste roidal anti-i nflamm atory drugs (N SAIDs) would be appropriate for t reatm ent of pai n associated with
• 19
po lymyalgia rheumatica ( PMR) or tempo ral arter itis, but t hey would not cure the cond it ion. NSAIDs are more
usefu l during corticoste roid tapering for t reatment of pain .
• 20 Corticosteroid Polymyalgia rheumatica Giant-cell arteritis Nonsteroidal anti -inflammatory drug Anti -inflammatory
• 21

6
lock
s
Suspend
0
End Block
Item:5of33 ~. , . M k <:] t> al ~· ~
QIO: 5076 .l. ar Previous Next lab 'lifllues Notes Calculator

1 •
Corticosteroid Polymyalgia rheumatica Methotrexate Giant-cell arteritis Immunosuppression Immunosuppressive drug Giant cell

2
C is not correct. 15% chose this.
3
Nonsteroidal anti-inflammatory drugs (NSAIDs) would be appropriate for treatment of pain associated with
4 po lymya lgia rheumatica (PMR) or temporal arter itis, but they would not cure the cond it ion. NSAIDs are more
5 usefu l during corticoste roid tapering for t reatment of pain.
Corticosteroid Polymyalgia rheumatica Giant-cell arteritis Nonsteroidal anti -inflammatory drug Anti -inflammatory
•6
•7 D is not correct. 3°/o chose this.
•8 Physica l therapy would be useful to regain function of a muscle group, but it wou ld not be a solution to morning
•9 stiffness caused by polymya lgia r heumatica (PMR). Additional ly, this patient has symptoms of tempora l arterit is,
which is not improved with physical the rapy .
• 10
Polymyalgia rheumatica Giant-cell arteritis Physical therapy Joint stiffness Muscle
· 11
E is not correct. 9°/o chose this .
• 12
Although night blindness is a symptom of vitamin A deficiency, there is no evidence that this patient's eye
• 13
symptoms are due to vitamin A deficiency. Add it ionally, she does not descr ibe night bl indness, but rather
• 14 blurriness when read ing f ine print or newspapers. Vitamin A supplements are thus not an app ropr iate t reatment
• 15 for her condition .
Vitamin A Vitamin A deficiency Nyctalopia Vitamin Visual impairment Symptom
• 16
• 17

• 18 Bottom Line:
• 19 The best treatment for temporal (giant ce ll) arter itis is rapid initiation of high-dose corticoste roids. Polymyalgia
• 20 r heumatica in the absence of giant cel l arterit is is treated with low-dose corticosteroids .
Polymyalgia rheumatica Giant-cell arteritis Giant cell Corticosteroid Arteritis
• 21

6
lock
s
Suspend
0
End Block
Item:5of33 ~. , . M k <:] t> al ~· ~
QIO: 5076 .l. ar Previous Next lab 'lifllues Notes Calculator

6
lock
s
Suspend
0
End Block
Item: 6 of 33 ~ 1 • M k -<:J 1>- Jil ~· !:';-~
QIO: 3921 ..L ar Pre v ious Next Lab fli!ltues Notes Calcula t o r
& &
1
A 52-year-old secretary presents to the cl inic compla ining of chron ic and sharp pain in her right f inger joints
2
over the past year. Her finger joints are stiff for around 30 m inutes in the morning. The stiffness generally
3 improves with rest and worsens with activity. She denies fevers, weight loss, fatigue, and weakness.
4 Physical examination reveals bony growth at her distal and proximal interphalangeal joints. Crepitus is heard with
active and passive movements. The rest of her physical exam is unremarkable.
5
•6 Mean
Choice Hemoglobin corpuscular
.7
(g/dl) volume (fL)
·8
.9 A 9.5 76

• 10 B 9.5 95
• 11

• 12
c 9.5 101

• 13 0 13.4 95
• 14
E 17.8 95
• 15
• 16
• 17 Wh ich of the laboratory va lues in the table are most consistent with this patient's condition?
. 18 :
• 19 A
• 20
B
• 21

a
Lock
s
Suspend
8
End Bl ock
Item: 6 of 33 ~ 1 • M k -<:J 1>- Jil ~· !:';-~
QIO: 3921 ..L ar Pre v ious Next Lab fli!ltues Notes Calcula t o r
& &
1
Mean
2 Choice Hemoglobin corpu scular
(g/dl) volume (fL)
3
4 A 9.5 76
5
B 9.5 95
•6
•7 c 9.5 101
·8
D 13.4 95
.9
• 10 E 17.8 95
• 11
• 12
Which of the laboratory values in the table are most consistent w ith th is patient's cond ition?
• 13
:
• 14
A
• 15
• 16
8

• 17 c
• 18 D
• 19
E
• 20
• 21

a
Lock
s
Suspend
8
End Bl ock
Item: 6 of 33 ~. I • M k <:] t> al ~· ~
QIO: 3921 .l. ar Previous Next lab 'lifllues Notes Calculator

1 •

2 The correct answer is D. 51°/o chose this.


3 Osteoarthritis, also known as degenerative joint disease, is a chronic non inflammatory joint disease. It is the
most common form of arth r itis. Risk factors for osteoarthritis include female gender, obesity, age >50 years,
4
and chronic mechanical trauma to the affected joints. The mechan ical trauma induced by this patient's work
5 puts her at risk for osteoarthritis. The disease is characterized by loss of cartilage elasticity, polished ivory-like
6 appearance of bone (eburnation), cystic changes in the subchondral bone, and osteophyte for mation at the
•7 perimeter of the articular surface. Osteoarthritis often affects the sma ller joints, causing bony spu rs or
osteophytes at the dista l inter phalangea l joints (Heberden nodes) and the proxima l inter phalangea l joints
•8
(Bouchard nodes). Patients also lack systemic symptoms like the fevers and weight loss commonly associated
•9 with r heumatoid arthritis . Without other comor bidities, this patient most likely has a nor mal hemog lobin level
• 10 and mean corpuscu lar volume .
Osteophyte Osteoarthritis Rheumatoid arthritis Epiphysis Hemoglobin Cartilage Arthritis Obesity Interphalangeal articulations of hand Weight loss
· 11
Anatomical terms of location Comorbidity Major trauma Bone
• 12
• 13 A is not correct. 17% chose this .
• 14 These lab values indicate microcytic anemia. Causes of microcytic anemia include iron deficiency (from
• 15
decreased intake or increased loss through bleeding), occasional ly chronic inflammatory disease (such as
r heumatoid arthritis), thalassemia, and sideroblastic anemia. This patient's symptoms are more consistent with
• 16 osteoarthritis; therefore she wou ld not be expected to have microcytic anemia.
• 17 Microcytic anemia Rheumatoid arthritis Osteoarthritis Thalassemia Sideroblastic anemia Anemia Iron deficiency Iron Arthritis Inflammation

• 18
B is not correct. 17% chose this .
• 19 These lab values indicate nor mocytic anemia . Causes of normocytic anemia include acute blood loss, aplastic
• 20 anemia, chronic renal failure, hemolytic anemia, hereditary spherocytosis, sickle cell anemia, glucose-6-
phosphate dehydrogenase deficiency, anemia of chronic disease (AOCD), and paroxysmal nocturnal
• 21
• L...---- 1-L-. : - . . .... : - 1\.r*\rr-"\ .... --··1•- c .... __ -L.- . . - ~ ---- _c --· ·•- _~,... ___ .... ---•--•- ---- :&:-- 1 1.~ L..--- : ...1:- . •. L.. : -L... - - - · · - - • - ...-

6
lock
s
Suspend
0
End Block
Item: 6 of 33 ~. I • M k <:] t> al ~· ~
QIO: 3921 .l. ar Previous Next lab 'lifllues Notes Calculator
-. - •• - - - - - - • • - •• - - - - • - - 'f • • - - • - • - ,..
1
osteoarthritis; therefore she wou ld not be expected to have microcytic anemia.
2 Microcytic anemia Rheumatoid arthritis Osteoarthritis Thalassemia Sideroblastic anemia Anemia Iron deficiency Iron Arthritis Inflammation

3
B is not correct. 17% chose this.
4
These lab values indicate nor mocytic anemia . Causes of normocytic anemia include acute blood loss, aplastic
5 anemia, chronic renal failure, hemolytic anemia, hereditary spherocytosis, sickle cell anemia, glucose-6-
6 phosphate dehydrogenase deficiency, anemia of chronic disease (AOCD), and paroxysmal nocturnal
•7 hemog lobinuria . AOCD results from the release of acute-phase reactants, specifica lly hepcidin, wh ich sequesters
iron in storage sites and decreases the amount of available iron, thus leading to a state of normocytic anem ia .
•8
These features of AOCD are not associated with osteoarthritis .
•9 Paroxysmal nocturnal hemoglobinuria Anemia of chronic disease Hereditary spherocytosis Aplastic anemia Sickle-cell disease Osteoarthritis Hepcidin

• 10 Hemolytic anemia Glucose-6-phosphate dehydrogenase deficiency Anemia Glucose-6-phosphate dehydrogenase Chronic kidney disease Hemolysis Chronic condition

· 11 Normocytic anemia Bleeding Acute-phase protein Kidney Hemoglobinuria Iron Spherocytosis

• 12
C is not correct. 12% chose this .
• 13
These laboratory values indicate macrocytic anemia . Causes of macrocytic anemia include vitamin B12
• 14 deficiency, folate deficiency, and pernicious anemia . Th is patient's primary symptoms are consistent with
• 15 osteoarthritis, wh ich is not associated with macrocytic anemia .
Pernicious anemia Macrocytic anemia Folate deficiency Anemia Folic acid Vitamin 812 Vitamin Macrocytosis B vitamins
• 16
• 17 E is not correct. 3°/o chose this .
• 18 These lab values indicate polycythemia, which is generally associated with chronic hypoxia or, in some cases,
• 19 polycythemia vera . It is not associated with osteoarth r itis .
Polycythemia vera Hypoxia (environmental) Hypoxia (medical) Osteoarthritis Polycythemia
• 20
• 21

6
lock
s
Suspend
0
End Block
Item: 6 of 33 ~. I • M k <:] t> al ~· ~
QIO: 3921 .l. ar Previous Next lab 'lifllues Notes Calculator

1 • phosphate dehydrogenase deficiency, anemia of chronic disease (AOCD), and paroxysmal nocturnal
2
hemog lobinuria . AOCD results from the release of acute-phase reactants, specifica lly hepcidin, wh ich sequesters
iron in storage sites and decreases the amount of available iron, thus leading to a state of normocytic anem ia.
3 These features of AOCD are not associated with osteoarthritis.
4 Paroxysmal nocturnal hemoglobinuria Anemia of chronic disease Hereditary spherocytosis Aplastic anemia Sickle-cell disease Osteoarthritis Hepcidin

5 Hemolytic anemia Glucose-6-phosphate dehydrogenase deficiency Anemia Glucose-6-phosphate dehydrogenase Chronic kidney disease Hemolysis Chronic condition

6 Normocytic anemia Bleeding Acute-phase protein Kidney Hemoglobinuria Iron Spherocytosis

•7
C is not correct. 12% chose this .
•8
These laboratory values indicate macrocytic anemia . Causes of macrocytic anemia include vitamin B12
•9 deficiency, folate deficiency, and pernicious anemia . Th is patient's primary symptoms are consistent with
• 10 osteoarthritis, wh ich is not associated with macrocytic anemia .
Pernicious anemia Macrocytic anemia Folate deficiency Anemia Folic acid Vitamin 812 Vitamin Macrocytosis B vitamins
· 11
• 12 E is not correct. 3°/o chose this .
• 13 These lab values indicate polycythemia, which is generally associated with chronic hypoxia or, in some cases,
• 14
polycythemia vera . It is not associated with osteoarthritis .
Polycythemia vera Hypoxia (environmental) Hypoxia (medical) Osteoarthritis Polycythemia
• 15
• 16
• 17 Bottom Line:
• 18 Osteoarthritis is a chron ic noninflammatory joint disease, whose risk factors include female gender, obesity,
• 19
age >50 years, and chronic mechanica l trauma . It does not cause abnormalities in hemoglobin level or
corpuscular volume .
• 20 Osteoarthritis Hemoglobin Obesity Arthritis Arthropathy
• 21

6
lock
s
Suspend
0
End Block
Item: 6 of 33 ~ 1 • M k -<:J 1>- Jil ~· !:';-~
QIO: 3921 ..L ar Pre v ious Next Lab fli!ltues Notes Calcula t o r
& &
1
2 FA17 p 439.1

3 Osteoarthritis and rheumatoid arthritis


4 Osteoarthritis Rheumatoid arthritis
5 PATHOGENESIS \ lechanical- wear and tear dcstroys articular Autoimmune-inflammation induces formation
6 cartilage (degenerati' c joint disorder) of pannus (proliferati,·e granulation tissue ),
- inflammation with inadequate repair. which erodes articular cartilage and bone.
0 7
Chondroc}1es mediate degradation and
o8
inadequate repair.
.9
PREDISPOSING FACTORS Age, female, obesity, joint trauma. Female, HLA-DR4, smoking, silica exposure.
• 10 ® rheumatoid factor (lgM antibody that
• 11 targets lgG Fe region; in 80%), anti-cyclic
• 12 citrullinated peptide antibody (more specific).
• 13 PRESENTATION Pain in weight-bearing joints after usc (cg, Pain, swelling, and morning stiffness lasting
0 14 at the end of the day), improving with rest. > 1 hour, improving with use. Symmetric
Asymmetric joint involvement. Knee carti lage joint involvement. Systemic symptoms
• 15
loss begins medially ("bowlegged"). o (fever, fatigue, weight loss). Extraarticular
0
16 systemic symptoms. manifestations common."
0
17
JOINT FINDINGS Osteophytes (bone spurs), joint space narrowing, Erosions, juxta-articular osteopenia, soft tissue
• 18 subchondral sclerosis and cysts. S) novial swelling, subchondral cysts, joint space
• 19 fluid non-inflammatory (WBC < 2000/mm 3). narrowing. Deformiti es: cervica I subluxation,
• 20 llwolves DIP (Hebcrdcn nodes : ) and PIP ulnar fin ger deviation, swan neck rn.
• 21
(Bouchard nodes B), and 1st C MC; not .MCP. boutonniere 0 . lnm h-es i\IC P, PIP, wrist; not

a
Lock
s
Suspend
8
End Bl ock
Item: 6 of 33 ~ 1 • M k -<:J 1>- Jil ~· !:';-~
QIO: 3921 ..L ar Pre v ious Next Lab fli!ltues Notes Calcula t o r
& &
1 JOINT FINDINGS Osteophytes (bone spurs), joint space narrowing, Erosions, juxta-articular osteopenia, soft I issue
2 subchondral sclerosis and cysts. S) novial swelling, subchondral cysts, joint space
3 Auid non-inflammatory (\.VBC < 2000/mm3). narrowing. Deformities: cen·ica I subluxation,
4
lm·okes DIP (Heberden nodes ) and PIP ulnar finger deviation, swan neck rn.
(Bouchard nodes ~). and lsi C .\ltC; not ~ J CP. boutonniere 1). ln\'Okes ~ICP, PIP, wrist; not
5
DIP or lst C~ IC. Svnovial
'
Auid inAammator\'. .
6
TREATMENT Acetaminophen, 1'\SAIDs, intra-articular $,\( Ds, glucocorticoids, disease-modifying
7
0
glucocorticoids. agents (methotrexate, sulfasalazine,
o8 hydroxych loroquine, leAunomide), biologic
.9 agents (eg, T ' F-a inhibitors).
• 10 •f:xtraarticular manifestations include rheumatoid nodules (fibrinoid necrosis with palisading histiocytes) in subcutaneous
• 11 tissue and lung (+ pneumoconiosis - Caplan syndrome), interstitial lung disease, pleuritis, pericarditis, anemia of chronic
• 12
disease, neutropenia+ splenomegaly (Felty syndrome), A amyloidosis, Sjogren syndrome, scleritis, carpal tunnel syndrome.
• 13 Normal Osteoarthritis Normal Rheumatoid
arthritis
0 14 - - - - - T hickened
capsule Boneand

Y
• 15 carti~age
Slight synovial
/ erosion
Joint capsule ""'\. Joint capsule ""'\.
0
16 and synovial '\ / . hypertrophy and synovial '\
lining ..----Osteophyte lining
0
17 - - - Increased
Synovial _ / - - - U lcerated Synovial _ / synovial fluid
. 18 ~ cartilage cavity ./'
cavity/ _.--- Pannus
• 19 Cartilage ~ ~ Sclerotic bone Cartilage / formation
JOtnt space
• 20 narroWing
• 21 Subchondral
• honPrv~

a
Lock
s
Suspend
8
End Bl ock
Item: 6 of 33 ~ 1 • M k -<:J 1>- Jil ~· !:';-~
QIO: 3921 ..L ar Pre v ious Next Lab fli!ltues Notes Calcula t o r
! ! I I I
1

2
*F.:xtraarticular manifestations include rheumatoid nodules (fibrinoid necrosis with pa lisad ing histiocytes) in subcutaneous
tissue and lung (+ pneumoconiosis - Caplan syndrome), interstitial lu ng disease, pleuritis, pericarditis, anemia of chronic
3
disease, neutropenia+ splenomegaly (Felty syndrome), AA amyloidosis, Sjogren syndrome, scleritis, carpal tunnel syndrome.
4
Normal Osteoarthritis Normal Rheumatoid
5 arthritis
- - - - - ThiCkened
6 capsule
=~~
/
0 7 Slight syllOVlal
/
Joint capsule """'\. Jomt capsule """'\. erosion
o8 and synovial \, / _hypertrophy and syn<Mal \,
lming - - -Osteophyte IIrung
.9 - - - Ulcerated
. - - -Increased
Synovial / SyllOVlal _ / SyllOVlal RUid
• 10 ~ cart1lage CaVIty /
cavity/ __..-- Pannus
Cartilage ~ ~ SclerOIJc bone Cartilage. / forma~on
• 11 Jomtspace
• 12 narrowing
Subchondral
• 13 bone cyst
0 14
• 15
0
16
0
17
• 18
• 19
• 20
• 21

a
Lock
s
Suspend
8
End Bl ock
Item: 7 of 33 ~. I • M k <:] t> al ~· ~
QIO: 3922 .l. ar Previous Next lab 'lifllues Notes Calculator

1 •
A 12-year-old boy comes to the doctor because of high spiking fevers for the past several weeks. The fevers
2
are periodic, with his temperature returning to normal between episodes. He does not have a history of
3 t ravel outside the United States, and does not report diarrhea or constipation. A macular, sa lmon-pink rash is
4 present on his axillae and around his waist, and the intensity of the rash coincides with the intensity of his fever.
He exh ibits hepatosplenomega ly and genera lized lymphadenopathy. Peripheral blood smea r shows norma l RBCs
5
and RBC- Iaden macrophages. Laboratory tests show:
6
•7 RBC count: 1,700,000/mm 3
•8 Hematocrit: 18%
Hemoglobin : 5.2 g/dL
•9
WBC count: 30,000/mm 3
• 10

· 11
• 12 What is another typica l f inding in th is boy's illness?
• 13 :
• 14 A. An autosomal recessive inherited gene mutation
• 15
B. I nflammation of the gastrointestinal tract limited to the mucosa and submucosa
• 16
C. I ntraerythrocytic parasitic protozoa
• 17

• 18 D. Polyarticular joint pain and swelling


• 19 E. Transmural inflammation of the gastrointestinal t ract
• 20
• 21

6
lock
s
Suspend
0
End Block
Item: 7 of 33 ~. I • M k <:] t> al ~· ~
QIO: 3922 .l. ar Previous Next lab 'lifllues Notes Calculator

1 •
The correct answer is D. 48°/o chose this.
2 This is a classic presentation of systemic juven ile idiopathic arthrit is (J IA), the most common arthrit is seen in
3 pediatric patients. Systemic JIA often presents as recurrent and relapsing spiking fevers accompanied by
4
po lyarticu lar joint pain and an evanescent macu lar, salmon-pink rash present anywhere on the body.
Rheumatoid factor is not usua lly present. JIA is also associated with chronic anterior uveitis and decreased
5 growth rates, and 30% -50% develop hemophagocytic synd rome, in which macrophages beg in digesting RBCs
6 (which is the cause of this patient's low hematocr it). Treatment consists of nonsteroida l anti-inflammatory
7 drugs, corticosteroids, methotrexate, intravenous immune globu lin, and newer biolog ic dr ugs such as
etanercept, which acts as an inh ibitor of tumor necrosis factor. I n adults, systemic JIA is also known as Still
•8
disease .
•9 Etanercept Methotrexate Rheumatoid factor Juvenile idiopathic arthritis Hematocrit Uveitis Corticosteroid Arthralgia Tumor necrosis factors Macrophage

• 10 Nonsteroidal anti-inflammatory drug Idiopathy Rash Intravenous therapy Arthritis Anti-inflammatory Tumor necrosis factor alpha Biopharmaceutical Neoplasm

· 11 Necrosis Pediatrics Antibody

• 12
A is not correct. 13% chose this .
• 13
Fami lial Med iter ranean fever is an autosoma l recessive inherited disorder occurr ing in persons of eastern
• 14 Mediterranean origin . Recu r rent abdominal pain and eventual amyloid deposition, but not rash and
• 15 hepatosplenomegaly, are characteristic of familial Mediterranean fever .
Familial Mediterranean fever Hepatosplenomegaly Dominance (genetics) Autosome Fever Genetic disorder Abdominal pain Mediterranean Sea Rash Recessive
• 16
Amyloid
• 17

• 18 B is not correct. 10% chose this .


• 19 Ulcerative colitis is characterized by inflammation of the gastrointestinal tract limited to the mucosa and
• 20 submucosa, the presence of pseudopolyps in the colon, and chronic diarrhea associated with the passage of
blood and mucus. The lack of bloody diarrhea in this patient argues against a diagnosis of ulcerative colitis .
• 21 Ulcerative colitis Human qastrointestinal tract Submucosa Diarrhea Gastrointestinal tract Colitis Mucus Colon (anatomy) Mucous membrane Inflammation

6
lock
s
Suspend
0
End Block
Item: 7 of 33 ~. I • M k <:] t> al ~· ~
QIO: 3922 .l. ar Previous Next lab 'lifllues Notes Calculator
:, I I ... I • I ...
1
Ulcerative colitis is characterized by inflammation of the gastrointestinal tract limited to the mucosa and
2
submucosa, the presence of pseudopolyps in the colon, and chronic diarrhea associated with the passage of
3 blood and mucus. The lack of bloody diarrhea in this patient argues against a diagnosis of ulcerative colitis.
4 Ulcerative colitis Human gastrointestinal tract Submucosa Diarrhea Gastrointestinal tract Colitis Mucus Colon (anatomy) Mucous membrane Inflammation

5 C is not correct. 22% chose this.


6 Intraerythrocytic parasitic protozoa are found in malaria and Babesia infection. Although ma laria may cause
7 cyclic fevers, the lack of a travel history outside of the US makes this diagnosis less likely. However, Babesia is
•8 a tick-borne parasite that is common in the US. I t can infect RBCs and may lead to high fevers and hemo lysis .
However, it wou ld be apparent on peripheral blood smear, and a rash is not a common feature of babesiosis .
•9
Babesiosis Protozoa Malaria Babesia Hemolysis Blood film Parasitism Red blood cell Infection Rash
• 10
E is not correct. 7°/o chose this.
· 11
Crohn disease resu lts from transmural inflammation of the gastrointestinal t ract wall. Although Crohn disease
• 12
can occu r in adolescents, the presenting symptoms are usual ly abdomina l pain and bloody diarr hea, wh ich are
• 13 not present in this patient. Crohn can affect any portion of the gastrointestinal t ract. Biopsy wi ll show
• 14 noncaseating granu lomas . The compl ications of Crohn include strictu res, fistulas, perianal disease,
• 15
ma labsorption, and nutrit iona l depletion .
Crohn' s disease Human gastrointestinal tract Diarrhea Granuloma Malabsorption Biopsy Abdominal pain Gastrointestinal tract Anus Inflammation
• 16
• 17

• 18 Bottom Line:
• 19 Systemic JIA, the most common arthrit is in the pediatric popu lation, often presents with joint pain/limping,
• 20 spiking fevers, and swe lling and rash. Treatment includes nonsteroidal anti-inflammatory drugs, steroids,
methotrexate, and intravenous immune globu lin .
• 21 Methotrexate Nonsteroidal anti-inflammatory drug Intravenous therapy Arthritis Steroid Anti-inflammatory Rash Anabolic steroid Pediatrics Antibody

6
lock
s
Suspend
0
End Block
Item: 7 of 33 ~ 1 • M k -<:J 1>- Jil ~· !:';-~
QIO: 3922 ..L ar Pre vious Next Lab fli!ltues Notes Calculator
& &
1
FA17 p439.1
2 Osteoarthritis a nd rheumatoid a rthritis
3 Osteoarthritis Rheumatoid arthritis
4 PATHOGENESIS r-. rechanical-wear and tear destroys articular Autoimmune-inAammation induces format ion
5 cartilage (degenerati,·e joint disorder) of pannus (proliferati,·e granulation tissue rJ),
6 .... inflammation with inadequate repai r. which erodes articular cartilage and bone.
7
Chondrocytes mediate degradation and
inadequate repair.
·8
.9
PREDISPOSING FACTORS Age, female, obesity, joint trauma. Female, HLA-DR4, smoking, silica exposure.
(t) rheumatoid factor (IgM antibody that
• 10
targets lgG Fe region; in 80%), anti-cyclic
• 11 citrullinated peptide antibody (more specific).
• 12 PRESENTATION Pain in weight-bearing joints after use (eg, Pain, swelling, and morning stiffness lasting
• 13 at the end of the day), improving with rest . > I hour, impro,·ing with use. Symmetric
• 14 symmetric joint involvement. Knee carti lage joi nt involvement. Systemic symptoms
• 15 loss begins media lly ("bowlegged"). o (fever, fatigue, weight loss). Extraarticular
systemic symptoms. manifestations common."
• 16
JOINTFINDINGS Osteophytes (bone spurs), joint space narrowi ng, Erosions, juxta-articular osteopenia, soft tissue
• 17
subchondral sclerosis and cysts. Srnovial swelling, subchondral cysts, joint space
. 18
Auid non-inAammatory (WBC < 2000/mm ~). narrowing. Deformities: cervical subluxation,
• 19 Involves DIP (licberdcn nodes : ) and PIP ulnar finger deviation, swan neck rn.
• 20 (Bouchard nodes B ), and 1st C MC; not VIC P. boutonniere 0 . lm·oh-es r-. IC P, PIP, wrist; not
• 21 Dl P or Ist cr-. !C. Syno,·ial fluid inflammatory.

a
Lock
s
Suspend
8
End Block
Item:8of33 ~. , . M k <:] t> al ~· ~
QIO: 5090 .l. ar Previous Next lab 'lifllues Notes Calculator

1 •
A 68-year-old female with a medical history notable for obesity and osteoporosis presents to her primary
2 care physician with a chief compliant of daily pain in her knees. She describes the pa in as a deep ache that
3 worsens throughout the day, especial ly on days when she works outside in her garden. She finds relief with
4 rest and nonsteroidal anti-inflammatory drugs (NSAIDs). She denies recent trauma, prolonged morn ing stiffness,
joint erythema, or systemic symptoms. She currently takes supplements for osteoporosis management. The
5
physical examination is remarkable for some bony outgrowths on the medial aspect of the left knee, crepitus in the
6 right knee, and mild tenderness and effusions bi laterally. Active and passive range of motion is restr icted in both
7 knees.
•8
•9 What structural changes might one expect upon histologic inspection of the joint space of this patient's knees'
• 10 :
· 11 A. A fibrocellular mass of synovium and granulomatous tissue
• 12 B. Clusters of neutrophils in the synovium with needle-shaped crystals in their cytoplasm
• 13
C. Dislodged pieces of carti lage and subchond ral bone
• 14
• 15 D. I nfiltration of lymphoid fol licles, plasma cells, and macrophages
• 16 E. Neutrophils and dense, proteinaceous debris
• 17

• 18
• 19
• 20
• 21

6
lock
s
Suspend
0
End Block
Item:8of33 ~. , . M k <:] t> al ~· ~
QIO: 5090 .l. ar Previous Next lab 'lifllues Notes Calculator

1 •
The correct answer is C. 66°/o chose this.
2
This is a classic presentation of osteoarthritis, also known as degenerative joint disease. For reasons that are
3
not completely understood, but likely are in part due to wear and tea r over t ime, the superficial layers of the
4 articular carti lage of affected joints begin to degrade and are eventually sloughed off into the joint space. The
5 exposed bone becomes the new articular surface and is smoothed (eburnated) by continued joint usage. The
6
dislodged pieces of cartilage and subchondral bone form loose bodies in the joint space ("joint mice"), causing
pain and restriction of joint movement. Un like the other options, th is answer does not include evidence of frank
7
inflammation . The pathophysiology of osteoarth r itis is not inflammatory .
8 Osteoarthritis Epiphysis Cartilage Articular cartilage Inflammation Pathophysiology Articular bone Arthropathy Arthritis Bone

•9
A is not correct. 12% chose this .
• 10
This option describes a pannus formation, a str uctura l abnormality that occurs in r heumatoid arthritis . Th is
· 11 tissue, composed of inflammatory cells, synovium, and fibroblasts, eventual ly erodes the under lying cartilage,
• 12 and over time, fibroses and ossifies the joint into a fixed position .
Rheumatoid arthritis Synovial membrane Cartilage Fibroblast Arthritis Pannus Ossification
• 13
• 14 B is not correct. 6°/o chose this .
• 15 This is a description of the find ings present in gouty arthritis . Urate crystals, which are negatively birefringent
under polarized light, are taken up by neutroph ils, leading to pain and inflammation of the joint space .
• 16
Birefringence Gout Uric acid Neutrophil Inflammation Arthritis Polarization (waves)
• 17

• 18
D is not correct. 8°/o chose this .
This describes a generic inflammatory response in the joint space, characteristic of r heumatoid arthritis or
• 19
infectious arthritis. Osteoarthritic joints may occasionally show chronic inflammatory cells, particularly in severe
• 20 disease, but more typical ly do not have frank evidence of inflammation.
• 21 Rheumatoid arthritis Septic arthritis Osteoarthritis Arthritis Inflammation

6
lock
s
Suspend
0
End Block
Item:8of33 ~. , . M k <:] t> al ~· ~
QIO: 5090 .l. ar Previous Next lab 'lifllues Notes Calculator

1 • I I
J I p
Rheumatoid arthritis Synovial membrane Cartilage Fibroblast Arthritis Pannus Ossification
2
3 B is not correct. 6°/o chose this.
4 This is a description of the find ings present in gouty arthritis . Urate crystals, wh ich are negatively birefringent
under polarized light, are taken up by neutroph ils, lead ing to pain and inflammation of the joint space.
5
Birefringence Gout Uric acid Neutrophil Inflammation Arthritis Polarization (waves)
6
7
D is not correct. 8°/o chose this.
This describes a generic inflammatory response in the joint space, characteristic of r heumatoid arthritis or
8
infectious arthritis. Osteoarthritic joints m ay occasionally show chronic infla mmatory cells, particularly in severe
•9 disease, but m ore typical ly do not have frank evidence of inflam mation.
• 10 Rheumatoid arthritis Septic arthritis Osteoarthritis Arthritis Inflammation

· 11 E is not correct. 8°/o chose this .


• 12 This is a description of the purulent effusion that can be found in bacterial arthritis . Osteoarthrit ic joints do not
• 13 typical ly have evidence of frank inflammation unless there is a secondary process, such as inf ection or crystal
• 14 arthropathy .
Osteoarthritis Arthropathy Arthritis Crystal arthropathy Inflammation Crystal Pus
• 15
• 16
• 17 Bottom Line:
• 18 Osteoarthritis is a ch ron ic disease of degenerating joints in which pieces of carti lage and bone are broken off
• 19 into the joint space . Unlike other pathologic joint processes, there is not typically frank histologic evi dence of
inflammation .
• 20
Osteoarthritis Cartilage Chronic condition Histology Inflammation Bone Pathologic Pathology
• 21

6
lock
s
Suspend
0
End Block
Item: 8 of 33 ~ 1 • M k -<:J 1>- Jil ~· !:';-~
QIO: 5090 ..L ar Pre vious Next Lab fli!ltue s Notes Calcula tor
& &
1
2 FA17 p 439.1

3
Osteoarthritis and rheumatoid arthritis
4 Osteoarthritis Rheumatoid arthritis
5 PATHOGENESIS \ lechanical- wear and tear destroys articular Autoimmune-inflammation induces formation
cartilage (degenerati\ c joint disorder) of pannus (proliferati,·e granulation tissue ),
6
-+ inflammation with inadequate repair. which erodes articular cartilage and bone.
7
Chondroc}1es mediate degradation and
8 inadequate repair.
.9 PREDISPOSINGFACTORS Age, female, obesity, joint trauma. Female, HLA-DR4, smoking, silica exposure.
• 10 ® rheumatoid factor (lgM antibody that
• 11 targets lgG Fe region; in 80%), anti-cyclic
• 12 citrullinated peptide antibody (more specific).
• 13 PRESENTATION Pain in weight-bearing joints after usc (cg, Pain, swelling, and morning stiffness lasting
• 14
at the end of the day), improving with rest. > I hour, improving with use. Symmetric
Asymmetric joint involvement. Knee carti lage joint involvement. Systemic symptoms
• 15
loss begins medially ("bowlegged"). o (rever, fatigue, weight loss). Extraarticular
• 16 systemic symptoms. manifestations common."
• 17 Erosions, juxta-articular osteopenia, soft I issue
JOINTFINDINGS Osteophytes (bone spurs), joint space narrowing,
• 18 subchondral sclerosis and cysts. Synovial swelling, subchondral cysts, joint space
• 19 Auid non-inflammatory (\.V BC < 2000/mm 3). narrowing. Deformiti es: cervical subluxation,
• 20 llwolves DIP (Heberden nodes : ) and PIP ulnar fin ger deviation, swan neck rn.
(Bouchard nodes B ), and 1st C MC; not .MCP. boutonniere (). ln\'Ok es i\IC P, PIP, wrist; not
• 21
• J) J P n r Id r.\Jr. <::vnm•i" J An irl i n A<~ rnrn<~ lnn•

a
Lock
s
Su spend
8
End Bl ock
Item:9of33 ~. , . M k <:] t> al ~· ~
QIO: 5187 .l. ar Previous Next lab 'lifllues Notes Calculator

1 •
A mother brings her 7- year- old son to the ped iatrician with concerns about his increasing weakness and
2
clumsiness. Though as a baby he met all of his early motor milestones, the mother recalls that he was not
3 able to walk on his own until 18 months of age. Currently, he is having increasing difficulty keeping up with
4 his peers during playtime. Examination of the lower extremities revea ls normal patellar and Ach illes reflexes,
normal sensation, and 3/5 muscle strength . His calf muscles have increased in diameter relative to the rest of his
5
body . Microscopic examination of a biopsy of th is muscle t issue reveals wide variation in the diameter of individua l
6 muscle fibers, and degenerative necrosis of some muscle fibers with interspersed regenerating fibers . Though she
7 is unaffected, the mother states that she had a "sickly" brother who died at a young age after being confined to a
8
wheelcha ir for some time . Her husband does not have any history of weakness in his family. She is 30 weeks'
pregnant with a baby girl and has concerns that this ch ild may be sick as well.
•9
• 10
What is the probability that the patient's sister wi ll develop the same condition that has caused his weakness'
· 11
:
• 12
A. Neg ligible
• 13
• 14 B. 25%
• 15 c. 50%
• 16
0 . 100%
• 17
E. Not enough data to determine this probability
• 18
• 19
• 20
• 21

6
lock
s
Suspend
0
End Block
Item: 9 of 33 ~ 1 • M k -<:J 1>- Jil ~· !:';-~
QIO: 5187 ..L ar Pre v ious Next Lab fli!ltues Notes Calcula t o r
& &
1
2
The correct answer is A. 65°/o chose this.
The most likely cause of the patient's increasing weakness is the X-linked
3
inherited disorder Duchenne muscula r dystrophy (DMD). The patient presents
4 with a characteristic history of progressive lower extremity weakness, initially
5 evident around the age of 5. Physical examination findings are characteristic,
6 as is the pseudohypertrophy of the calf muscles (resembling that shown in
the image) from increased fatty connective tissue. DMD is inherited in an X-
7
linked recessive pattern. I n this case, the history of an early death in the
8 maternal uncle suggests that the mother is a carrier of the mutated
9 dystrophin gene. The unborn daughter has a 50% chance of being a carrier of
• 10
th is gene, but a negligible chance of expressing the phenotype, as she will
inherit a normal X chromosome from her father. Although t here is the
• 11
possibility that the daughter could exhibit the phenotype due to mosaicism Image copyright © 2014
• 12 from X- inactivation or due to Turner syndrome, this possibility is incredib ly Senanayake eta/.; licensee
• 13 rare . BioMed Central Ltd.
Oystrophin Turner syndrome Ouchenne muscular dystrophy X chromosome X-inactivation Gene
• 14 Mosaic (genetics) Phenotype Muscular dystrophy

• 15 X-linked recessive inheritance Recessive Dominance (genetics) Genetic disorder Sex linkage Connective tissue Chromosome

• 16
B is not correct. 12°/o chose this .
• 17
Some rarer fo rms of limb gi rd le m uscu lar dyst rophy are in herit ed in an autosomal-recessive pattern. However,
• 18 the lack of any hist ory in the father's fam ily makes th is a much less likely diagnosis .
• 19 Umb- girdle muscular dystrophy Muscular dystrophy

• 20 C is not correct. 14°/o chose this.


• 21 The daughter has a 50% chance of being an unaffected carrier of the mutated dystrophin gene .

a
Lock
s
Suspend
8
End Bl ock
Item:9of33 ~. , . M k <:] t> al ~· ~
QIO: 5187 .l. ar Previous Next lab 'lifllues Notes Calculator

1 •
B is not correct. 12% chose this.
2
Some ra rer forms of limb girdle muscular dystrophy are inherited in an autosomal-recessive pattern . However,
3 the lack of any history in the father's fami ly makes this a much less likely diagnosis.
4 limb-girdle muscular dystrophy Muscular dystrophy

5 C is not correct. 14% chose this.


6 The daughter has a 50% chance of being an unaffected carrier of the mutated dystrophin gene.
7 Dystrophin Gene

8 D is not correct. 5°/o chose this.


9 Some forms of muscular dystrophy have an autosoma l-dominant or a mitochond r ial pattern of inher itance. In
0 10 this case, because the mother is unaffected, these diagnoses are much less like ly than Duchenne muscu lar
dystrophy.
· 11
Duchenne muscular dystrophy Muscular dystrophy Genetic disorder Dominance (genetics) Mitochondrion Mitochondrial DNA
0
12
0
13
E is not correct. 4°/o chose this.
0 14
From the cl inical characteristics of the patient, the diagnosis of Duchenne muscular dystrophy, with a well-
character ized pattern of inheritance, may be presumed.
0 15 Duchenne muscular dystrophy Muscular dystrophy
0
16
0 17
Bottom Line:
0
18
0 19
Duchenne muscu lar dystrophy is inher ited in an X-linked recessive pattern .
Duchenne muscular dystrophy Muscular dystrophy Recessive Dominance (genetics) X-linked recessive inheritance Sex linkage
0 20
0
21

6
lock
s
Suspend
0
End Block
Item: 9 of 33 ~ 1 • M k -<:J 1>- Jil ~· !:';-~
QIO: 5187 ..L ar Pre v ious Next Lab fli!ltues Notes Calcula t o r
& &
1

2 FA17 p 57.1
Muscular dystrophies
3
4 Duchenne X-linked disorder typicall) due to frameshift D uchenne = d eleted dystrophin.
or nonsense mutations - truncated or Dystrophin gene (DMD) is the largest
5
absent dystrophin protein - progrcssi,·c protein-coding human gene - t chance of
6
myofiber damage. Weakness begins in pcl"ic spontaneous mutation. Dystrophin helps
7 girdle muscles and progresses superiorly. ancho r muscle fibers, primarily in skeletal and
8 Pseudohypertrophy of calf muscles due to cardiac muscle. It connects the intracellular
9 fibrofatty replacement of muscle . \ addling crtoskeleton (actin) to the transmembrane
gait. Onset before 5 yea rs of age. Dilated proteins a- and ~-dystroglycan, which are
• 10 D
cardiomyopathy is common cause of death . connected to the extracellular matrix (EC~ I ) .
• 11
l.oss of dystrophin results in myonecrosis.
• 12 f CK and aldolase are seen; genetic testing
• 13 con ti rms diagnosis .
• 14 Becker X-linked disorder ty pically due lo non- Deletions can cause both Duchenne and
• 15 frameshift dele tions in dystrophin gene Becker muscular dystrop hies. Y, of cases have
• 16 (partially functional instead of tnmc:1tecl). Less large deletions spann ing one or more exons.
severe than Duchcnnc. Onset in adolescence
• 17
or early adult hood .
. 18
Myotonic type 1 Autosomal dominant. CTG trinucleotide repeal C ataracts, Toupee (early balding in men),
• 19
expansion in the DMPK gene - abnormal G onadal atrophy.
• 20 expression of myotonin protein kinase
• 21 -+ myotonia, muscle wasting, cataracts,

a
Lock
s
Suspend
8
End Bl ock
Item: 9 of 33 ~ 1 • M k -<:J 1>- Jil ~· !:';-~
QIO: 5187 ..L ar Pre v ious Next Lab fli!ltues Notes Calcula t o r
& &
1
Myotonic type 1 Autosomal dominant. CTG trinucleotide repeal Cataracts, Toupee (early balding in men),
2 expansion in the DMPK gene ... abnormal Gonadal atrophy.
3 expression of myotonin protein kinase
4 ... myotonia, muscle wasting, cataracts,
5 testicular atrophy, frontal balding, arrhythmia.
6 Gower sign-patient uses upper e!\trcmities to
help stand up.
7
Classically seen in Duchenne muscular
8
dystrophy, but also seen in other muscular
9 dystrophies and inflammatory m)Opathies (eg,
• 10 polymyositis). loni<M-._, )
• 11

• 12
• 13
• 14
• 15
FA17 p 56.4
• 16
X-linked recessive O rnith ine transcarbamylase der,ciency, Fabry Oblivious Female \Viii O ften G ive ller Boys
• 17
disorders disease, \\'iskott-Aldrieh S) ndromc, Ocular ller x-Linked Disorders
. 18 albinism, G6PD deficiency, I hmter syndrome,
• 19 Bruton agammaglobulinemia, Hemophilia
• 20 A and B, Lesch-1 yhan syndrome, D uchcnnc
(and Becker) muscular dystrophy.
• 21
• I vl"\ni?:':llt-inn _f~m"" J "' ,..'l rr;l"\rco ' •""r;,..l,lu ,..(f,..,..t~,.l

a
Lock
s
Suspend
8
End Bl ock
Item: 9 of 33 ~ 1 • M k -<:J 1>- Jil ~· !:';-~
QIO: 5187 ..L ar Pre v ious Next Lab fli!ltues Notes Calcula t o r
I • I t I t I :
1

2
Gower sign-patient uses upper extremities lo
help stand up.
3
Classically seen in Duchen ne muscular
4 dystrophy, but also seen in other muscular
5 dystrophies and innammator) 111)0pathies (eg,
6 polymyositis). ...._-......... )
7
8
9
• 10
• 11 FA17 p 56.4
• 12
X-linked recessive Ornithine transcarbamylase de~cienC)', Fabry Oblivious Female \\"ill O ften G ive ller Boys
• 13 disorders disease, \Viskott-Aidrich syndrome, Ocular ller x-Linked Disorders
• 14 albinism, G6PD deficiency, ll unter syndrome,
• 15 Bruton agammaglobul inem ia, llemoph ilia
• 16 A and B, Lesch- yhan syndrome, Duchcnnc
(and Becker} muscular dystrophy.
• 17
Lyonization -fcmale carriers \'ariably <~ffcctcd
• 18 depending on the pattern of inacl ivai ion of the
• 19 X chromosome carrying the mutant vs normal
• 20 gene.
• 21

a
Lock
s
Suspend
8
End Bl ock
Item: 10 of 33 ~. I • M k <:] t> al ~· ~
QIO: 5003 .l. ar Previous Next lab 'lifllues Notes Calculator

1 •
A 68-year-old man presents to his physician's office with diffuse pelvic pain . He reports that it gradua lly
2
started about 2 weeks ago, and the aching qual ity of the pain has been worsening. It does not seem to be
3 related to activity but is worse at night. He denies any history of arthritis and has tried taking ibuprofen
4 and acetaminophen to no ava il. On review of systems he admits to an unintentional weight loss of 10 pounds in
the past 2 months and den ies any recent trauma, shooting rad iculopathy, or alarm symptoms such as sadd le
5
numbness or bowel and bladder incontinence. On physical examination, the patient is negative for point
6 tenderness over the spine, but is tender over several pelvic areas. You recommend imaging to further evaluate the
7 patient's source of pain; an x-ray of the patient's pelvis is shown in the image.
8
9
0 10
o ll
0
12
0
13
0 14
0 15
0
16
0 17
0
18
0 19
0 20
0
21

6
lock
s
Suspend
0
End Block
Item: 10 of 33 ~ 1 • M k -<:J 1>- Jil ~· !:';-~
QIO: 5 003 ..L ar Pre v ious Next Lab fli!ltues Not es Calcula t o r
. .. . .. . . .. . .. . .. . . . -. . .. - . . .. .. . . .. . .. .. ..
1
numbness or bowel and bladder incontinence. On physical examination, the patient is negative for point
2 tenderness over the spine, but is tender over several pelvic areas. You recommend imaging to further evaluate the
3 patient's source of pain; an x-ray of the patient's pelvis is shown in the image.
4
5
6
7
8
9
• 10
• 11
• 12
• 13
• 14
• 15
• 16
• 17
• 18
• 19
• 20
• 21

a
Lock
s
Suspend
8
End Bl ock
• 11
• 12
Which of the following is t he next best step to find the sou rce of this patient's condition?
• 13
:
• 14
A. Digita l recta l exa mi nat ion
• 15
• 16
B. Pa lpation of the abdomen
• 17 C. Pa lpation of the costovertebral ang le
• 18 D. Palpation of the neck
• 19
E. Thoroug h skin exa mination
• 20
• 21

a
Lock
s
Suspend
8
End Block
Item: 10 of 33 ~. I • M k <:] t> al ~· ~
QIO: 5003 .l. ar Previous Next lab 'lifllues Notes Calculator

1 •
The correct answer is A. 56°/o chose this.
2
The image demonstrates osteoblastic lesions of the pelvis secondary to
3 metastatic cancer (indicated here by the ar rows). Osteoblastic lesions are
4 generally wh ite, hyperdense regions on x-ray, representing the excess
5 deposition of new bone that's taking place in the skeleton ( "osteoBiasts
deposit Bone "). In contrast, osteolytic lesions, as seen in diseases like
6
multiple myeloma, are hyper lucent regions of bone loss secondary to bone
7 resportion. Prostate cancer, the most common cause of cancer in men, is
8 notorious for producing osteoblastic lesions upon metastasis. Prostate
9
cancer can often be detected with a dig ital rectal examination . Digital rectal
examination can also be useful in detecting a smal l number of colorectal
10 cancers, though most bony lesions of metastatic colorectal cancer are
· 11 osteolytic.
Multiple myeloma Rectal examination Colorectal cancer Prostate cancer Metastasis Pelvis Bone Osteolysis
• 12 Image courtesy of William Scott,
• 13 Osteoblast Osteoporosis Prostate Cancer X-ray Rectum MD
• 14
B is not correct. 12% chose this .
• 15
Palpation of the abdomen may assist in the detection of leukemia or lymphoma, as splenomegaly is a common
• 16 presenting symptom . These cancers occur much less frequently than prostate cancer, and more often produce
• 17 osteolytic lesions .
Splenomegaly Prostate cancer leukemia lymphoma Osteolysis Palpation Abdomen Cancer Prostate Symptom
• 18
• 19 C is not correct. 11% chose this .
• 20 Signs of renal cell cancer include flank pain (noted upon palpation of the costovertebral angle), the presence of
• 21 a flank mass, hematu r ia, and/or weight loss. Renal metastases are more common ly osteolytic .

6
lock
s
Suspend
0
End Block
Item: 10 of 33 ~. I • M k <:] t> al ~· ~
QIO: 5003 .l. ar Previous Next lab 'lifllues Notes Calculator

1 •
B is not correct. 12% chose this.
2
Pal pation of the abdomen may assist in the det ection of leukemi a or lymphoma, as splenomega ly is a common
3
presenting symptom . These cancers occur much less f requent ly t han prostate cancer, and mo re often produce
4 osteolytic lesions.
5 Splenomegaly Prostate cancer leukemia lymphoma Osteolysis Palpation Abdomen Cancer Prostate Symptom

6 C is not correct. 11% chose this.


7 Signs of rena l ce ll cancer include flank pain (not ed upon pa lpation of t he costovertebral angle), the presence of
8 a fl ank mass, hemat ur ia, and/or weight loss. Renal metastases are more common ly osteolytic.
Hematuria Renal cell carcinoma Kidney cancer Metastasis Abdominal pain Cancer Kidney Palpation Weight loss
9
10 D is not correct. 10% c hose this.
· 11 Pa lpation of the neck may be helpful in detecting thyroid cancer. Thyroid metastases are more often osteolyt ic.
Thyroid cancer Metastasis Osteolysis Thyroid Palpation Cancer
• 12
• 13 E is not correct. 11% c hose this .
• 14 A thorough skin examination wou ld be vita l in det ect ing a melanoma . Howeve r, melanoma is much less
• 15
common than prostate cance r, and most often produces osteolytic lesions when it met ast asizes t o bone .
Melanoma Prostate cancer Metastasis Osteolysis Bone Cancer Prostate
• 16
• 17

• 18 Bottom Line:
• 19 Prost ate cancer common ly produces osteoblast ic lesions upon metastasis .
Prostate cancer Metastasis Prostate Cancer Osteoblast
• 20
• 21

6
lock
s
Suspend
0
End Block
Item: 10 of 33 ~ 1 • M k -<:J 1>- Jil ~· !:';-~
QIO: 5 003 ..L ar Pre v ious Next Lab fli!ltues Not es Calcula t o r
& &
1
2 FA17 p 221.2

3 Common m etastases Most sarcomas spread hcmatogcnously; most carcinomas spread ,·ia ly1nphatics. l lowevcr, Four
4 C arcinomas Route llematogenously: Foil icular thyroid carcinoma, C horiocarcinoma, Renal cell
carcinoma, and Hepatocellular carcinoma.
5
SITE OF METASTASIS !• TUMOR NOTES
6
Brain Lung> breast> melanoma, colon, kidney. 50% of brain tumors are from metastases
7
Commonly seen as multiple well-circumscribed
8 tumors at gray/" hite matter junction.
9 liver Colon >>stomach > pancreas. Li,·cr Bm and lung are the most common sites
10 of metastasis after the regional lymph nodes.
• 11 Bone Prostate, breast > lung, thyroid, kidney. Bone metastasis D >> I o bone tumors (eg,
• 12 multiple myeloma, lytic). Common mcts to
• 13 bone: breast (m ixed), lung (lytic), th yroid
(lytic), kid ney (lytic), prostate (blastic) .
• 14
Predilection for axial skeleton I'!J.
• 15
• 16
• 17
• 18
• 19
• 20
• 21

a
Lock
s
Suspend
8
End Bl ock
Item: 10 of 33 ~ 1 • Ma rk -<:J I> ~ £!1}>'
• !!":-~
QIO: 5 003 ..L Pre v ious Next Lab lues Not es Calcula t o r

1
Predilection for axial skeleton [!:).
2

3
4
5
6
7
8
9
10
• 11

• 12
• 13
• 14
• 15
• 16
• 17
. 18
• 19
• 20
• 21
• I=A17 n R1Q A

a
Lock
s
Suspend
8
End Bl ock
2
3
4
5
6
7
8
9
10
• 11
• 12
• 13
• 14
• 15
FA17 p 61 9.4
• 16
Prostatic Common in men> 50 years old. Arises most oflcn from posterior lobe {peripheral zone) of prostate
• 17
adenocarcinoma gland and is most frequently diagnosed by t PSA and subsequent needle core biopsies. Prostatic
• 18
acid phosphatase (PAP) and PSA are useful tumor markers (t total PSA, with L fraction of free
• 19 PSA). Osteoblastic metastases in bone may de,·elop in late stages, as indicated by lower back pain
• 20 and t serum ALP and PSA .
• 21

a
Lock
s
Suspend
8
End Block
Item: 11 of 33 ~ 1 • M k -<:J 1>- Jil ~· !:';-~
QIO: 4575 ..L ar Pre v ious Next Lab fli!ltues Not es Calcula t o r

2
A 72-year-old woman presents to her prima ry ca re physician complaining of a new-onset headache different IAA]
from any she has experienced previously. She is also experiencing fevers and blu rry vision in her rig ht eye.
3 Laboratory studies show the following:
4
WBC count: 8000/ mm3
5
Hematocrit: 46%
6 Hemoglobin : 15 g/ dl
7 Platelet count: 310, 000/ mm 3
8
Na+ : 137 mEq/ L
K+ : 3 .5 mEq/ L
9
cl - : 102 mEq/ L
10 HC0 3- : 24 mEq/ L
• 11 Erythrocyte sed imentation rate: 98 mm/ hr
C-reactive protein : 3.5 mg/ L
• 12
• 13 A biopsy specimen of her right temporal artery shows the presence of giant cel ls .
• 14
• 15 What disease is most common ly associated with this woman's cond ition?
• 16
:
• 17 A . Der matomyositis
. 18
B. Fibromya lgia
• 19
• 20
C. Kawasa ki disease
• 21 D . Polymya lgia rheumatica

a
Lock
s
Suspend
8
End Bl ock
Item: llof33 ~. , . M k <:] t> al ~· ~
QIO: 4575 .l. ar Previous Next lab 'lifllues Notes Calculator

1 •

2 The correct answer is D. 74°/o chose this.


3 This woman has temporal arteritis (also called giant cell arteritis), which is an
inflammatory vasculitis, most commonly of large and medium arteries of the
4
head. Her blu r ry vision may be the result of central retinal artery occlusion,
5 like that shown in the image, which illustrates the cherry red spot (ar row) and
6 retina l pal lor characteristic of this condition .
7

8 Polymya lgia r heumatica is a related inflammatory process that often coexists


9 with temporal arteritis. Both conditions can be treated by corticosteroids.
10 Polymyalgia rheumatica Giant-cell arteritis Central retinal artery occlusion Vasculitis Corticosteroid

11 Ocular ischemic syndrome Artery Central retinal artery Arteritis Giant cell Vascular occlusion Inflammation Image copyright © 2009 Raja et
• 12 a/; licensee BioMed Central Ltd.
• 13
• 14
A is not correct. 8°/o chose this .
Der matomyositis is an inflammatory disease of muscles and skin. It is commonly associated with cancer, but
• 15
not with vascu litis. Patients with der matomyositis often need to be screened regula r ly for ma lignancies.
• 16 However, in patients affected with dermatomyositis in the pediatr ic population, studies have shown that it is
• 17 associated with vasculitis because juveni le dermatomyositis is a capillary vascu lopathy .
Juvenile dermatomyositis Dermatomyositis Vasculitis Malignancy Capillary Cancer Pediatrics Inflammation
• 18
• 19 B is not correct. 8°/o chose this .
• 20 Fibromyalgia is an idiopathic and chronic condition of diffuse or specific muscle pain that usually manifests in
• 21
young to middle-aged females. Patients often complain of stiffness. Fatigue, cognitive disturbance, and
• c:: \1 n-. n t,., n-. c:: ,., f rl "' n r"' c:: c:: i ,., n ,., r ::> n v i "'t\1 ::> r"' ::> I c:: ,., ,., ft"' n c:: "' "' n i n t h i c:: c:: \1 n rl r,., n-. "' h 1 1t t"' n-. n ,., r ::> I ::> rt"' r i t i c:: i c:: n ,., t ::> c:: c:: ,., r i ::> t"' rl

6
lock Suspend
s End Block
0
Item: llof33 ~. , . M k <:] t> al ~· ~
QIO: 4575 .l. ar Previous Next lab 'lifllues Notes Calculator

1 • Juvenile dermatomyositis Dermatomyositis Vasculitis Malignancy Capillary Cancer Pediatrics Inflammation

2 B is not correct. 8°/o chose this.


3 Fibromyalgia is an idiopathic and chronic condition of diffuse or specific muscle pain that usually manifests in
4 young to middle-aged females. Patients often complain of stiffness. Fatigue, cognitive disturbance, and
symptoms of depression or anxiety are also often seen in th is syndrome, but temporal arteritis is not associated
5
with fibromya lgia.
6 Fibromyalgia Giant-cell arteritis Myalgia Anxiety Idiopathy Fatigue (medical) Major depressive disorder Arteritis Depression (mood) Chronic condition Muscle
7
C is not correct. 6°/o chose this.
8
Kawasa ki disease is a ped iatr ic vascu litis that affects many organs. It can cause vasculitis in the coronary
9 arter ies, resulting in a coronary aneurysm . It does not affect older patients and is not associated with temporal
10 arter itis.
Kawasaki disease Giant-cell arteritis Coronary artery aneurysm Vasculitis Coronary circulation Aneurysm Arteritis Artery
11
• 12 E is not correct. 4°/o chose this .
• 13 Ulcerative colitis is an inflammatory bowel disease t hat leads to malabsorption, bloody dia r rhea, and is
• 14 associated with primary sclerosing cholangitis, but is not associated with temporal arteritis .
Primary sclerosing cholangitis Ulcerative colitis Inflammatory bowel disease Giant-cell arteritis Diarrhea Malabsorption Colitis Ascending cholangitis
• 15
Gastrointestinal tract Inflammation
• 16
• 17

• 18 Bottom Line:
• 19 Polym ya lgia r heumatica often is associated with giant cell arteritis . Both are t reated with prednisone .
• 20 Polymyalgia rheumatica Prednisone Giant-cell arteritis Giant cell Arteritis

• 21

6
lock
s
Suspend
0
End Block
Item: 11 of 33 ~ 1 • M k -<:J 1>- Jil ~· !:';-~
QIO: 4575 ..L ar Pre v ious Next Lab fli!ltues Not es Calcula t o r

1 y y ) g g
2 "Pulseless disease" (weak upper extrem ity aortic arch and proximal great vessels rn.
pulses), fever, night S\\ Cats, arthritis, myalgias, t ESR.
3
skin nodules, ocular disturbances. 'li·eat with corticosteroids.
4
Medium vessel vasculitis
5
Polyarteritis nodosa Usually middle-aged men. Typically im·oh-es renal and visceral vessels, not
6
Hepatitis B seropositivity in 30% of patients. pulmonary arteries.
7 Fe,·er, weight loss, malaise, headache. Transmural inAammation of the arterial wall
8 Cl: abdominal pain, melena. with fi brinoid necrosis.
9 Hypertension, neurologic dysfunction, Different stages of inAammation ma} coc\ist in
10 cutaneous eruptions, renal damage. different vessels.
Innumerable renal microaneurysms and spasms
11
on arteriogram.
• 12 Treat with corticosteroids, cyclophosphamide .
• 13
Kawasaki disease Asian children < 4 years old. CRASH and burn.
• 14 (mucocutaneous Conjunctival injection, Rash (polymorphous 1ay develop coronary artery aneurysms
• 15 lymph node -+ desquamating), Adenopathy (cervical), thrombosis or rupture can cause death.
• 16 syndrome) Strawberry tongue (oral mucosit is) fi], I land- Treat with IV immunoglobuli n and aspirin.
foot changes (edema, erythema), fc,cr.
• 17
• 18
Buerger disease I Ieavy smokers, malcs < 40 years old. Segmental thrombosing ,·asculitis.
(thromboangiitis Intermittent claudication may lead to Treat with smoking cessation.
• 19
obliterans) gangrene D. autoamputation of digits,
• 20 superficial nodular phlebitis.
• 21 Raynaud phenomenon is oft en present.

a
Lock
s
Suspend
8
End Bl ock
Item: 11 of 33 ~ 1 • M k -<:J 1>- Jil ~· !:';-~
QIO: 4575 ..L ar Pre v ious Next Lab fli!ltues Not es Calcula t o r
& &
1
Small -vessel vasculitis
2
Granulomatosis Upper respiratory tract: perforation of nasal Triad:
3
with polyangiitis septum, chronic sinusitis, otitis media, Focal necrotizing ,·asculitis
4 (Wegener) mastoiditis. Necrotizing granulomas in the lung and
5 Lower respiratory tract: hemoptysis, cough, upper airway
6 dyspnea. 'ecrotizing glomerulonephritis
Renal: hematuria, red cell casts. PR3-ANCA/c-A 'CA [!) (anti-proteinase 3).
7
C R: large nodular densities.
8
Treat with cyclophosphamide, corticosteroids.
9
Microscopic Necrotizing ,·asculitis commonly involving 'o granulomas.
10 polyangiitis lung, kidneys, and skin with pauci-immune \I PO- j CA/p-A1 CA CJ (anti-
11 glomerulonephritis and palpable purpura. •nrclopcroxidase).
• 12 Presentation similar to granulomatosis with Treat with cyclophosphamide, corticosteroids.
• 13 polyangiitis but without nasopharyngeal
involvement.
• 14
• 15 Eosinophilic Asthma, sinusitis, skin nodules or purpura, Cranulornatous, necrotizing vasculitis with
• 16 granulomatosis with peripheral neuropathy (eg, wrist/foot drop). eosinophilia 0 .
polyangiitis (Churg- Can also involve heart, Gl, kidneys (pauci- M PO-A CA/p-A TCA, t TgE level.
• 17
Strauss) immune glomerulonephritis).
. 18
Henoch-Schonlein Most common childhood systemic vascu litis. Vasculitis 2° to lg immune complex
• 19
purpura Often follows URI. deposition.
• 20 Classic triad: Associated with lgA nephropathy (Berger
• 21 • Skin: palpable purpura on buttocks/legs disease).

a
Lock
s
Suspend
8
End Bl ock
Item: 11 of 33 ~ 1 • M k -<:J 1>- Jil ~· !:';-~
..L ar Pre v ious Lab fli!ltues Not es Calcula t o r
QIO: 4575

1
&
. . . . .. ..
Next
.. ~ . .. . .. . - .. .. .. . &

polyangiitis (Churg- Can also involve heart, Gl, kidneys (pauci- M PO-AI CA/p-A TCA, t lgE level.
2 Strauss) immune glomerulonephritis).
3 Henoch-Schonlein 1ost common childhood systemic vasculitis. Vasculitis 2° to lgA immune complex
4 purpura Often follows URI. deposition.
5 Classic triad: Associated with lgA nephropathy (Berger
6 • Skin: palpable purpura on buttocks/legs disease).
Arthralgias
7
Gl: abdominal pain
8
. ~~~-~~ ~~~====~~~~
9
10
11
• 12
• 13
• 14
• 15
• 16
• 17
• 18
• 19
• 20 a
• 21

a
Lock
s
Suspend
8
End Bl ock
Item: 12 of 33 ~ 1 • M k -<:J 1>- Jil ~· !:';-~
QIO: 3187 ..L ar Pre v ious Next Labfli!llues Not es Calcula t o r

IAA]
A A

9
A 42-year-old woman has had increasing pain and swelling of the joints of her hands and feet for several
10 months . It is becoming very difficult for her to perform common household tasks. A microscopic image of the
11 synovium of a proximal interphalangeal joint in her hand is shown .
---~~...,....
• 12
• 13
• 14
• 15
• 16
• 17
. 18
• 19
• 20
• 21
• 22
• 23
• 24
• 25
• 26
Wh ich of the following laboratory serolog ic find ings would most likely be positive in th is patient?
• 27
:
• 28
A. Anticentromere antibody
• 29 •
a
Lock
s
Suspend
8
End Bl ock
9
10
11
• 12
• 13
• 14
• 15
• 16
• 17
• 18
• 19
• 20 Which of the following laboratory serologic findings wou ld most likely be posit ive in this patient?
• 21
:
• 22 A. Anticentromere antibody
• 23
B. Antinuclear antibody
• 24
• 25 C. Borrelia burgdorferi antibody
• 26 D. HLA-827
• 27
E. lgM ant i-lgG
• 28
• 29 •
a
Lock
s
Suspend
8
End Block
Item: 12 of 33 ~. I • M k <:] t> al ~· ~
QIO: 3187 .l. ar Previous Next lab 'lifllues Notes Calculator


9
10 The correct answer is E. 60°/o chose this.
11 Rheumatoid factor would most likely be posit ive in this woman, who is suffer ing from rheumatoid arth r itis (RA) .
Eighty percent of patients with RA have posit ive r heumatoid factor (anti -lgG antibody) . This autoimmune
12
cond ition causes a marked infl ux of inflammatory cells into the joint synovium, as seen here, resulting in
• 13 destructive change, pannus for mation, and eventua lly joint deformity . The disease is more common in women,
• 14 and classica lly symmetrical ly affects the proxima l inter pha langea l joints, as described here .
• 15 Under light microscopy, high- power v iew of the synovium in rheumatoid arthrit is shows hyperplasia of the
• 16 intima l cells (upper right corner of the image) as we ll as an intense lymphocytic infiltrate .
• 17 Rheumatoid arthritis Rheumatoid factor Synovial membrane Arthritis Interphalangeal articulations of hand Autoimmune disease Autoimmunity Antibody Pannus

• 18 Hyperplasia Anatomical terms of location

• 19
A is not correct. 9°/o chose this .
• 20 Anticentromere antibody is associated with the CREST variant of scleroderma (prog ressive systemic sclerosis) .
• 21 In this disease patients suffer from Calcinosis, Raynaud phenomenon, Esophagea l dysmotility, Sclerodactyly,
• 22 and Telang iect asia. Arthritis is not associated with this synd rome .
Systemic scleroderma Scleroderma Arthritis CREST syndrome Antibody
• 23
• 24 B is not correct. 15% chose this.
• 25 Antinuclear antibody is associated with systemic lupus erythematosus, an autoimmune disease with a wide
variety of symptoms, including fever, rash, joint pa in, and photosensitivity. The joint pain in lupus is typica lly
• 26
transient, asymmetrica l, and nondeforming .
• 27 Systemic lupus erythematosus Anti-nuclear antibody Autoimmune disease Photosensitivity Antibody Arthralgia Autoimmunity lupus erythematosus Rash Fever

• 28
C is not correct . 3°/o chose this .
• 29 •

6
lock
s
Suspend
0
End Block
Item: 12 of 33 ~. I • M k <:] t> al ~· ~
QIO: 3187

9
.l.
. ar
p
Previous
-
Next

I . y
Lab 'lifllues
p
and Telang iectasia. Arthritis is not associated with this synd rome .
Notes
. Calculator
p g .y y, . y y,

10 Systemic scleroderma Scleroderma Arthritis CREST syndrome Antibody

11
B is not correct. 15% chose this.
12
Antinuclear antibody is associated with systemic lupus erythematosus, an autoimmune disease with a wide
• 13 variety of symptoms, including fever, rash, joint pa in, and photosensitivity. The joint pain in lupus is typica lly
• 14 transient, asymmetrica l, and nondeforming .
Systemic lupus erythematosus Anti-nuclear antibody Autoimmune disease Photosensitivity Antibody Arthralgia Autoimmunity lupus erythematosus Rash Fever
• 15
• 16 C is not correct . 3°/o chose this .
• 17 Borrelia burgdorferi is the cause of Lyme disease. The third stage of Lyme disease can manifest as migratory
• 18
po lyarthrit is, but th is patient has none of the other signs and symptoms associated with th is disease .
Lyme disease Borrelia burgdorferi Borrelia Bird migration
• 19
• 20 D is not co rrect. 13% chose this .
• 21
HLA-B27 is strong ly associated with arth r itides that are negative for rheumatoid factor, the most wel l- known of
wh ich are the seronegative spondyloarthropathies . You can remember these with the mnemonic, PAIR:
• 22 Psoriatic arthritis, A nkylosing spondylitis, I nflammatory bowel disease, and Reactive arth ritis.
• 23 Spondyloarthropathy Reactive arthritis Rheumatoid factor HLA-B27 Arthritis Seronegative arthritis Serostatus Spondylitis Mnemonic

• 24
• 25
Bottom Line :
• 26
Most cases of RA invo lve accumu lation of l gM anti -l gG, wh ich prompts an influx of inflammatory ce lls into the
• 27
synovium .
• 28 Synovial membrane

• 29

6
lock
s
Suspend
0
End Block
Item: 12 of 33 ~ 1 • M k -<:J 1>- Jil ~· !:';-~
QIO: 3187 ..L ar Pre v ious Next Labfli!llues Not es Calcula t o r
A A

9
FA17p101 .1
10
Immunoglobulin All isotypes can exist as monomers. Mature, naive B cells prior to activation express lg\11 and lgD
11 isotypes on their surfaces. They may differentiate in germinal centers of lymph nodes by isOt) peS\\ itching
12 (gene rearrangement; mediated b) cytokines and CD40L) into plasma cells that secrete lg , lgE,
• 13 or lgC.
• 14 lgG i\ lain antibody in 2° (delayed) respon e to an antigen. ~ lost abundant isotype in serum. Fixes
complement, crosses the placenta (pro,·ides infants with passive immunity), opsonizes bacteria,

v
• 15
• 16
neutralizes bacterial toxins and ' 'iruses.

• 17
. 18
lgA Prc,·cnts attachment of bacteria <lnd viruses to mucous membranes; does not fi x complement.
Ylonomer (in circulation) or dimer (with J chain when secreted). Crosses epithelial cells by
• 19
transcytosis. Produced in C J tract (cg, by Peycr patches) and protects against gut infections (eg,
• 20 Giardia). .\!lost produced antibody overall, but h<1s lower serum concentrations. Released into
• 21 secretions (tears, saliva, mucus) and breast mi lk. Picks up secretory component from epithelial cells,
• 22 wh ich protects the Fe portion from luminal proteases.
• 23 lgM Produced in the 1° (immediate) response to <Ill ant igen. Fixes complement but does not cross the
• 24 placenta. Antigen receptor on the surface of B cells. Monomer on B cell, pentamer with J chain
when secreted. Pentamer cm1blcs avid bindiug to antigen while humoral response evolves.
• 25
• 26
• 27
• 28 IgO Unclear function. Found on smfacc of many B cells and in serum.
• 29 •
a
Lock
s
Suspend
8
End Bl ock
Item: 12 of 33 ~ 1 • M k -<:J 1>- Jil ~· !:';-~
QIO: 3187 ..L ar Pre v ious Next Labfli!llues Not es Calcula t o r
A A

9 IgO Unclear function. Found on surface of many B cells and in serum.

v
10
11
12
• 13 lgE Binds mast cells and basophils; cross-links when exposed to allergen, mediating immediate (trpe I)
• 14 hrpersensiti\'ity through release of inAammatory mediators such as histamine. Contributes to
• 15
inununity to worms by activating cosinophils. L O\\CSl concentration in serum.

• 16
• 17
FA17 p439.1
. 18
Osteoarthritis and rheumatoid arthritis
• 19
Osteoarthrit is Rheumatoid arthritis
• 20
PATHOGENESIS l\ lechanical-wear and tear destroys articular Autoimmune-inAammation induces formation
• 21
cartilage (degenerati\'e joint disorder) of pannus (proliferative granulation tissue rJ),
• 22
..... inAammation with inadequate repair. which erodes articular cartilage and bone .
• 23 Chondrocytes mediate degradation and
• 24 inadequate repair.
• 25 PREDISPOSING FACTORS Age, female, obesity, joint trauma . Female, I-ILA-DR4, smoking, silica exposure.
• 26 $ rheumatoid factor (lgM antibody that
• 27
targets lgG ~c region; in 80%), anti-cyclic
citrullinated peptide antibody (more specific).
• 28
Pain, swelling, and morning stiffness lasting
• 29 •
PRESENTATION
. ..
Pain in weight-bearing joints after use (eg,
• r.• 1 ' •
... o I ... ..

Lock
a s
Suspend
8
End Bl ock
Item: 12 of 33 ~ 1 • M k -<:J 1>- Jil ~· !:';-~
QIO: 3187 ..L ar Pre v ious Next Labfli!llues Not es Calcula t o r
A A

9 PREDISPOSING FACTORS Age, female, obesity, joint t·rauma. Female, I-ILA-DR4, smoking, silica exposure.
10 EE> rheumatoid factor (lgM antibody that
11 targets lgG Fe region; in 80%), anti-cyclic
citrullinated peptide antibody (more specific).
12
• 13
PRESENTATION Pain in weight-bearing joints after u~e (eg, Pain, swelling, and morning stiffn ess lasting
at the end of the day), improving with rest. > I hour, impro,·ing with use. Symmetric
• 14
Asymmetric joint im·ohcmcnt. Knee cartilage joint involvement. Systemic symptoms
• 15 loss begins medially (" bowlegged"). i o (fever, fatigue, weight loss). Extraarticular
• 16 systemic symptoms. manifestations common."'
• 17 JOINT FINDINGS Osteophytes (bone spurs), joint space narrO\\ ing, Erosions. juxta-articular osteopenia, soft tissue
. 18 subchondral sclerosis and cysts. Synovial swelling, subchondral cysts, joint space
• 19 Auid non-inAammatory (WBC < 2000/mml). mtrro\\ ing. Deformities: cen·ical subluxation,
Involves DIP (llcberdcn nodes : ) and PIP ulnar finger deviation, swan neck [!),
• 20
(Bouchard nodes ), and 1st CMC; not VICP. boutonniere D. lm·oh-es i\ ICP, PIP, wrist; not
• 21
DIP or 1st CMC. Synovial Auid inA;ummtory.
• 22
TREATMENT Acetaminophen, 1S IDs, intra-articu lar NSA IDs, glucocorticoids, disease-modifying
• 23 glucocorticoids. agents (methotrexate, sui fasa lazine,
• 24 hyd roxychloroquine, IeAunomide), biologic
• 25 agents (eg, T J F'-o: inh ibitors).
• 26 *Extraarticular manifestations include rheumatoid nodules (fibrinoid necrosis '' ith palisading histiocytes) in subcutaneous
• 27 tissue and lung (+ pneumoconiosis .... Caplan syndrome), interstitial lung disease, pleuritis, pericarditis, anemia of chronic
disease, neutropenia+ splenomegaly (Felty syndrome), AA amyloidosis, Sjogren syndrome, scleritis, carpal tunnel syndrome.
• 28
• 29 Normal Osteoarthritis Normal Rheumatoid

a
Lock
s
Suspend
8
End Bl ock
Item: 12 of 33 ~ 1 • M k -<:J 1>- Jil ~· !:';-~
QIO: 3187 ..L ar Pre v ious Next Labfli!llues Not es Calcula t o r
A A

9 Normal Osteoarthritis Normal Rheumatoid


•• arthritis
10 - - - - - ThiCkened '
capsule •''
~:~~:d

/
11 •'''
Shght synovial
/
12 Joint capsule ~ Joint capsule ~ •'' erosion
/ hypertrophy I
and synovial ".
- - - Osteophyte
and synOVIal ". ••
• 13 lining lining •'
SynoVIal _ / - - - Ulcerated Syn<Mal _ / '•'' - - - Increased
syn<Mallluld
• 14 '
~
cavrty/ cartilage cavrty /" ••'' - - - Pannus
• 15
Cartilage ~ ~ SclerOIJC bone Cartttage/ •••

IormatiOn
• 16 J0111tspace ••
narrow.ng ••

I
• 17 Subchondral •••
bone cyst
. 18
• 19
• 20
• 21
• 22
• 23
• 24
• 25
• 26
FA17 p442.1
I
• 27
Seronegative Arthritis without rheumatoid factor (no anli-IgC antibody). Strong association with HLA-8 27
• 28
spondyloarthritis (1\IHC class I serot) pc). Subl) pes (P \IR) share , ·ariable occurrence of inAammatory back
• 29 • oain (associated with morn in~ stiffness. imorO\·es "ith exercise). oerioheral arthritis. enthesitis

a
Lock Suspend
s 8
End Bl ock
Item: 12 of 33 ~ 1 • M k -<:J 1>- Jil ~· !:';-~
QIO: 3187 ..L ar Pre v ious Next Labfli!llues Not es Calcula t o r
A A

9 FA17 p 442.1
10 Seronegative Arthritis without rheumatoid factor (no anti-IgC antibody). Strong association with HLA-B27
11 spondyloarthritis ( ~II IC class I serotype). Subt) pes (P\I R) share ,·ariable occurrence of inflammatory back
12 pain (associated with morning stiffness, impro,·es with exercise), peripheral arthritis, enthesitis
• 13 (inAamed insertion sites of tendons, cg, Achilles), dactylitis rsausage fingers"), uveitis.
• 14 Psoriatic arthritis Associated with skin psoriasis and nail lesions. Seen in fe"er than 11> of patients" ith psoriasis.
symmetric and patchy im·okement
• 15
Dactylitis and "pencil-in-cup" deformity of
• 16
DIP on x-rav• g .
• 17
Ankylosing Srmmetric im·olvement of spine and sacroiliac Bamboo spine (,·ertebral fusion) . Can cause
. 18 spondylitis joints - ankylosis (joint fusion), uveitis, aortic restrictive lung disease due to limited chest
• 19 regurgitation . wa 11 expansion (costovertebral and costosternal
• 20 ankylosis).
• 21
lore common in males.
• 22 Inflammatory bowel Crohn disease and ulcerative colitis arc often
disease associated with spondyloa rthritis.
• 23
• 24
Reactive arthritis Formerly known as Reiter syndrome. "Can' t see, can' t pee, can't bend my knee."
Classic triad: Shigella, Yersinia, Chlamydia, Campylobacter,
• 25
Conjuncti\ iti~ Salmonella (ShY C hiCS).
• 26
• 27
• 28
• 29
• Urethritis
\ rthritis
I

a
Lock
s
Suspend
8
End Bl ock
Item: 12 of 33 ~ 1 • M k -<:J 1>- Jil ~· !:';-~
QIO: 3187 ..L ar Pre v ious Next Labfli!llues Not es Calcula t o r

9
A
Ankylosing Symmetric involvement of spine and sacroil iac Bamboo spine (,·ertebral fusion) ~. Can cause A

spondylitis joints - ankylosis (joint fusion), uveitis, aortic restrictive lung disease due to limited chest
10
regurgitation. wall expansion (cosiO\'ertebral and costosterna l
11 an kr losis).
12 ~l ore common in males.
• 13 Inflammatory bowel Crohn disease and ulcerative colitis are often
• 14 disease associated with spondyloarthritis.
• 15 Reactive arthritis F'ormerly known as Reiter syndrome. "(',an ' t ~ee, can ' t pee, can ' t bend Ill) k-nee...
• 16 Classic triad: Shigella, Yersinia, Chlamydia, Campylobacter,
• 17
Conjuncti\ iti ~ Salmo,ella (ShY C hiCS).
Urethritis
. 18
\rthritis
• 19
• 20
• 21
• 22
• 23
• 24
• 25
• 26

I
• 27
• 28
• 29 •
.
a
Lock
s
Suspend
8
End Bl ock
Item: 14 of 33 ~ 1 • M k -<:J 1>- Jil ~· !:';-~
QIO: 5 0 91 ..L ar Pre v ious Next Lab fli!ltues Not es Calcula t o r

IAA]
A A

9
A 28-year-old man presents to the emergency department with complaints of bilateral knee pa in (right worse
10 than left) . The pain is described as an aching stiffness that began spontaneously, w ithout any acute injury.
11 The patient is in mild distress, stating that he is "fed up" with doctors th is week, as he was recently
prescribed a course of antibiotics by his primary care physician for conjunctivitis, but the medication has not
12
seemed to make a difference. The patient's past medical history is unremarkable except for a recent history of
• 13 receiving "a shot" for a venereal infection 3 weeks prior to presentation .
• 14
• 15 What is the most likely cause of this patient's joint pain?
• 16
:
• 17 A. Ankylosing spondylitis
• 18
B. Drug reaction
• 19
• 20
C. Pseudogout
• 21 D. Reactive arthritis
• 22 E. Rheumatoid arthritis
• 23
F. Systemic lupus erythematosus
• 24
• 25 G. Viral arthritis
• 26
• 27
• 28
• 29 •
a
Lock
s
Suspend
8
End Bl ock
Item: 14 of 33 ~. I • M k <:] t> al ~· ~
QIO: 5091 .l. ar Previous Next lab 'lifllues Notes Calculator


9
The correct answer is D. 77°/o chose this.
10
Reactive arthritis is one of the seronegative spondyloarthropathies. Commonly associated with positive HLA-
11 827, typical patients are men between the ages of 20 and 40 . The classic triad of symptoms is urethritis,
12 conjunctivit is, and arthrit is, wh ich can be reca lled by the mnemonic "Can't see, can't pee, can't climb a
• 13
tree." Current evidence suggests that the syndrome occurs as an autoimmune reaction triggered by a recent
genitourinary or gastrointestinal infection with Chlamydia, Shigella, or Campy!obacter. The most commonly
14 affected areas are the weight-bearing joints of the lower extremities as wel l as the lower back. Distribution may
• 15 be asymmetric .
Spondyloarthropathy Reactive arthritis HLA-827 Urethritis Conjunctivitis Campylobacter Shigella Arthritis Autoimmune disease Autoimmunity Chlamydia infection
• 16
• 17 Serostatus Genitourinary system Seronegative arthritis Mnemonic Gastrointestinal tract

• 18 A is not correct. 3°/o chose this .


• 19 Ankylosing spondyl itis is a chronic inflammatory disease of the joints, most commonly affecting the sacroi liac
• 20 joints of adolescent ma les. Like reactive arthrit is, it is associated with the HLA-827 genotype, and uveitis may
be a compl ication. However, ankylosing spondylitis is a more chronic condition, and the recent genitourinary
• 21
infection in th is patient makes reactive arth r itis the most like ly diagnosis .
• 22 Ankylosing spondylitis Reactive arthritis Uveitis HLA-827 Arthritis Genotype Spondylitis Genitourinary system Sacroiliac joint Chronic condition

• 23
B is not correct. 7°/o chose this .
• 24
Some antibiotics cause problems with tendon and joint deve lopment, but wou ld not be the most likely cause of
• 25 this patient's symptoms.
• 26 Antibiotics Tendon

• 27
C is not correct. 3°/o chose this .
• 28 Pseudogout results from calcium pyrophosphate crystal deposition in the joint space . It usual ly occurs in
• 29 •
individuals over 50 years of age and most often affects the knees and wrists .

6
lock
s
Suspend
0
End Block
Item: 14 of 33 ~. I • M k <:] t> al ~· ~
QIO: 5091 .l. ar Previous Next lab 'lifllues Notes Calculator


9 C is not correct. 3°/o chose this.
10 Pseudogout results from calcium pyrophosphate crystal deposition in the joint space . It usual ly occurs in
11 individuals over 50 years of age and most often affects the knees and wrists .
Chondrocalcinosis Calcium pyrophosphate Pyrophosphate Crystal Calcium
12
• 13 E is not correct. 2°/o chose this .
14 Rheumatoid arth r itis is a chronic inflammatory disease that often manifests with joint pa in. The small joints of
• 15 the body, such as the metacarpophalangea l and interphalangeal joints, as well as the ankle and elbow are most
commonly affected .
• 16
Rheumatoid arthritis Interphalangeal articulations of hand Arthritis Arthralgia Metacarpophalangeal joint
• 17
F is not correct. 2°/o chose this .
• 18
Systemic lupus erythematosus (SLE) is a chronic autoimmune inflammatory disorder that may manifest as
• 19
arthritis . However, patients with SLE more common ly present with a rash, oropharyngeal ulceration, or
• 20 serositis .
• 21 Systemic lupus erythematosus Serositis Mouth ulcer Autoimmune disease Autoimmunity lupus erythematosus Arthritis Pharynx Ulcer Rash Peptic ulcer

• 22 G is not correct. 6°/o chose this .


• 23 Arthritic symptoms can occur in the setting of parvovirus B19, rubella, and hepatitis C infections. However, this
• 24 patient has a history of a bacterial gen itourinary infection and no clear indication of a history of vi ral infection .
Parvovirus 819 Hepatitis C Parvovirus Hepatitis Arthritis Rubella Viral disease Genitourinary system
• 25
• 26
• 27 Bottom Line:
• 28 Reactive arthritis occurs 2-3 weeks after a genitou r inary or gastrointestinal infection and man ifests with the
• 29 •
classic triad of conjunctivitis, arthritis, and ureth r itis. •

6
lock
s
Suspend
0
End Block
Item: 14 of 33 ~ 1 • M k -<:J 1>- Jil ~· !:';-~
QIO: 5 0 91 ..L ar Pre v ious Next Lab fli!ltues Not es Calcula t o r
A A

9
FA17 p 442.1
10
Seronegative Arthritis without rheumatoid factor (no anti-IgC antibody). Strong association with HLA-8 27
11
spondyloarthritis (1\IHC class I serot) pe). Subtypes (P \IR) share ,·ariable occurrence of inflammatory back
12 pain (associated with morning stiffness, impro,·es with exercise), peripheral arthritis, enthesitis
• 13 (inAamed insertion sites of tendons, eg, Achilles), dactylitis ("sausage fingers"), uveitis.
14 Psoriatic arthritis Associated with skin psoriasis and nail lesions. Seen in fewer than 11> of patients with psoriasis.
• 15 symmetric and patchy im·okement
• 16 Dactylitis and "pencil-in-cup" deformity of
DIP on x-rad].
• 17 '
Ankylosing Srmmctric involvement of spine and sacroiliac Bamboo spine (,·ertebral fusion) . Can cause
. 18
spondylitis joints - ankylosis (joint fusion), uveit is, aortic restrictive lung disease due to limited chest
• 19
regurgitation . wall expansion (costovertebral and costostcrnal
• 20 ankrlosis).
• 21 lore common in males.
• 22 Inflammatory bowel Crohn disease and ulcerative colitis arc often
• 23 disease associated with spondyloa rth ritis.
• 24 Reactive arthritis Formerly known as Reiter syndrome. "Can' t see, can' t pee, can' t bend my knee."
• 25 Classic triad: Shigella, Yersinia, Chlamydia, Campylobacter,
Conjuncti\ iti~ Salmollella (ShY C hiCS).
• 26
• Urcth ritis
• 27
\rthritis
• 28
• 29 •
a
Lock
s
Suspend
8
End Bl ock
Item: 14 of 33 ~ 1 • M k -<:J 1>- Jil ~· !:';-~
QIO: 5 0 91 ..L ar Pre v ious Next Lab fli!ltues Not es Calcula t o r

9
A
Ankylosing Symmetric involvement of spine and sacroiliac Bamboo spine (,·ertebral fusion) ~. Can cause A

spondylitis joints - ankylosis (joint fusion), uveitis, aortic restrictive lung disease due to limited chest
10
regurgitation. wall expansion (cosiO\'ertebral and costosternal
11 an krlosis).
12 ~lore common in males.
• 13 Inflammatory bowel Crohn disease and ulcerative colitis are often
14 disease associated with spondyloarthritis.
• 15 Reactive arthritis F'ormerly known as Reiter syndrome. "(',an ' t ~ee, can ' t pee, can ' t bend Ill) k-nee...
• 16 Classic triad: Shigella, Yersinia, Chlamydia, Campylobacter,
• 17
Conjuncti\ iti ~ Salmo,ella (ShY C hiCS).
Urethritis
. 18
\rthritis
• 19
• 20
• 21
• 22
• 23
• 24
• 25
• 26
• 27
• 28
• 29 •
a
Lock
s
Suspend
8
End Bl ock
Item: 15 of 33 ~ 1 • M k -<:J 1>- Jil ~· !:';-~
QIO: 1420 ..L ar Pre v ious Next Lab fli!ltues Not es Calcula t o r
A A

9
A previously healthy 23-year-old white woman is brought to the emergency department afte r she is involved
10 in a motor vehicle crash that left her w ith multiple lower extremity fractures. About 48 hours after being
11 admitted to the hospital, she begins to have some trouble breathing and seems confused. Her blood pressure
is 92/45 mm Hg, pu lse is 128/min, respiratory rate is 28/min, and oxygen saturation is 92%. Physical examination
12
shows no penetrating injury to the chest, but some petechiae are found on her neck. An embolus in the pulmonary
• 13 circulation is suspected.
14
• 15 Which of the following is most likely the type of embolus in this patient?
• 16
:
• 17 A. Air embolus
• 18
B. Bacterial embolus
• 19
• 20
C. Cho lesterol embolus
• 21 D. Fat embolus
• 22 E. Thrombotic embolus
• 23
• 24
• 25
• 26
• 27
• 28
• 29 •
a
Lock
s
Suspend
8
End Bl ock
Item: 15 of 33 ~. I • M k <:] t> al ~· ~
QIO: 1420 .l. ar Previous Next lab 'lifllues Notes Calculator


9 The correct answer is D. 75°/o chose this.
10 Fractures are among the most common types of inju ry seen in motor veh icle accidents. Fat embo li are
11 associated with fractu res of long bones, such as the femur. Particles of bone ma r row and other fatty
12
intraosseous t issue enter the circu lation and can lodge in the lungs, kidneys, brain, and other organs. They can
be asymptomatic or manifest cl inical ly as a fat embolus syndrome character ized by pu lmonary distress,
• 13 cutaneous petechiae, and various neurologic abnormal ities .
14 Femur Petechia Embolism Fat embolism Embolus Bone marrow Bone Kidney Asymptomatic Neurology Traffic collision lung

15
A is not correct. 6°/o chose this .
• 16
Ai r embol i result from the introduction of ai r into the ci rcu lation . They can originate from a penetrating chest
• 17 injury, a poorly perfor med abortion, or decompression sickness in divers who resurface too rapidly. As noted in
• 18 the vignette, this patient does not have a penetrating lung injury and her history is not otherwise consistent
with an air embolus .
• 19
Decompression sickness Embolus Embolism Air embolism lung
• 20
• 21
B is not correct. 3°/o chose this .
Bacteria l emboli typica lly originate from a focus of bacteria l endocard it is. Since the stem does not mention a
• 22
prior history of valvu lar disease, a bacterial embolus is not a likely cause .
• 23 Endocarditis Embolism Embolus Infective endocarditis Valvular heart disease

• 24
C is not correct. 3°/o chose this .
• 25
Cho lesterol embol i generally for m in patients with systemic atherosclerosis in whom plaque ruptu re occurs .
• 26 Aortic (arterial) plaque ruptures are most common, resu lting in ischemia in organs distal to the
• 27 thromboembolus . I n a young patient who is otherwise healthy, this is an un likely scenario, and this patient's
• 28 pulmona ry symptoms would not be the resu lt of a systemic circu lation embolus .
Atherosclerosis Cholesterol embolism Ischemia Circulatory system Embolus Cholesterol Embolism Vulnerable plaque Anatomical terms of location
• 29 •

6
lock
s
Suspend
0
End Block
Item: 15 of 33 ~. I • M k <:] t> al ~· ~
QIO: 1420 .l. ar Previous Next lab 'lifllues Notes Calculator
. . .

9
C is not correct. 3°/o chose this.
10
Cho lesterol embol i generally for m in patients with systemic atherosclerosis in whom plaque ruptu re occurs .
11
Aortic (arterial) plaque ruptures are most common, resu lting in ischemia in organs distal to the
12 thromboembolus . In a young patient who is othe rwise healthy, this is an un likely scenario, and this patient's
• 13 pulmona ry symptoms would not be the resu lt of a systemic circu lation embolus .
Atherosclerosis Cholesterol embolism Ischemia Circulatory system Embolus Cholesterol Embolism Vulnerable plaque Anatomical terms of location
14
15 E is not correct. 13% chose this.
• 16 Thrombotic embol i are the most frequent type of embo lus. They can originate in the venous or arterial system .
• 17 Those originating on the venous side, such as a deep venous thrombosis in a calf vein, can dislodge and lead to
a pulmonary embolism. Those that originate in the arteria l system can dislodge and lead to myocardial
• 18 infarction or cerebrovascula r accident. Conside ring this patient is young and has been in good hea lth pr ior to the
• 19 accident makes the possibility of a th rombotic embolus less likely. If respiratory distress developed after the
• 20 patient had lain in bed with he r injuries for severa l days, a deep vein thrombosis leading to a pu lmona ry
embolism is more likely .
• 21
Deep vein thrombosis Pulmonary embolism Myocardial infarction Stroke Embolism Embolus Thrombosis Venous thrombosis Dyspnea Artery
• 22
Cerebrovascular disease Vein Deep vein
• 23
• 24
• 25 Bottom Line:
• 26 Fat embolism shou ld be considered high ly likely in patients with evident or suspected bone fractures and
• 27 respiratory distress fo llowing trauma .
Fat embolism Embolism Dyspnea Respiratory distress Bone Fat
• 28
• 29 •

6
lock
s
Suspend
0
End Block
Item: 15 of 33 ~ 1 • M k -<:J 1>- Jil ~· !:';-~
QIO: 1420 ..L ar Pre v ious Next Lab fli!ltues Not es Calcula t o r
A A

9
FA17 p637.1
10
11
Pulmonary emboli V/Q mismatch, hypoxemia, respiratory alkalosis. CT pulmonary angiography is imaging lest of
Sudden-onset dyspnea, pleuritic chest pain, choice for PE (look for filling defects) (i.
12
tachypnea, tachycardia. Large emboli or
• 13 saddle embolus fJ may cause sudden death.
14 Lines ofZahn are interdigitating areas of pink
15 (platelets, fibrin) and red (RBCs) found on I} in
• 16
thrombi fonned before death; help distinguish
pre- and postmortem thrombi
• 17
T} pes: Fat, Air, T hrombus, Bacteria, \ mniolic An embolus moves like a FAT B.\T.
. 18 Auid, Tumor.
• 19 Fat emboli-associated with long bone fractures
• 20 and liposuction; classic triad of h) poxemia,
• 21
neurologic abnormalities, petechial rash .
Amniotic Auid emboli-can lead to DIC,
• 22
especially postpartum .
• 23 Air emboli-n itrogen bubbles preeipil<tle
• 24 in ascending divers (caisson disease,
• 25 decompression sickness); treat with hyperbaric
• 26
0 2; or, can be iatrogenic 2° to invasive
procedures (eg, cent ratline placement).
• 27
... g . ~ .
• 28
-. . ' \
• 29 • J •

a
Lock
s
Suspend
8
End Bl ock
Item: 15 of 33 ~ 1 • M k -<:J 1>- Jil ~· !:';-~
QIO: 1420 ..L ar Pre v ious Next Lab fli!ltues Not es Calcula t o r
A A

9
10
11
12
• 13
14
15
• 16
• 17
. 18
• 19
FA17 p 636.4
• 20
• 21
Deep venous Blood clot within a deep vein ... swelling, 1ost pulmonarr emboli arise from proximal
thrombosis redness rJ, warmth, pain. Predisposed by deep veins of lower cxtremily.
• 22
Virchow triad (SHE): Use unfractionated heparin or low-molecular-
• 23 • Stasis (eg, post-op, long drive/flight) weight heparins (eg, enoxaparin) for
• 24 llypercoagulability (eg, defect in prophylaxis and acute management.
• 25 coagulation cascade proteins, such as Use oral anticoagulants (eg, warfarin,
• 26
factor Leiden) rivaroxaban) for treatment (long-term
Endothelial damage (exposed collagen pre,·ention).
• 27
triggers clotting cascade) Imaging test of choice is compression ultrasound
• 28 o-dimer lab test used clinically to rule oul DVT with Doppler.
• 29 • (high sensitivity, low specificitv).
a
Lock
s
Suspend
8
End Bl ock
Item: 15 of 33 ~ 1 • M k -<:J 1>- Jil ~· !:';-~
QIO: 1420 ..L ar Pre v ious Next Lab fli!ltues Not es Calcula t o r
A A

9
10 FA17 p299.2

11 Bacterial endocarditis Fe,·er (most common symptom), new murmur, l\ Iitral vah-e is most frequently involved.
12 Roth spots (round '' hite spots on ret ina Tricuspid valve endocarditis is associated ''it h
surrounded by hemorrhage }, Osler nodes IV drug abuse (don't '" tri" dmg~). Associated
• 13
(tender raised lesions on finger or toe pads "ith S au reus, Pseudomonas, and Candida.
14
due to immune complex deposit ion}, Jancwar Culture 8; most likely Coxiella bumetii,
15 lesions (small, painless, erythematous lesions Bartonella spp., HACEK (1-laemophi/us,
• 16 on palm or sole) [!1, glomerulonephritis, Aggregatibacler (formerlr Actinobacillus),
• 17 septic arterial or pulmonary emboli, splinter Cardiobacterium, Eikenella, Kingella)
. 18
hemorrhages [!] on nail bed. Multiple blood • Bacteria FRO~ l JAN E •=
cultures necessary for diagnosis. J<'c,·cr
• 19
Acute-$ aureus (high virulence), Roth spots
• 20 Large vegetations on previously normal Osler nodes
• 21 valves 0 . Rapid onset. 1\ lurmur
• 22 • Subacute-viridans streptococci (low Janeway lesions
• 23
virulence). Smaller vegetations on Anemia
congenitally abnormal or diseased valves. Nail-bed hemorrhage
• 24
Sequela of dental procedures. Gradua l Emboli
• 25 onset.
• 26 S bovis (gallolyticus) is present in colon cancer,
• 27 S epidermidis on prosthetic vakcs.
• 28
Endocarditis may also be nonbactcrial
(marantic/thrombotic) zoto malignancy,
• 29 • L._ - - - - - - - 1-LI_ - ~ - &- - - - 1--- -- -

a
Lock
s
Suspend
8
End Bl ock
Item: 15 of 33 ~ 1 • M k -<:J 1>- Jil ~· !:';-~
QIO: 1420 ..L ar Pre v ious Next Lab fli!ltues Not es Calcula t o r

p ) g
9
hemorrhages (!] on nail bed. Multiple blood • Bacteria FRO~l }.\.NE • :
10 cultures necessary for diagnosis. Fe,·er
11 Acute-S aureus (high virulence). Roth spots
12 Large ,·egetations on previous!)' normal Osler nodes
• 13 valves D. Rapid onset. l\ lurmur
Subacute-viridans streptococci (lo'' Janeway lesions
14
virulence). Smaller vegetations on \ nemia
15 congenitally abnormal or diseased vahes. l\ail-bed hemorrhage
• 16 Sequela of dental procedures. G radual F mboli
• 17 onset.
• 18 S bovis (gallolyticus) is present in colon cancer,
S epidennidis on prosthetic vakes.
• 19
Endocarditis may also be nonbacterial
• 20
(marantic/thrombotic) 2° to malignancy,
• 21 hypercoagulable state, or lupus .
• 22
• 23
• 24
• 25
• 26
• 27 t
• 28
• 29 •
a
Lock
s
Suspend
8
End Bl ock
Item: 13 of 33 ~ 1 • M k -<:J 1>- Jil ~· !:';-~
QIO: 5188 ..L ar Pre v ious Next Lab fli!ltues Notes Calcula t o r

IAA]
A A

9
A 52-year-old woman presents to her physician w ith 3 weeks of blisters on her skin and in her mouth. She
10 says when these blisters rupture they leave painful "raw spots. " On examination, tender erosions are found
11 on her back and trunk. A biopsy of one of the lesions is taken. Direct immunofluorescence against lgG in the
biopsy specimen is shown in the image.
12
• 13
14
15
• 16
• 17
• 18
• 19
• 20
• 21
• 22
• 23
• 24
• 25
• 26
• 27
• 28
Image courtesy of Wikimedia Commons
• 29 •
a
Lock
s
Suspend
8
End Bl ock
Item: 13 of 33 ~. I • M k <:] t> al ~· ~
QIO: 5188 .l. ar Previous Next lab 'lifllues Notes Calculator


9 The correct answer is A. 72°/o chose this.
10 This patient is suffering from pemph igus vulgaris, an autoimmune bl istering disorder. Th is disease most
11 commonly manifests in the fourth to sixth decades of life. It is characterized by frag ile (or flaccid) blisters over
the face, axi lla, t r unk, and mucosa . Large lesions can jeopardize fluid balance and temperatu re regu lation, and
12
can be sources of infection; thus severe cases may be life-threaten ing. The disease is caused by an autoimmune
13 reaction against desmoglein 3, a component of desmosomes (a lso ca lled macula adherens). Desmosomes are
14 "spot-junctions" that attach epithel ial cells to one another. As shown in the direct immunofluorescence image,
15 immunog lobul in is deposited in a net-l ike pattern surrounding keratinocytes. This condition is treated with
corticosteroids and other immunosuppressive medications .
• 16 Desmoglein-3 Pemphigus Pemphigus vulgaris Immunofluorescence Desmoglein Corticosteroid Autoimmunity Antibody Keratinocyte Direct fluorescent antibody
• 17
Axilla Autoimmune disease Immunosuppressive drug Epithelium Mucous membrane Immunosuppression Desmosome
• 18
B is not correct. 4°/o chose this .
• 19
Gap junctions, composed of connexons, allow for communication between adjacent cel ls, classical ly in cardiac
• 20
muscle cel ls. These are not involved in pemphigus vu lgaris .
• 21 Pemphigus Pemphigus vulgaris Cardiac muscle Gap junction

• 22
C is not correct. 17% chose this .
• 23
Hemidesmosomes are simi lar to desmosomes, but are found exclusively between the basa l layer of
• 24 keratinocytes and the basement membrane. Autoimmune disease targeting hemidesomsomes causes bul lous
• 25 pemphigoid, a separate, mi lder disease characterized by tense bu llae that do not easily rupture. Bu llous
• 26
pemphigoid tends to occur in the elde rly, and oral invo lvement is less common . Di rect immunofluorescence
imag ing in this condition demonstrates a linear pattern along the basement membrane, rather than the net-like
• 27 pattern seen in the case above .
• 28 Autoimmune disease Immunofluorescence Basement membrane Bullous pemphigoid Hemidesmosome Desmosome Keratinocyte Direct fluorescent antibody Blister

• 29 • Cutaneous condition Autoimmunity Stratum basale Pemphigoid

6
lock
s
Suspend
0
End Block
Item: 13 of 33 ~. I • M k <:] t> al ~· ~
QIO: 5188 .l. ar Previous Next lab 'lifllues Notes Calculator

9
Hemidesmosomes are sim ilar to desmosomes, but are found exclusively bet ween t he basal layer of
10
keratinocyt es and the basement membrane. Autoimmune disease ta rgeting hemidesomsomes causes bu llous
11 pemphigoid, a separate, mi lder disease characterized by tense bul lae t hat do not easi ly rupture . Bu llous
12 pemphigoid t ends to occur in t he elderly, and oral involvement is less common . Direct immunofluorescence
imag ing in this condit ion demonst rates a linea r pattern along t he basement membrane, rather than t he net-like
13
pattern seen in the case above .
14 Autoimmune disease Immunofluorescence Basement membrane Bullous pemphigoid Hemidesmosome Desmosome Keratinocyte Direct fluorescent antibody Blister
15 Cutaneous condition Autoimmunity Stratum basale Pemphigoid
• 16
D is not correct. 4°/o chose this .
• 17
Zona adherens, or inter mediate junct ions, are found j ust deep to t he zona occludens. These junctions are not
• 18 involved in pemph igus vu lgaris .
• 19 Pemphigus Pemphigus vulgaris

• 20
E is not correct. 3°/o chose this .
• 21
Zona occludens, or t ight junctions, are located close to t he surface of epithelial cells. They prevent diffusion
• 22 between cel ls. These junctions are not invo lved in pemph igus vu lgaris .
• 23 Pemphigus Pemphigus vulgaris Epithelium Tight junction

• 24
• 25 Bottom Line:
• 26
Pemph igus vu lgaris is an autoimmune blistering disease of skin and mucosa, caused by antibod ies direct ed
• 27 against desmosomes (macula adherens) .
• 28 Pemphigus vulgaris Antibody Pemphigus Autoimmune disease Desmosome Mucous membrane Autoimmunity

• 29 •

6
lock
s
Suspend
0
End Block
Item: 13 of 33 ~ 1 • M k -<:J 1>- Jil ~· !:';-~
QIO: 5188 ..L ar Pre v ious Next Lab fli!ltues Notes Calcula t o r
A A

9
FA17 p 4 52.1
10 Blistering skin disorders
11 Pemphigus vulgaris Potentially fatal autoimmune skin disorder with lgC antibody against desmoglein (component of
12 desmosomes, which connect keratinocytes in the stratum spinosum).
13 Flaccid intraepidermal bullae caused b) acantholysis (separation ofkeratinocytes, resembling a
14 "row of tombstones"); oral mucosa is also in,olved. T) pe II hypersensitivity reaction.
lmmunoAuorcscencc rc,·cals antibodies around epidermal cells in a reticular (net-like) pattern
15
' ikolskr sign<±> (separation of epidermis upon manual stroking of skin).
• 16
Bullous pemphigoid Less severe than pemphigus vulgaris. Involves lgC antibody against hemidesmosomes (epidermal
• 17
basement membrane; antibodies are " bullow" the epidermis).
. 18 Tense blisters containing eosinophils affect skin but spare oral mucosa.
• 19 lmmunoAuorcscencc reveals linear pattern at epidermal-dermal junction [!J.
• 20 ~ikolsky sign e.
• 21 Dermatitis Pruritic papules, vesicles, and bullae (often found on elbows) D. Deposits of IgA at tips of dermal
• 22 herpetiformis papillae. Associated with celi<1c dise;~se. Treatment: dapsone, gluten-free diet.
• 23 Erythema multi forme Associated with infections (cg, Mycoplasma pneumoniae, 1-1$\1}, drugs (cg, sulfa dmgs, ~-l actams,
• 24
phenytoin), cancers, autoimmune disease. Presents with multiple types of lesions-maculcs,
papules, vesicles, target lesions (look like targets with multiple rings and dusky center showing
• 25
epithelial disruption)
• 26
Stevens-Johnson Characterized by fever, bullae formation and necrosis, sloughing of skin at dermal-epidermal
• 27 syndrome junction, high mortality rate. Typically 2 mucous membranes are im·olved [!I rn.
and targetoid
• 28 skin lesions may appear, as seen in erythema multifonne. Usually associated with ad,·crsc drug
• 29 • reaction. A more severe form of Stc\ ens-johnson S) ndrome (SJS) with > 30% of the body surface

a
Lock
s
Suspend
8
End Bl ock
Item: 13 of 33 ~. I • M k <:] t> al ~· ~
QIO: 5188 .l. ar Previous Next lab 'lifllues Notes Calculator

9 Dermatitis Pruritic papules, vesicles, and bullae (often found on elbows) D. Deposits of IgA at tips of dermal
10 herpetiform is papillae. Associated with celiac disease. Treatment: dapsone, gluten-free diet.
11 Erythema multiforme Associated with infections (eg, Mycoplasma pneumoniae, HSV), drugs (cg, sulfa drugs, ~-l actams,
12
phenytoin), cancers, autoimmune disease. Presents with multiple types of lesions-macules,
papules, vesicles, target lesions (look like targets with multiple rings and dusky center showing
13
epithelial disruption) D.
14
Stevens-Johnson Characterized by fever, bullae forma tion and necrosis, sloughing of skin at dermal-epidermal
15
syndrome junction, high mortality rate. Typically 2 mucous membranes are involved [!! C), and targetoid
• 16 skin lesions may appear, as seen in erythema multiformc. Usually associated with adverse drug
• 17 reaction. A more severe form of Stevens-Johnson syndrome (SJS) with> 30% of the body surface
• 18 area involved is toxic epidermal necrolysis (TEN). 10-30% involvement denotes SJS-TE .
• 19
• 20
• 21
• 22
• 23
• 24
• 25

• 26
• 27

• 28
• 29

6
lock
s
Suspend
0
End Block
10
11 FA17 p 447.1
12 Epithelial cell junctions
13
14
15 )>-- - - Tight junction (zonula occludens)-prevents paracellular
. ~
E-c.adhenn movement of solutes. composed of claudins and occlud1ns.
• 16
• 17 Actin / ) ~ } - Adhefens junction (belt desmosome. zonula adherensl-below
filaments _, ttght JUnctiOil, forms 'belt' connecbng aebn cytosketetons of
• 18 adjacent cells with CADherins (C..~· -dependent adhesion

• 19 •
L proteinS). Loss of E-cadherin promotes metastasis.
Cytokeratin -
~ Dts•oso~~~e (spot desmosome. macula adherens)-structural
• 20 Desmoplakin ~: support via Intermediate filament interactions. Autoantibodies
• 21 - pemphigus vulgaris.

• 22
• 23
Connexon
with central
( 1--- ~p junction- channel proteins called connexons permit
electrical and chemical communication between cells.
channel
• 24
~ Cell membrane
• 25
Basolateral iiiiiiiW~-IIIiiiiR::;;.- Basement membrane
• 26
lntegrins-membrane proteins that maintain =./ \_ Htllicltsmosome-connects kerabn in basal cells to
• 27 integrity of basolateral membrane by binding underly1ng basement membrane Autoantibodies - bullous
to collagen and lam1nin in basement membrane. pemphtgoid. (Hem1desmosomes are down "bullow").
• 28
• 29 •
a
Lock
s
Suspend
8
End Block
Item: 16 of 33 ~ 1 • M k -<:J 1>- Jil ~· !:';-~
QIO: 4 0 57 ..L ar Pre v ious Next Labfli!llues Not es Calcula t o r

IAA]
A A

9
After a night of heavy drinking, a 37-year-old man experiences pain, warmth, swell ing, and redness around
10 his big toe, shown here radiographically. He had been previously warned against drinking heavily, as it cou ld
11 exacerbate the pain in his toe .
12
13
14
15
• 16
• 17
. 18
• 19
• 20
• 21
• 22
• 23
• 24
• 25
• 26
• 27
• 28
• 29 • Wh ich of the following enzymes is responsible for the final conversion of the product responsible for th is patient's

a
Lock
s
Suspend
8
End Bl ock
9
10
11
12
13
14
15
• 16
• 17
• 18
• 19
Which of the fo llowing enzymes is responsible for the fina l conversion of the product responsible fo r this patient's
• 20 pa in?
• 21
:
• 22 A. Adenosine deaminase
• 23
B. Cystath ion ine synthase
• 24
• 25 C. Hypoxanth ine guan ine phosphoribosyltransferase
• 26 D. Phenylalan ine hyd roxylase
• 27
E. Xanthine oxidase
• 28
• 29 •
a
Lock
s
Suspend
8
End Block
Item: 16 of 33 ~ 1 • M k -<:J 1>- Jil ~· !:';-~
QIO: 4 0 57 ..L ar Prev ious Next Labfli!ll ues Not es Calculat o r

A A

9
10
The correct answer is E. 89°/o chose this.
11
This patient has gout, which is characterized by Nucleoc acids Nuc:leoc: acids
precipitation of monosod ium urate crystals in the joints.
12 The diagram shows that xanthine is converted into uric
1
y (d• oxy
u f)

I
Nueleoi>CMs
13 acid by xanthine oxidase, the final step of the purine MP ~ ----- --- MP ------- --------- ~ '
14 salvage pathway. Allopurinol, a medication used to
prevent gout attacks, inhibits xanthine oxidase. Acute
l G·--"" .t "-. •xy~ ~
15
gout attacks are instead treated with nonsteroidal anti- a....
PR~ / PRPP1 /z
Hypox~ Adenine
16 inflammatory drugs (NSAIDs) like indomethacin.
• 17 NSAIDs are useful in the acute setting because they /
3--
• 18
treat pain and inflammation, which are the primary Xanu
I ADA
issue in an acute attack.
• 19 Xanthine oxidase Allopurinol Indometacin Uric acid Purine Xanthine Gout u.
1 2:PNP

• HCPRT
• 20 Nonsteroidal anti-inflammatory drug Anti-inflammatory Nucleotide salvage f
In lammat1on Pharmaceutical drug
• 21
• 22 A is not correct. 2°/o chose this .
• 23 Adenosine deaminase converts aden osine to inosine, another step in the purine pathway. Deficiency of this
enzyme can cause severe combined immunodeficiency disease .
• 24
Adenosine deaminase Severe combined immunodeficiency Purine Enzyme Inosine Adenosine Immunodeficiency
• 25
• 26
B is not correct. 1 °/o chose this .
Cystath ionine synthase converts homocysteine to cystath ion ine w ith the help of vitamin 8 6 . It is not in the
• 27
purine salvage pathway, and its deficiency leads to homocystinuria. Homocystinuria due to cystath ion ine
• 28 synthase deficiency is treated by decreasing methionine, increasing cysteine, 8 12 , and folate in diet.
• 29 Homocystinur'a Cysteine Pur'ne Vitamin 612 Cystathionine beta synthase Fol c acod Cystathionine B vitamins Methionine Homocysteine Vitamin

a
Lock
s
Suspend
8
End Block
Item: 16 of 33 ~. I • M k <:] t> al ~· ~
QIO: 4057 .l. ar Previous Next lab 'lifllues Notes Calculator


9
B is not correct. 1 °/o chose this.
10
Cystath ionine synthase converts homocysteine to cystath ion ine with the help of vitamin B6 . It is not in the
11 purine sa lvage pathway, and its deficiency leads to homocystinuria. Homocystinuria due to cystath ion ine
12 synthase deficiency is t reated by decreasing methionine, increasing cysteine, B12 , and folate in diet .
Homocystinuria Cysteine Purine Vitamin 812 Cystathionine beta synthase Folic acid Cystathionine B vitamins Methionine Homocysteine Vitamin
13
14 C is not correct. 7°/o chose this.
15 Hypoxanthine guanine phosphoribosyl transferase deficiency is associated with Lesch-Nyhan syndrome. Th is
16
enzyme converts hypoxanthine to inosine monophosphate and guanine to guanosine monophosphate, but is not
the fina l step in the pathway .
• 17 Transferase Hypoxanthine-guanine phosphoribosyltransferase lesch-Nyhan syndrome Hypoxanthine Inosinic acid Enzyme Guanine Inosine Guanosine
• 18 Guanosine monophosphate
• 19
Dis not correct. 1°/o chose this .
• 20
Phenylalanine hyd roxylase converts phenylalanine to tyrosine and is not in the purine salvage pathway. Its
• 21
deficiency leads to phenylketonuria, in which tyrosine becomes essential to the diet. Treat with decreased
• 22 phenylalan ine in the diet and increase tyrosine consumption .
• 23 Phenylketonuria Phenylalanine hydroxylase Phenylalanine Purine Tyrosine

• 24
• 25
Bottom Line:
• 26
Acute gout is associated with heavy drinking, great toe invo lvement, and ur ic acid deposit ion. The final step of
• 27 the pathway is conversion of xanthine to uric acid by the enzyme xanthine oxidase.
• 28 Xanthine oxidase Uric acid Gout Enzyme Xanthine Toe

• 29 •

6
lock
s
Suspend
0
End Block
Item: 16 of 33 ~ 1 • M k -<:J 1>- Jil ~· !:';-~
QIO: 4 0 57 ..L ar Pre v ious Next Labfli!llues Not es Calcula t o r
A A

9
FA17 p 440.1
10 Gout
11
FINDINGS Acute inflammatory monoarthritis caused by precipitation of monosodium urate crystals in
12 joints r.J. More common in males. Associated with hyperuricemia, which can be caused by:
13 Underexcretion of uric acid (90% of patients)-largely idiopathic, potentiated by renal failure;
14 can be exacerbated b) certain medications (eg, thiazide diuretics).
15
Q,·erproduction of uric acid (10% of patients)-Lesch-1\yhan syndrome, PRPP excess, t cell
tumO\·er (eg, tumor lysis S}11drome}, von Gierke disease.
16
Crystals are needle shaped and 0 birefringent under polarited light (yellow under parallel light,
• 17 blue under perpendicular light : ).
. 18 SYMPTOMS Asrmmetric joint distribution. Joint is swollen, red, and painful. Classic manifestation is painful
• 19 MTP joint of big toe (podagra). Tophus formation ~ (often on external ear, olecranon bursa,
• 20 or Achilles tendon). Acute attack tends to occur after a large meal with foods rich in purines
• 21 {eg, red meat, seafood), trauma, surgery, dehydration, diuresis, or alcohol consumption (alcohol
metabol ites compete for same excretion sites in kidney as uric acid - l uric acid secretion and
• 22
subsequent buildup in blood).
• 23
TREATMENT Acute: lSAIDs (eg, indomet hacin), glucocorticoids, colchicine.
• 24
Chronic (preventive): xanthine oxidase inhibitors (eg, allopurinol, febuxostat).
• 25
• 26
• 27
• 28
• 29 •
a
Lock
s
Suspend
8
End Bl ock
Item: 16 of 33 ~ 1 • M a rk -<:J 1>- Jil ~· !:';-~
QIO: 4 0 57 ..L Pre v ious Next Labfli!llues Not es Calcula t o r


9
10
11
12
13
14
15
16
• 17
. 18
• 19 FA17 p457.2

• 20 Gout drugs
• 21 Chron1c gout drugs (preventive)
• 22 Allopurinol Competitive inhibitor of xanthine oxidase. Diet - - Purines Nucleic acids

• 23
• 24
! conversion of hypoxanthine and xanthine to
urate. Also used in lymphoma and leukemia !
Hypoxanthine
to prevent tumor lysis- associated urate
• 25
nephropathy. t concentrations or azathioprine
! Xanthine
oxidase )
• 26
• 27
and 6-MP (both normally mctabol ized by
xanthine oxidase). -
Xanthine

.
- Allopurinol.
febuxostat

• 28 Febuxostat Inhibits xanthine oxidase. ! Xanthine


OXIdase

Plasma Urate crystals - - Gout


• 29 • Peqloticase Recombinant uricase that catalv:tes metabolism
a
Lock
s
Suspend
8
End Bl ock
9
urate. Also used in lymphoma and leukemia
10 Hypoxanthine
to pre\'ent tumor lysis-associated urate
11
12
nephropathy. t concentrations of azathioprine
and 6-:\IP (both normally mctaboli7cd by
l Xanthine
oxidase )
Xanthine - Allopurinol.
xanthine oxidase). - febuxostat
13 Xanthine
Febuxostat Inhibits xanthine oxidase. O)C]dase
14
Pegloticase Recombinant uricase that catalyzes metabolism Plasma - - Urate crystals - - Gout
15 uric acid deposited
16
of uric acid to allantoin (a more "ater-soluble in joints
product).
• 17
Probenecid Inhibits reabsorption of uric acid in proximal Tubular
• 18
conmluted tubule (also inhibits secretion of
~
reabsorptiOn
• 19
penicillin). Can precipitate uric acid calculi. Probenecid and
• 20 high·dose salicylates
Acute gout drugs Tubular
• 21
~
secretion
~
NSAIDs Any full -dose SAID (eg, naproxcn,
• 22 indomethacin). Avoid sal icylates (may decrease Diuretics and
Urine low-dose salicytates
• 23 uric acid excretion, particularly at low doses).
• 24 Glucocorticoids Oral, intra-articular, or parenteral.
• 25 Colchicine Binds and stabilizes tubulin to inhibit
• 26 microtubule polymerization, impairing
• 27 neutrophil chemotaxis and degranulation .
• 28
Acute and prophylactic value. C l side effects.
• 29 •
a
Lock
s
Suspend
8
End Block
Item: 18 of 33 ~ 1 • M k -<:J 1>- Jil ~· !:';-~
QIO: 3484 ..L ar Pre v ious Next Lab fli!ltues Not es Calcula t o r
A A

9
Patients in the first phase of Paget disease of the bone usua lly have serum ca lcium levels between 8.4 and
10 10.5 mg/dl and phosphate levels between 2.7 and 4.0 mg/d l.
11
Alkaline
Phosphate
12 Choice phosphatase
(mgldL)
(U/L)
13
14 A 20 3.7
15 B 50 5.2
16
c 100 54
• 17
0 400 3.5
. 18
• 19 E 1500 5.2
• 20
• 21 During the second phase of the disease, which of the laboratory values from the tab le above is most likely to be
• 22 seen?
• 23 :
• 24 A
• 25 B
• 26
c
• 27
• 28
D
• 29 • E

a
Lock
s
Suspend
8
End Bl ock
Item: 18 of 33 ~. I • M k <:] t> al ~· ~
QIO: 3484 .l. ar Previous Next lab 'lifllues Notes Calculator


9
The correct answer is D. 51°/o chose this.
10
Paget disease of the bone is character ized by four stages : (1) an initial osteolytic lesion invo lving marked bone
11 resorption, (2) a mixed period of disorganized bone formation, (3) a thi rd stage of osteoblastic-induced sclerosis
12 and ( 4) a fina l, quiescent stage in wh ich osteoclastic and osteoblastic activity is minimal. The prima ry
abnorma lity is the over production and overactivity of osteoclasts, which are derived from the bone marrow.
13
Al kal ine phosphatase is a marker of bone for mation. During the period of haphaza rd bone formation, bone-
14 specific alkal ine phosphatase levels are elevated. Rarely, the total alkaline phosphatase level is norma l, whereas
15 the bone-specific al kal ine phosphatase leve l is elevated. Although the al kal ine phosphatase was histor ica lly used
16 to characterize the severity of the disease, the level of elevation of alka line phosphatase rarely exceeds 10
times the upper limit of normal. The serum phosphate level stays nor mal, and the ca lci um level may be normal
• 17
or sl ightly elevated .
18 Alkaline phosphatase Bone resorption Osteoclast Osteoblast Paget' s disease of bone Bone Phosphate Bone marrow Phosphatase lesion Ossification Blood plasma

• 19 Calcium Alkalinity Serum (blood)

• 20
A is not correct. 7°/o chose this .
• 21
I n the second stage of Paget disease, there is increased bone turnover, which would lead to an increase in
• 22 alkaline phosphatase; however, this answe r choice has a va lue of 20 U/L , which is an abnormally low level of
• 23 alkaline phosphatase .
Alkaline phosphatase Paget' s disease of bone Bone remodeling Alkalinity Phosphatase Bone
• 24
• 25 B is not correct. 9°/o chose this .
• 26 A value of 50 U/L is with in the norma l range of alkaline phosphatase, which wou ld be found in the first phase of
• 27 Paget disease, whe reas in the second phase an increase in al kal ine phosphatase wou ld be expected because of
the increased bone turnover .
• 28 Alkaline phosphatase Paget' s disease of bone Bone remodeling Alkalinity Phosphatase Bone
• 29 •

6
lock
s
Suspend
0
End Block
Item: 18 of 33 ~. I • M k <:] t> al ~· ~
QIO: 3484 .l. ar Previous Next lab 'lifllues Notes Calculator


9
B is not correct. 9°/o chose this.
10
A value of 50 U/L is with in the norma l range of alkal ine phosphatase, which wou ld be found in the first phase of
11 Paget disease, whe reas in the second phase an increase in alkal ine phosphatase wou ld be expected because of
12 the increased bone turnover.
Alkaline phosphatase Paget' s disease of bone Bone remodeling Alkalinity Phosphatase Bone
13
14 C is not correct . 20% chose this.
15 A value of 100 U/L is indeed elevated above the norma l range of alkal ine phosphatase. However, we wou ld
16
expect th is elevation to be around five or six t imes that of a no rma l alka line phosphatase va lue du ring the
second stage of the disease. Furthermore, the phosphate level in th is cho ice is elevated; Paget disease is
• 17
associated with a norma l leve l of phosphate .
18 Alkaline phosphatase Phosphate Phosphatase Alkalinity Paget' s disease of bone

• 19
E is not correct. 13% chose this .
• 20
A value of 1500 U/L is > 10 times the normal level of alkaline phosphatase, and it is not like ly to be found, even
• 21 in patients with highly active Paget disease. Highly elevated alkaline phosphatase levels are typically found in
• 22 patients with involvement of the skul l and at least one othe r site . I n addition, this phosphate leve l is elevated,
wh ich is not typical ly seen in Paget disease .
• 23
Alkaline phosphatase Paget' s disease of bone Phosphatase Phosphate Alkalinity
• 24
• 25
• 26 Bottom Line:
• 27 Paget disease of the bone is characterized by an elevated alka line phosphatase level ( rarely > 10 t imes the
• 28
norma l limit), but the level of phosphate usua lly stays with in norma l range .
Alkaline phosphatase Paget' s disease of bone Phosphate Alkalinity Phosphatase Bone
• 29 •

6
lock
s
Suspend
0
End Block
Item: 18 of 33 ~. I • M k <:] t> al ~· ~
QIO: 3484 .l. ar Previous Next lab 'lifllues Notes Calculator


9 Alkaline phosphatase Paget' s disease of bone Phosphatase Phosphate Alkalinity

10
11
Bottom Line:
12
Paget disease of the bone is characterized by an elevated alkaline phosphatase level (rarely > 10 t imes the
13 normal limit), but the level of phosphate usua lly stays with in normal range.
14 Alkaline phosphatase Paget' s disease of bone Phosphate Alkalinity Phosphatase Bone

15
16
• 17 1iU;fii!1J•J for year:[ 2017
FI RST AID FA CT S
"
18
• 19 FA17 p436.2
• 20
Paget disease of bone Common, localized disorder of bone Hat size can be increased due to skull
• 21 (osteitis deformans) remodeling caused by t osteoclastic activity thickening r.J; hearing loss is common due to
• 22 followed by t osteoblastic activity that forms auditory foramen narrowing.
• 23 poor-quality bone. Serum Ca 2+, phosphorus, Stages of Paget disease:
and PTH levels are normaL t ALP. Mosaic • Lytic-osteoclasts
• 24
pattern of woven and lamellar bone (osteocytes • Mixed-osteoclasts + osteoblasts
• 25
within lacunae in chaotic juxtaposit ions); long • Sclerotic-osleoblasts
• 26 bone chalk-stick fractures. t blood flow from • Quiescent-minimal osteoclast/osteoblast
• 27 t arteriovenous shunts may cause high-output activity
• 28 heart failure. t risk of osteogen ic sarcoma. Treatment: bisphosphonatcs.
• 29 •

6
lock
s
Suspend
0
End Block
Item: 19 of 33 ~ 1 • M k -<:J 1>- Jil ~· !:';-~
QIO: 1664 ..L ar Pre v ious Next Lab fli!ltues Notes Calcula t o r

IAA]
A A

9
A patient visits the doctor because her parents are worried that she is not growing proportionately. The
10 patient's parents are also of very short stature. On examination the patient appears t o have a normal trunk
11 size, but has a large head and short arms and legs.
12
13 Which of the following is associated with this condition?
14 :
15 A. Advanced maternal age
16 B. Advanced paternal age
• 17
C. Diethylstilbestrol exposure in utero
18
D. Iodide deficiency in utero
• 19
• 20 E. Thalidomide exposure in utero
• 21
• 22
• 23
• 24
• 25
• 26
• 27
• 28
• 29 •
a
Lock
s
Suspend
8
End Bl ock
Item: 19 of 33 ~. I • M k <:] t> al ~· ~
QIO: 1664 .l. ar Previous Next lab 'lifllues Notes Calculator


9
10
The correct answer is B. 50°/o chose this.
This patient has achondroplasia, a syndrome in which affected
11
patients have growth defects that are particularly pronounced in
12 the limbs. Achondroplasia is due to a defect in the fibroblast
13 growth factor receptor 3 (FGFR3), which causes an abnormal ity in
14 carti lage formation. Gain-of-function mutations in FGFR3 cause a
constitutively active receptor with impaired chondrocyte
15
proliferation and endochondra l bone formation. As shown in the
16 images, it results in short limbs with a normal-sized trunk and a
• 17 large head relative to the limbs (hence the disproportionate growth
described in the vignette). It is associated with advanced paternal
18
age .
19 Chondrocyte Achondroplasia Fibroblast Fibroblast growth factor receptor 3 Growth factor Images courtesy of Wikimedia Commons
• 20 Fibroblast growth factor Cartilage Ossification Fibroblast growth factor receptor Mutation Bone
Receptor (biochemistry) Endochondral ossification
• 21
• 22 A is not correct. 14% chose this .
• 23 Advanced maternal age (defined as a first pregnancy after 35 years of age) is associated with Down syndrome .
• 24
As a woman ages, the chance of nondisjunction increases. Although advanced maternal age is not associated
with disproportionate growth, it is associated with an increased rate of twinning and preeclampsia .
• 25 Down syndrome Nondisjunction Advanced maternal age Pregnancy Pre-eclampsia
• 26
C is not correct. 9°/o chose this .
• 27
Diethylstilbestrol exposure increases the risk of vaginal clear cell adenocarcinoma in the daughters of mothers
• 28 who took this drug during pregnancy. It was originally approved to prevent miscarriages and to treat vaginitis,
• 29 • menopausal symptoms, and postpartum lactation; it is no lonqer used for these indications. Althouqh it remains •
6
lock
s
Suspend
0
End Block
Item: 19 of 33 ~. I • M k <:] t> al ~· ~
QIO: 1664 .l. ar Previous Next lab 'lifllues Notes Calculator

9 C is not correct. 9°/o chose this.


10 Diethylstilbestrol exposure increases the risk of vaginal clea r cel l adenocarcinoma in the daughters of mothers
11 who took this drug during pregnancy. I t was or iginally app roved to prevent miscarriages and to treat vaginitis,
12
menopausal symptoms, and postpartum lactation; it is no longe r used for these indications. Although it remains
approved for use in metastatic prostate cance r, he re, too, it is no longer used. It is not associated with
13 disproportionate growth.
14 Clear-cell adenocarcinoma of the vagina Diethylstilbestrol Prostate cancer Vaginitis Adenocarcinoma Clear-cell adenocarcinoma Menopause Metastasis Pregnancy

15 Cancer Postpartum period lactation

16
D is not correct. 12% chose this .
• 17
I odine deficiency in utero causes congen ita l goiter or hypothyroidism. This can lead to cretinism, a congen ita l
18 hypothyroidism manifesting with decreased activity, poor feed ing and weight gain, smal l statu re or poor growth,
19 jaund ice, hypotonia, and hoarse cry. It is more common in areas of endemic goiter. It is not associated with
• 20
disproportionate growth .
Hypothyroidism Cretinism Goitre Congenital hypothyroidism Iodine deficiency Jaundice Hypotonia Iodine Congenital disorder In utero Endemic goitre
• 21
• 22
E is not correct. 15% chose this .
Thal idomide use by pregnant women is associated with fetal limb deformities known as phocomel ia (often
• 23
refer red to as "flipper limbs" based on thei r physica l appearance). Original ly the drug was prescribed to
• 24 pregnant women as an antiemetic to decrease symptoms of morning sickness. It is no longe r app roved for
• 25 pregnant women, but is still used fo r treating diseases as dive rse as leprosy and multiple myeloma. It is not
• 26 associated with disproportionate growth .
Antiemetic Thalidomide Multiple myeloma Phocomelia leprosy Morning sickness Fetus
• 27

• 28
• 29 Bottom Line:

6
lock
s
Suspend
0
End Block
Item: 19 of 33 ~. I • M k <:] t> al ~· ~
QIO: 1664 .l. ar Previous Next lab 'lifllues Notes Calculator
• • ••• - • - - , -,u
9 Clear-cell adenocarcinoma of the vagina Diethylstilbestrol Prostate cancer Vaginitis Adenocarcinoma Clear-cell adenocarcinoma Menopause Metastasis Pregnancy
10 Cancer Postpartum period lactation
11
D is not correct. 12% chose this.
12
I odine deficiency in utero causes congen ita l goiter or hypothyroidism. This can lead to cretinism, a congen ita l
13
hypothyroidism manifesting with decreased activity, poor feed ing and weight gain, smal l statu re or poor growth,
14 jaund ice, hypotonia, and hoarse cry . It is more common in areas of endemic goite r. It is not associated with
15 disproportionate growth.
Hypothyroidism Cretinism Goitre Congenital hypothyroidism Iodine deficiency Jaundice Hypotonia Iodine Congenital disorder In utero Endemic goitre
16
• 17 E is not correct. 15% chose this .
18 Thal idomide use by pregnant women is associated with fetal limb deformities known as phocomel ia (often
refer red to as "flippe r limbs" based on thei r physica l appearance). Original ly the drug was prescribed to
19
pregnant women as an antiemetic to decrease symptoms of morning sickness. It is no longe r app roved for
• 20 pregnant women, but is still used fo r treating diseases as dive rse as leprosy and multiple myeloma . It is not
• 21 associated with disproportionate growth .
Antiemetic Thalidomide Multiple myeloma Phocomelia leprosy Morning sickness Fetus
• 22
• 23
• 24 Bottom Line:
• 25
Achond roplasia is an autosomal dominant for m of dwarfism in wh ich the affected individual inherits one no rma l
• 26 and one defective copy of FGFR3. They have overal l short statu re with short limbs, a large head relative to the
• 27 limbs, and a near norma l trun k size. This diso rder is associated with advanced paternal age .
Achondroplasia Fibroblast growth factor receptor 3 Dwarfism Dominance (genetics) Autosome Short stature
• 28
• 29 •

6
lock
s
Suspend
0
End Block
Item: 19 of 33 ~. I • M k <:] t> al ~· ~
.l. ar
- .. . . . .. . . - . - . - .. .
QIO: 1664 Previous Next lab 'lifllues Notes Calculator

9
'' . - '' . - ... - . ''
ref erred to as "flipper limbs" based on thei r physical appearance). Original ly the drug was prescribed to
10 pregnant women as an antiemetic to decrease symptoms of morning sickness. It is no longer approved for
11
pregnant women, but is still used for treating diseases as diverse as leprosy and multiple myeloma . It is not
associated with disproportionate growth.
12 Antiemetic Thalidomide Multiple myeloma Phocomelia leprosy Morning sickness Fetus
13
14
15
Bottom Line:
16 Achondroplasia is an autosomal dominant form of dwarfism in wh ich the affected individua l inherits one norma l
and one defective copy of FGFR3. They have overall short stature with short limbs, a large head relative to the
• 17
limbs, and a near normal trun k size. This disorder is associated with advanced paternal age .
18 Achondroplasia Fibroblast growth factor receptor 3 Dwarfism Dominance (genetics) Autosome Short stature

19
• 20
• 21 1iU;fii!1J•J for year:[ 2017
FI RST AID FAC TS
"
• 22
• 23
FA11 p435.1
• 24
Achondroplasia Failure of longitudinal bone growth (endochondral ossification) -+ short limbs. ~embranous
• 25 ossification is affected -+ large head relative to limbs. Constitutive acti,·ation of fi broblast growth
• 26 factor receptor (FGFR3) actually inhibits chondrocyte prol ife ration. > 85% of mutations occur
• 27 sporadically; autosomal dom inant with full penetrance (homozygosity is lethal). Most com mon
• 28 cause of dwa rfism .

• 29 •

6
lock
s
Suspend
0
End Block
Item: 17 of 33 ~ 1 • M k -<:J 1>- Jil ~· !:';-~
QIO: 3920 ..L ar Pre v ious Next Lab fli!ltues Not es Calcula t o r

IAA]
A A

9
A 62-year-old white woman visits her family doctor with complaints of back pain located over the spine.
10 When asked about the onset of pa in, she recalls a fa ll from tripping while walking several weeks ago. She
11 has also suffered from severe osteoarthritis of the knees for the past 10 years, which has sign ificantly limited
her physical activity . Her mother died at age 58 years from a massive pu lmonary embolus following hip
12
replacement surgery after fall ing .
13
Alkaline
Choice Calcium Phosphorus
14 phosphatase
15
A ~ f ~
16
B normal normal normal
• 17
18 c normal normal f
19
• 20
D
f ~ f
• 21
• 22
Using the table above, which of the following sets of serum laboratory findings best describes her condition?
• 23 :

• 24
A
• 25 B
• 26 c
• 27
D
• 28
• 29 •
a
Lock
s
Suspend
8
End Bl ock
Item: 17 of 33 ~ 1 • M k -<:J 1>- Jil ~· !:';-~
QIO: 3920 ..L ar Pre v ious Next Lab fli!ltues Not es Calcula t o r

The correct answer is B. 63°/o chose this.


A A

9
10 The patient's lower back pa in and history suggest the possibility of a
vertebral compression fracture (see X-ray). These fractures are more
11
likely to occur in patients with osteoporosis, a metabolic bone disease
12 characterized by decreased bone mass. This patient has several risk factors
13 for osteoporosis: postmenopausal female, low physical activity level, and
14 family history of osteoporosis (as evidenced by her mother, who fell at age 58
and needed hip replacement surgery).
15
Osteoporosis can be caused by impaired synthesis or increased resorption of
16
bone matrix protein. Predisposing factors include postmenopausal state,
17 family history, small/thin build, physical inactivity, hyperthyroidism, smoking,
18 hypercortisolism, and calcium deficiency. Lab tests in osteoporosis reveal
19 normal serum calcium, normal serum phosphorus, and normal or decreased
alkaline phosphatase levels. Treatments for osteoporosis include weight-
• 20
bearing exercise, calcium supplementation, hormone replacement therapy,
• 21 and bisphosphonates .
• 22 Vertebral compression fracture Osteoporosis Hyperthyroidism Alkaline phosphatase
• 23 Hormone replacement therapy (menopause) Hormone replacement therapy Bisphosphonate Protein Phosphorus
• 24
Hormone Physical exercise Back pain Menopause Cushing's syndrome Hypocalcaemia Calcium Fracture X-ray Image courtesy of James
• 25 Heilman, MD
Hip replacement Hormone replacement therapy (male-to-female) Bone Bone fracture Blood plasma
• 26
Bone resorption Metabolism Low back pain Bone disease Tobacco smoking
• 27
A is not correct. 13°/o chose this.
• 28
These findings describe hypoparathyroidism, which may occur congenitally (DiGeorge syndrome) or after
• 29 • ._k,u··-iA,..,-._,..._...,1 c,,_..._..__ _....- -."'"'- ..-- 1-.+--A ..__ h,~" __ ,.. .... J .... -_...i-.· ..__ .__,,..." , ...1-- ....-.-.-i-- ,.. _ _... __ .._; ..... -.-,.I,.._ ;_ ,,,.._,... err: ,.,ill

a
Lock
s
Suspend
8
End Bl ock
Item: 17 of 33 ~. I • M k <:] t> al ~· ~
QIO: 3920 .l. ar Previous Next lab 'lifllues Notes Calculator
- - -

9 Bone resorption Metabolism low back pain Bone disease Tobacco smoking
10
A is not correct. 13% chose this.
11
These findings describe hypoparathyroidism, which may occur congenital ly (DiGeorge syndrome) or after
12 thyroidectomy. Symptoms are related to hypocalcemia : tetany, depression, dementia, and seizures. ECG wi ll
13 show a prolonged QT interval, which pred isposes the patient to develop torsades de pointes. This patient does
14 not have symptoms of hypocalcemia.
Torsades de pointes Hypoparathyroidism Hypocalcaemia Thyroidectomy Tetany QT interval DiGeorge syndrome Electrocardiography Dementia Epileptic seizure
15
Depression (mood) long QT syndrome Major depressive disorder
16
17 C is not correct. 14% chose this.
18 These lab values are seen in Paget disease, also known as osteitis deformans. This is a disease of abnormal
bone architecture due to haphazard osteoblastic and osteoclastic activity . Symptoms include bone pain,
19
increased risk for bony fractures, hearing loss, and headaches. Patients are at increased risk for osteosarcoma
0 20 as we ll as high-output card iac failure from multiple arteriovenous shunts. This patient does not have symptoms
0
21 of Paget disease.
Osteosarcoma Paget' s disease of bone Bone pain Osteoblast Bone Heart failure Hearing loss
0
22
0
23 D is not correct. 10% chose this.
0 24 These findings describe hyperparathyroidism. Parathyroid hormone stimu lates bone resorption, calcium
0 25 reabsorption, and phospho rus excretion . Symptoms are re lated to hypercalcemia: osteopen ia, kidney stones,
po lyuria, constipation, abdomina l pa in, depression, and psychosis. ECG wi ll show a shortened QT interval. Th is
0
26
patient does not have symptoms of hyperca lcemia.
0 27 Hyperparathyroidism Parathyroid hormone Polyuria Hypercalcaemia Osteopenia QT interval Constipation Psychosis Parathyroid gland Kidney Bone resorption
0
28 Kidney stone Abdominal pain Phosphorus Electrocardiography Calcium Hormone Bone Excretion Depression (mood) Major depressive disorder

0 29 •

6
lock
s
Suspend
0
End Block
Item: 17 of 33 ~. I • M k <:] t> al ~· ~
QIO: 3920 .l. ar Previous Next lab 'lifllues Notes Calculator
- ---- - -- - - - -- -- -- -- - -- - -- -- --
9 not have symptoms of hypocalcemia.
10 Torsades de pointes Hypoparathyroidism Hypocalcaemia Thyroidectomy Tetany QT interval DiGeorge syndrome Electrocardiography Dementia Epileptic seizure

11 Depression (mood) long QT syndrome Major depressive disorder

12 C is not correct. 14% chose this.


13 These lab values are seen in Paget disease, also known as osteitis deformans. This is a disease of abnormal
14 bone architecture due to haphazard osteoblastic and osteoclastic activity . Symptoms include bone pain,
increased risk for bony fractures, hearing loss, and headaches. Patients are at increased risk for osteosa rcoma
15
as we ll as high-output ca rdiac failure from multiple arteriovenous shunts. This patient does not have symptoms
16 of Paget disease.
17 Osteosarcoma Paget' s disease of bone Bone pain Osteoblast Bone Heart failure Hearing loss

18 D is not correct. 10% chose this.


19 These findings describe hyperparathyroidism. Parathyroid hormone stimu lates bone resorption, calcium
0 20 reabsorption, and phosphorus excretion . Symptoms are re lated to hypercalcemia: osteopen ia, kidney stones,
0
21 po lyuria, constipation, abdomina l pa in, depression, and psychosis. ECG wi ll show a shortened QT interval. Th is
patient does not have symptoms of hyperca lcemia.
0
22
Hyperparathyroidism Parathyroid hormone Polyuria Hypercalcaemia Osteopenia QT interval Constipation Psychosis Parathyroid gland Kidney Bone resorption
0
23
Kidney stone Abdominal pain Phosphorus Electrocardiography Calcium Hormone Bone Excretion Depression (mood) Major depressive disorder
0 24
0 25
0
26 Bottom Line:
0 27 Osteoporosis is associated with norma l calcium, phosphorus, and alka line phosphatase levels.
Osteoporosis Alkaline phosphatase Phosphorus Calcium Alkalinity Phosphatase
0
28
0 29 •

6
lock
s
Suspend
0
End Block
Item: 17 of 33 ~ 1 • M k -<:J 1>- Jil ~· !:';-~
QIO: 3920 ..L ar Pre v ious Next Lab fli!ltues Not es Calcula t o r
A A

9
10 FA17 p 435.2

11 Osteoporosis Trabecular {spongy) and cortical bone lose mass Can lead to vertebral compression fractures (fJ,
12 NOITTloll Normal and interconnections despite normal bone small arrows; large arrows show normal-for-age
mte!Veltebral lllte!Veltebral mineralization and lab values (serum Ca 2+ and ,·ertebral body height for comparison)-acute
13 dose ~ forMnen
P0_.3- ). back pain, loss of height, kyphosis. lso can
14
l\lost commonly due to t bone resorption present with fractures of femoral neck, distal
15 related to l estrogen le\·els and old age. radius (Colles fracture).
16 ~ Can be secondary to drugs (eg, steroids,
17 alcohol, anticom ulsants, anticoagulants,
18 thyroid replacement therapy) or other
Central expanwn Restncted
of inteMrtebral lllte!Veltebral medical conditions (eg, hyperparathyroidism,
19 dose ,.. v/ foramen hyperthyroidism, multiple myeloma,
• 20
::-~fj~
malabsorption syndromes).
• 21
• 22
:~>-..... N Diagnosed by a bone minera l density sca n (dual-
energy x-ray absorptiometry fOF.XA]) with a
• 23 Mold compression friKture T-score of s; -2.5 or by a fragilit y fr<~cture of
hip or ,·ertebra. Screening recommended in
• 24
women> 65 years old.
• 25
Prophylaxis: regular weight-bearing exercise
• 26 and adequate Ca 2+ and vitamin D intake
• 27 throughout adu lthood .
• 28 Treatment: bisphosphonatcs, tcriparatide,
SER.\-1s, rarely calcitonin; denosumab
• 29 • . .. . . ..... ..... ' . ,. .
a
Lock
s
Suspend
8
End Bl ock
Item: 17 of 33 ~. I • M k <:] t> al ~· ~
QIO: 3920 .l. ar Previous Next lab 'lifllues Notes Calculator

• •
9
FA17 p437.1
10
Lab values in bone disorders
11
DISORDER SERUMCa 2+ POl- ALP PTH COMMENTS
12
Osteoporosis l bone mass
13
Osteopetrosis - !l Dense, brittle bones. Ca 2+ l in severe,
14 malignant disease
15
Paget disease of bone t Abnormal "mosa ic" bone architecture
16
Osteitis fibrosa cystica "Brown tumors" due to fibrous replacement of
17 bone, subperiosteal thinning
18 Primary t t t Id iopathic or parathyroid hyperplasia, adenoma,
19 hyperparathyroidism carcmoma
0 20 Secondary f t f Often as compensation for C KD (l P0 4 3-
0
21 hyperparathyroidism excretion and production of activated
vitamin D)
0
22
0
23
Osteomalacia/rickets t f Soft bones; vitamin D deficiency also causes zo
hyperparathyroidism
0 24
0 25
Hypervitaminosis D t f Caused by oversupplementation or
granu lomatous disease (eg, sarcoidosis)
• 26
0 27
• 28
29 FA17 p439.1 •
0

6
lock
s
Suspend
0
End Block
Item: 17 of 33 ~ 1 • M k -<:J 1>- Jil ~· !:';-~
QIO: 3920 ..L ar Pre v ious Next Lab fli!ltues Not es Calcula t o r
A A

9 FA17 p439.1

10 Osteoarthritis a nd rheumatoid a rthritis


11 Osteoarthritis Rheumatoid arthritis
12 PATHOGENESIS \lechanical-wear and tear destroys articular Autoimmune-inflammation induces formation
13 cartilage (degenerati' e joint disorder) of pannus {proliferati,·e granulation tissue rJ).
-+ inflammation with inadequate repair. "hich erodes articular cartilage and bone.
14
Chondrocytes mediate degradation and
15 inadequate repair.
16
PREDISPOSINGFACTORS Age, female, obesity, joint trauma. Female, HLA-DRt, smoking, silica exposure.
17 ® rheumatoid factor (lgM antibody that
18 targets lgG Fe region; in 80%), anti-cyclic
19 citrullinated peptide antibody (more specific).
• 20 PRESENTATION Pain in weight-bearing joints after usc (cg, Pain, S\\'clling, and morning stiffness lasting
• 21 at the end of the day), improving with rest . > l hour, impro\'ing with use. Symmetric
Asymmetric joint involvement. Knee carti lage joint involvement. Systemic symptoms
• 22
loss begins media lly ("bowlegged"). o (rever, fatigue, weight loss). Extraarticular
• 23 manifestations common.~
systemic symptoms.
• 24
JOINTFINDINGS Osleophyles (bone spurs), joint space narrowing, Erosions, juxta-articular osteopen ia, sofll issue
• 25 subchondral sclerosis and cysts. Synovial swelling, subchondral cysts, joint space
• 26 Auid non-inAammatory (WBC < 2000/mm3). narrowing. Deformities: cervical subluxation,
• 27 Im·olves DlP (Heberden nodes : ) and PIP ulnar finger deviation, swan neck rn.
• 28
(Bouchard nodes B), and 1st C MC; not .MCP. boutonniere 0 . ln\'Oives ~ICP, PIP, wrist; not
DIP or lst C~ IC. Synovial Auid inAammatorr
• 29 •
a
Lock
s
Suspend
8
End Bl ock
Item: 17 of 33 ~ 1 • M k -<:J 1>- Jil ~· !:';-~
QIO: 3920 ..L ar Pre v ious Next Lab fli!ltues Not es Calcula t o r

! - • t
9
*Extraarticular manifestations include rheumatoid nodules (fibrinoid necrosis with palisad ing histiocytes) in subcutaneous
10
tissue and lung (+ pneumoconiosis - Caplan syndrome), interstitial lung disease, pleuritis, pericarditis, anemia of chronic
11 disease, neutropenia + splenomegaly (Felty syndrome), AA amyloidosis, Sjogren syndrome, scleritis, carpal tunnel syndrome.
12
Normal Osteoarthritis Normal Rheumatoid
13 arthritis
- - - - -ThiCkened
capsule
~~~
/
14
15 / Slight syi10Y1al eros1on
Joint capsule """'\. Joint capsule """'\.
and S)'IIOVIclt \, / _hypertrophy and S)'llOVIcll \,
16 - - -Osteophyte
llning IIrung
_ _ - Increased
17 Synovial . . / - - - Ulcerated S)'IIOVIcll _ / S)'llOVIcll RUid
~ cart1lage CaVIty /
18 cavity/ _..-- Pannus
Cartilage ~ ~ SclerOilc bone Cartilage. / forma~on
19 Jomt space
narrowing
• 20
Subchondral
• 21 bone cyst

• 22
• 23
• 24
• 25
• 26
• 27
• 28
• 29 •
a
Lock
s
Suspend
8
End Bl ock
Item: 20 of 33 ~ 1 • M k -<:J 1>- Jil ~· !:';-~
QIO: 4512 ..L ar Pre v ious Next Lab fli!ltues Not es Calcula t o r

IAA]
A A

9
A 60-year-old man presents to the physician complaining of a "strange rash" that began with it ching but has
10 worsened over t he past 2 weeks to create the fluid-filled blisters seen in the image. Physical examination
11 revea ls sim ilar blisters on many areas of his skin . Histopatholog ic examination of one of the lesions
demonstrates linear immunofluorescence at the dermal-epidermal junction.
12
13
14
15
16
17
18
19
• 20
• 21
• 22
• 23
• 24
• 25
• 26
• 27
Wh ich of the fol lowing cellular structures is most likely affected by th is patient's cond ition?
• 28
:
• 29 •
a
Lock
s
Suspend
8
End Bl ock
9
10
11
12
13
14
15
16
17
18
19
• 20 Which of the following cellular structures is most likely affected by this patient's condit ion?
• 21
:
• 22 A. Connexons
• 23
B. Desmosomes
• 24
• 25 C. Hemidesmosomes
• 26 D. Nuclear lam ina
• 27
E. Tig ht junctions
• 28
• 29 •
a
Lock
s
Suspend
8
End Block
Item: 20 of 33 ~. I • M k <:] t> al ~· ~
QIO: 4512 .l. ar Previous Next lab 'lifllues Notes Calculator


9
The correct answer is C. 71°/o chose this.
10
The diso rde r describes bul lous pemphigoid (BP) . The presentation of BP may vary between patients, but the
11
elde rly are most affli cted and report the classic bullae depicted in the image . BP is caused by an autoimmune
12 attack against hemidesmosomes, which are junctional complexes t hat anchor the basal layer of the epider mis to
13 the unde rlying basement membrane. Loss of normal hemidesmosome function leads to bu llae fo rmation
14
because the epidermis is lifted from the dermis as a continuous sheet.
Hemidesmosome Bullous pemphigoid Basement membrane Dermis Epidermis Stratum basale Blister Cutaneous condition Autoimmune disease Autoimmunity
15
Epidermis (zoology) Cell junction
16
17 A is not correct. 3°/o chose this.
18
Connexons are hexa mers of connexin proteins t hat fo rm gap junctions between adjacent cel ls. Gap junctions
are crucial fo r certain types of cell-cell signa ling, but are not affected in BP.
19 Connexin Gap junction Connexon Protein Cell signaling
20
B is not correct. 20% chose this .
• 21
Desmosomes, like he midesmosomes, are important ce ll ad hesion complexes. Un like hemidesmoso mes,
• 22
however, desmosomes mediate cell-cell rather than cell-base ment membrane adhesion . Autoim mune attack
• 23 against desmoso mes results in pe mphigus vu lgaris, a vesicu lobul lous diso rder cha racte rized by potent ially fata l
• 24 acantholysis .
Acantholysis Autoimmune disease Pemphigus Pemphigus vulgaris Desmosome Hemidesmosome Autoimmunity Cell adhesion
• 25
• 26 D is not correct. 3°/o chose this .
• 27 Nuclear lamina are special ized intermediate fila ment cytoske letal proteins that are important in maintaining
norma l nuclear structu re. Lamina are not involved in cell-cell adhesion and are unaffected by BP .
• 28
Intermediate filament Nuclear lamina Cytoskeleton Protein leaf
• 29 •

6
lock
s
Suspend
0
End Block
Item: 20 of 33 ~. I • M k <:] t> al ~· ~
QIO: 4512 .l. ar Previous Next lab 'lifllues Notes Calculator

• Connexin Gap junction Connexon Protein Cell signaling


9
10 B is not correct. 20% chose this.
11 Desmosomes, like hemidesmosomes, are important ce ll adhesion complexes. Un like hemidesmosomes,
12 however, desmosomes mediate cell-cell rather than cell-basement membrane adhesion . Autoimmune attack
against desmosomes results in pemphigus vu lgaris, a vesicu lobul lous disorder characterized by potentially fata l
13
acantholysis.
14 Acantholysis Autoimmune disease Pemphigus Pemphigus vulgaris Desmosome Hemidesmosome Autoimmunity Cell adhesion
15
D is not correct. 3°/o chose this.
16
Nuclear lamina are special ized intermediate filament cytoske letal proteins that are important in maintaining
17 norma l nuclear structure. Lamina are not involved in cell-cell adhesion and are unaffected by BP.
18 Intermediate filament Nuclear lamina Cytoskeleton Protein leaf

19
E is not correct. 3°/o chose this.
20 Tight junctions form impermeable barriers between epithelial ce lls to prevent fluid and so lutes from passing
• 21 between the joined ce lls. In add it ion, these ju nctions separate epithel ial cells into segregated apical and
• 22 basolateral plasma membrane divisions, allowing specific functions to occur at each site. Tight junctions are not
affected in BP .
• 23
Tight junction Cell membrane Blood plasma Epithelium Solution Biological membrane
• 24
• 25
• 26 Bottom Line:
• 27 Bu llous pemph igoid is caused by autoimmune attack aga inst hemidesmosomes .
Bullous pemphigoid Hemidesmosome Autoimmune disease Autoimmunity Cutaneous condition Pemphigoid
• 28
• 29 •

6
lock
s
Suspend
0
End Block
Item: 20 of 33 ~ 1 • M k -<:J 1>- Jil ~· !:';-~
QIO: 4512 ..L ar Pre v ious Next Lab fli!ltues Not es Calcula t o r
A A

9
FA17 p452.1
10
Bliste ring skin disorders
11
Pemphigus vulgaris Potentially fatal autoimmune skin disorder with lgC antibody against desmoglein (component of
12
desmosomes, which connect keratinocytes in the stratum spinosum).
13
F'laccid intraepidermal bullae caused b) acantholysis (separation ofkeratinocytes, resembling a
14 "ro\\ of tombstones"); oral mucosa is also imolved. T) pe II h~ persensitivity reaction.
15 Immunofluorescence rc,·cals antibodies around epidermal cells in a reticular (net-like) pattern
16 ' ikolskr sign <B (separation of epidermis upon manual stroking of skin).
17 Bullous pemphigoid Less severe than pemphigus vulgaris. Involves lgC antibody against hemidesmosomes (epidermal
18
basement membrane; antibodies are " bullow" the epidermis).
Tense blisters contain ing eosinophils affect skin but spa re oral mucosa.
19
lmmunoAuorcscence reveals linear pattern at epidermal-dermal junction [!J.
20 1\:i kolsky sign e.
• 21
Dermatitis Pruritic papules, vesicles, and bullae (often found on elbows) D. Deposits of Ig at tips of dermal
• 22 herpetiform is papillae. Associated with celi<ICdisease. Treatment: dapsone, gluten-free diet.
• 23 Erythema multiforme Associated with infections (eg, Mycoplasma pneumoniae, 1-1$\1}, drugs (eg, sulfa drugs, ~-l actams,
• 24 phenytoin), cancers, autoimmune disease. Presents with multiple types of lesions-maeules,
• 25 papules, vesicles, target lesions (look like targets with multiple rings and dusky center showing
• 26
epithelial disruption)
• 27 Stevens-Johnson Characterized by fever, bullae formation and necrosis, sloughing of skin at dermal-epidermal
syndrome junction, high mortality rate. Typically 2 mucous membranes are im·olved [!I l:l. and targetoid
• 28
skin lesions may appear, as seen in erythema multifonne. Usually associated with ad\'erse drug
• 29 • ron,...~,·u·, .\. ..-nnro rt:u•o r o fnrn'\ n( ~lt:H nn,.. l n l,nrnn " ' n rlrnn, o fC:: IC::' ,.,~.h ......._ :tnoz. .nf •1,-n l..._n,)u c ~trf.,,...n

Lock
a s
Suspend
8
End Bl ock
Item: 20 of 33 ~ 1 • M k -<:J 1>- Jil ~· !:';-~
QIO: 4512 ..L ar Pre v ious Next Lab fli!ltues Not es Calcula t o r
A A

9 Erythema multiforme Associated with infections (cg, fvlycofJ/asma pneumoniae, HSV), drugs (cg, sulfa drugs. ~-l ac tams,
10 phenytoin), cancers, autoimmune disea~e. Presents with multiple types of lesions-maculcs,
11 papules, \'esicles. target· lesions (look like targets with multiple rings and dusky center showing
12
epithelial disruption)

13 Stevens-Johnson Characterized by fever, bullae formation and necrosis, sloughing of skin at dermal-epidermal
syndrome junction, high mortality rate. Typically 2 mucous membranes are im·olved @ • and targctoid
14
skin lesions may appear, as seen in er} thema multi forme. Usually associated with acherse drug
15
reaction. A more severe form of Ste' ens· Johnson S} ndrome (SJS) with > 30% of the body surface
16 area inml"ed is toxic epidermal necrolysis (TEN). 10-30% im·okement denotes SJS-TE '.
17
18
19
20
• 21
• 22
• 23
• 24
• 25
• 26
• 27
• 28
• 29 •
a
Lock
s
Suspend
8
End Bl ock
Item: 20 of 33 ~ 1 • M k -<:J 1>- Jil ~· !:';-~
QIO: 4512 ..L ar Pre v ious Next Lab fli!ltues Not es Calcula t o r
A A

9
FA17 p 447.1
10
11
Epithelial cell junctions

12
Apical
13
. )>---- Tight junction (zonula occludens)-prevents paracellular
14 E-c.adhenn ~ movement of solutes. composed of claudins and occlud1ns.
15
16
Actin /
filaments ..../
: e }- Adhtrtns junction (belt desmosome. zonula adherensl-below
light JUOCIIOO. forms 'belt" connecting actin cytoskeletons of
adjacent cells with CADhenns (u 1 ' ·dependent adhesion
17

L prote1ns1. Loss of E-cadherin promotes metasta.sis.
Cytokeratin -
18
Desmoptakin ~ •
.f'--.- Dts11oso111t (spot desmosome. macula adherens)- structural
19 support via mlermed~ate filament interactions. Autoantibodies
- pemphigus vulgaris

~p
20
• 21 Connexon ----<( ~ }--_ junction- channel proteins called connexons permit
with central electrical and chemical communication between cells.
• 22 channel
• 23 ~ Cell membrane
• 24 Basolaleral iiiiiiW~~;;;:--;.-;;;__ Basement membrane
• 25 lntegrins-membrane proteins that maintain =../ \_ Htllidesmosome-connects keratin in basal cells to
Integrity of basotateral membrane by binding underly1ng basement membrane. Autoantibodies - bullous
• 26 to collagen and lam1nin in basement membrane. pemphtgoid. (Hem1desmosomes are down "bullowl
• 27
• 28 FA17 p 446.2
• 29 •
a
Lock
s
Suspend
8
End Bl ock
Item: 20 of 33 ~ 1 • M k -<:J 1>- Jil ~· !:';-~
QIO: 4512 ..L ar Pre v ious Next Lab fli!ltues Not es Calcula t o r
A A

9
Skin layers Skin has 3 layers: epidermis, dermis, C alifornians Like G irls in String Bikinis.
10 subcutaneous fat (hypodermis, subculis).
11 Epidermis layers from surface to base · ·
12 Stratum Corneum (keratin)
13 • Stratum Lucidum
• Stratum Granulosum
14
~ Stratum Spinosum (desmosomes)
15
Stratum Basale (stem cell site)
16
17
18
19
20
• 21
• 22
• 23
~

• 24
• • •.. w

• 25 , 4 ,
• 26
~ .

• .......
" • - ~
• 27
• 28
• .. '"
• 29 •
a
Lock
s
Suspend
8
End Bl ock
Item: 21 of 33 ~. I • M k <:] t> al ~· ~
QIO: 5010 .l. ar Previous Next lab 'lifllues Notes Calculator


9
A 17-year-old male with an uncomp licated past medical history presents to his ped iatrician with pain in his
10 right leg. The pain has been present for several months now and is loca lized above the right knee. The pain
11 waxes and wanes but sometimes worsens at night. He denies any trauma, and anti-inflammatory
medications have failed to provide long-term rel ief. He lacks any systemic signs such as mala ise or fatigue. Of
12
note, his family history is relevant for what appears to be some sort of ocu lar cancer in his younger sister.
13 Physical examination is remarkable for a well-appearing young male with a firm soft-tissue mass superior to the
14 right knee that is tender to pa lpation . A subsequent biopsy reveals a mal ignant bone tumor of the dista l femur.
15
16 Wh ich of the fo llowing is the most like ly diagnosis?
17 :
18 A. Chondrosarcoma
19 B. Enchondroma
20
C. Ewing sarcoma
• 21
• 22 D. Osteochondroma
• 23 E. Osteosarcoma
• 24
• 25
• 26
• 27
• 28
• 29 •

6
lock
s
Suspend
0
End Block
Item: 21 of 33 ~. I • M k <:] t> al ~· ~
QIO: 5010 .l. ar Previous Next lab 'lifllues Notes Calculator


9
The correct answer is E. 62°/o chose this.
10
Osteosarcoma is the most common nonhematopoietic bone tumor. These tumors are seen predominantly in
11 ma les <20 years old and occur at the metaphysea l region of long bones. Plain fi lms of the affected bone often
12 reveal a characteristic "sunburst" pattern. Genetic mutations of the Rb gene are associated with both
13 osteosarcoma and retinob lastoma, which was likely present in th is patient's sister.
Osteosarcoma Retinoblastoma Bone tumor Gene Neoplasm Metaphysis Retinoblastoma protein Bone long bone Mutation
14
15 A is not correct. 9°/o chose this.
16 Chondrosarcomas are ma lignant carti laginous tumors that occur most common ly in men 30-60 years old,
usua lly in the pelvis, spine, scapu la, humerus, tibia, or femur.
17
Femur Scapula Humerus Tibia Pelvis Cartilage Malignancy Chondrichthyes Cancer
18
B is not correct. 3°/o chose this.
19
Enchondromas are benign cartilaginous tumors in intramedu llary bone and most often in the distal extremities.
20
Cartilage Anatomical terms of location Medullary cavity Chondrichthyes Neoplasm Benign tumor Bone
21
C is not correct. 17% chose this .
• 22
Ewing sarcoma is the second most common primary mal ignant bone tumor. Most patients present between the
• 23
ages of 10 and 15 years. Ewing sarcoma most commonly occurs in the diaphyses of long bones, pelvis, scapu la,
• 24 and ribs.
• 25 Diaphysis Scapula Bone tumor Ewing' s sarcoma Pelvis Sarcoma Malignancy Neoplasm long bone Cancer Bone

• 26 D is not correct. 9°/o chose this.


• 27 Osteochond romas are ben ign growths that are often first diagnosed in late adolescence or ea rly adulthood. They
• 28 are the most common benign bone tumor.
Bone tumor Adenoma Neoplasm Benign tumor Bone Benignity
• 29 •

6
lock
s
Suspend
0
End Block
Item: 21 of 33 ~. I • M k <:] t> al ~· ~
QIO: 5010 .l. ar Previous Next lab 'lifllues Notes Calculator

• Osteosarcoma Retinoblastoma Bone tumor Gene Neoplasm Metaphysis Retinoblastoma protein Bone long bone Mutation
9
10 A is not correct. 9°/o chose this.
11 Chondrosarcomas are ma lignant cartilag inous t umors that occur most common ly in men 30- 60 years old,
usua lly in t he pelvis, spine, scapu la, humerus, t ibia, or femur.
12
Femur Scapula Humerus Tibia Pelvis Cartilage Malignancy Chondrichthyes Cancer
13
14 B is not correct. 3°/o chose this.
Enchond romas are benig n cartilaginous tumo rs in intramedu llary bone and most often in the dista l ext remit ies .
15
Cartilage Anatomical terms of location Medullary cavity Chondrichthyes Neoplasm Benign tumor Bone
16
17
C is not correct. 17% chose this.
Ewing sarcoma is t he second most common primary mal ignant bone tumor. Most patient s present between the
18
ages of 10 and 15 yea rs. Ewing sa rcoma most commonly occu rs in the diaphyses of long bones, pelv is, scapu la,
19 and ribs.
20 Diaphysis Scapula Bone tumor Ewing' s sarcoma Pelvis Sarcoma Malignancy Neoplasm long bone Cancer Bone

21
D is not correct. 9°/o chose this .
• 22 Osteochondromas are ben ign growths t hat are often first diagnosed in late adolescence or early adulthood . They
• 23 are the most common benign bone tumor.
Bone tumor Adenoma Neoplasm Benign tumor Bone Benignity
• 24
• 25
• 26 Bottom Line:
• 27
Genetic mut ations of the Rb gene are associated wit h both osteosarcoma and ret inoblastoma .
• 28 Osteosarcoma Retinoblastoma Gene Retinoblastoma protein Mutation

• 29 •

6
lock
s
Suspend
0
End Block
Item: 21 of 33 ~. I • M k <:] t> al ~· ~
QIO: 5010 .l. ar Previous Next lab 'lifllues Notes Calculator

9
10 FA17 p42.1

11 Cell cycle phases Checkpoints control transitions between phases of cell cycle. This process is regulated by cyclins,
12 cyclin-dependcnt kinases (CDKs), and tumor suppressors. M phase (shortest phase of cell cycle)
13
includes mitosis (prophase, prometaphase, metaphase, anaphase, telophase) and cytokinesis
(cytoplasm splits in h,·o). C 1 and C 0 arc of variable duration.
14
REGULATION OF CELL CYCLE
15
16
Cyclin-dependent Constitutive and inactive.
kinases
17
Cyclins Regulatory proteins that control cell cycle
18
events; phase specific; activate CDKs.
19
Cyclin-CDK complexes Phosphorylate other proteins to coordinate
20
cell cycle progression; must be activated and
21 inactivated at appropriate times for cell cycle
• 22 to progress .
• 23 Tumor suppressors p53 induces p21, which inhibits CDKs
• 24 - hypophosphorylation (activation) of Rb
• 25 - inhibition of C 1-S progression. Mutations
in tumor suppressor genes can result in Rb, p53 modulate
• 26
unrestrained cell division (eg, Li-Fraumeni G1restriction point
• 27
syndrome).
• 28 CELL TYPES
• 29 Permanent Remain in Cn. regenerate from stem cells. Neurons. skeletal and cardiac muscle. RBCs.
6
lock
s
Suspend
0
End Block
Item: 21 of 33 ~. I • M k <:] t> al ~· ~
QIO: 5010 .l. ar Previous Next lab 'lifllues Notes Calculator

9
Permanent Remain in G0, regenerate from stem cells. Neurons, skeletal and cardiac muscle, RBCs.
10
Stable (quiescent) Enter G 1 from G0 when stimulated. Hepatocytes, lymphocytes.
11
Labile 1ever
go to G0, divide rapidly with a short G 1• Bone marrow, gut epithelium, skin, hair fol licles,
12
Most affected by chemotherapy. germ cells.
13
14
FA17 p 218.2
15
Tumor suppressor Loss of function -+ f cancer risk; both (two) alleles of a tumor suppressor gene must be lost for
16 genes expression of disease.
17 GENE GENE PRODUCT ASSOCIATED CONDITION
18 APC 1\'cgative regulator of ~-catenin/WNT pathway Colorcctal cancer (associated with FAP)
19 BRCA 1/ BRCA2 D A repair protein Breast, ovarian, and pancreatic cancer
20 CDKN2A pl6, blocks c) -+ s phase lelanoma, pancreatic cancer
21 DCC DCC- Deleted in C olon Cancer Colon cancer
• 22
DPC4/SMAD4 DPC - Deleted in Pancreatic Cancer Pancreatic cancer
• 23
MEN1 Men in ME 1
• 24
NF1 eurofibromin (Ras GTPase activating protein) Neurofibromatosis type 1
• 25
NF2 Merlin (schwannomin} protein Neurofibromatosis type 2
• 26
PTEN Tyrosine phosphatase of PIP3 (eg, protein kinase Breast cancer, prostate cancer, endometrial
• 27
B [AKT] activation) cancer
• 28
Rb Inhibits E2F; blocks C 1 -+ S phase Retinoblastoma, osteosarcoma
• 29

6
lock
s
Suspend
0
End Block
Item: 21 of 33 ~ 1 • M k -<:J 1>- Jil ~· !:';-~
QIO: 5 0 10 ..L ar Pre v ious Next Lab fli!ltues Not es Calcula t o r
A
PTEN Tyrosine phosphatase of PI P3 (eg, protein kinase Breast cancer, prostate cancer, endometrial A

9
B [AKT] activation) cancer
10
Rb Inhibits E2F; blocks G 1 .... S phase Retinoblastoma. osteosarcoma
11
12
TPS3 p53, activates p2l, blocks cl .... s phase \lost human cancers, Li-Fraumeni S) ndrome
(multiple malignancies al early age, aka, SBL \
13
cancer srndrome: Sarcoma, Breast, Leukemia,
14 \ drenal gland}
15 TSC7 Hamartin protein Tuberous sclerosis
16 TSC2 Tuberin protein 'I uberous sclerosis
17
VHL Inhibits hrpoxia inducible factor Ia \ Oil llippei-L indau disease
18
WT1 Transcription factor that regulates mogenital W ilms tumor (nephroblastoma)
19
development
20
21 FA17 p 438.1
• 22 Primary bone tumors
• 23 TUMOR TYPE EPIDEMIOLOGY/LOCATION CHARACTERISTICS

• 24 Benign tumors
• 25 Osteochondroma Most common benign bone tumor. Bony exostosis with cartilaginous (chondroid)
• 26 l\lales < 25 years old. captl
Rarely transforms to chondrosarcoma .
• 27
Giant cell tumor 20-40 years old. Locally aggressive benign tumor.
• 28
Epiphysis of long bones (often in knee region). "Soap bubble" appearance on x-ray • .
• 29 • ~ 1.. h-:'"'''"'l.o.., l-o rl ,..;.., ....... ,..oil~ 1-J.....,1- avnr.o'""" D & ' l it I

a
Lock Suspend
s 8
End Bl ock
Item: 21 of 33 ~ 1 • M k -<:J 1>- Jil ~· !:';-~
QIO: 5 0 10 ..L ar Pre v ious Next Lab fli!ltues Not es Calcula t o r

9
•• g

10
Osteochondroma l\1ost common benign bone tumor. Bony exostosis with cartilaginous (chondroid)
Males< 25 years old. cap · .
11
Rarely transforms to chondrosarcoma.
12
Giant cell tumor 20-40 years old. Locally aggressive benign tumor.
13 Epiphysis of long bones (often in knee region). "Soap bubble" appearance on x-rar • .
14 rises most commonly at distal femur and \ lultinucleated giant cells that express RA K1••
15 proximal tibia.
16 "Osteoclastoma."
17 Malignant tumors

18 Ost eosarcoma One of the most common malignant bone Cod man triangle {from elevation of periosteum)
(osteogenic sarcoma) tumors. or sunburst pattern on x-ray.
19
Bimodal distribution: 10-20 years old {I ), > 65 Aggressive. Treat with surgical en bloc resection
20
(20). (with limb sakage) and chemotherapy.
21 Predisposing fa ctors: Paget disease of bone, bone
• 22 infarcts, radiation, fam ilial retinoblastoma,
• 23 Li-Fraumeni syndrome (gcrmline p53
• 24 mutation).
Metaphysis of long bones, often around knee ~.
• 25
Ewing sarcoma Boys< 15 years old. Anaplastic small blue cellma lignant t11mor [!].
. 26
Commonly appears in diaphysis of long bones, Extremely aggressive with early metastases, b11t
. 27
pelvis, scapula, ribs. responsi' e to chemotherapy.
. 28 "Onion skin" periosteal reaction in bone.
. 29 • Associated with t(ll ;22) translocation causing

a
Lock
s
Suspend
8
End Bl ock
Item: 21 of 33 ~ 1 • M k -<:J 1>- Jil ~· !:';-~
QIO: 5 0 10 ..L ar Pre v ious Next Lab fli!ltues Not es Calcula t o r
A A

9 Commonly appears in diaphysis of long bones, Extremely aggressive with ea rly metastases, but
10 pelvis, scapula, ribs. responsive to chemotherapy.
11
"Onion skin" periosteal reaction in bone.
Associated with t(ll ;22) translocation causing
12
fusion protein EWS-FLI I.
13 11 + 22 = 33 (Patrick Ewing's jersey number).
14 ~-----;.;;~~~-

15
16
17 Round cell lesions -;..,__.
Ewing sarcoma
18 Myeloma
~ Fibrous dysplasia
19
20 Osteoid osteoma
lr !f'!U r» f" c 'fill 1'10
21
• 22 Simple bone cyst

• 23
• 24
ttsteosarcoma Osteochondroma
• 25 J!
~
. 26 Physis

. 27 t{
·a.
w
G~ant cell tumor
It]
. 28
. 29 •
a
Lock
s
Suspend
8
End Bl ock
Item: 22 of 33 ~. I • M k <:] t> al ~· ~
QIO: 5001 .l. ar Previous Next lab 'lifllues Notes Calculator


9
A mother brings her 5-year-o ld son to his ped iatrician's office. The boy has been experiencing frequent fal ls ~~AI
10 and has progressive difficulty walking, jumping, and hopping . Laboratory testing revea ls a creatine kinase
11 level of 2840 U/L. Muscle biopsy revea ls variation in fiber diameter, an increased number of interna lized
nuclei, and muscle fiber degeneration and regeneration, and Western blot ana lysis confirms the diagnosis.
12
13
Wh ich of the fo llowing diseases demonstrates the same inheritance pattern as the cond it ion described above?
14
15 :
A. Cystic fibrosis
16
17 B. Glucose-6-phosphate dehydrogenase deficiency
18 C. Hemochromatosis
19
D. Hereditary spherocytosis
20
E. Huntington disease
21
• 22
• 23
• 24
• 25
• 26
• 27
• 28
• 29 •

6
lock
s
Suspend
0
End Block
Item: 22 of 33 ~. I • M k <:] t> al ~· ~
QIO: 5001 .l. ar Previous Next lab 'lifllues Notes Calculator


9
The correct answer is B. 50°/o chose this.
10
Duchenne muscu lar dystrophy (DMD) is the most common form of muscu lar
11
dystrophy, affecting 1 in 3500 live male births. Symptoms become evident by
12 the age of 5 yea rs, and include delayed wa lking, clumsiness, weakness in the
13 pe lvic girdle muscles, and en largement of the calf muscles (termed
pseudohypertrophy-shown in the image) . Laboratory testing reveals an
14
elevated creatine kinase level. DMD is caused by an abnorma lity of Xp21, the
15 region that codes for dystroph in, a myocyte-anchor ing protein. It is an X-
16 linked recessive disorder, as is glucose-6-phosphate dehydrogenase
17 deficiency.
Dystrophin Creatine kinase Glucose-6-phosphate dehydrogenase deficiency Muscular dystrophy
18
X-linked recessive inheritance Protein Pelvis Sex linkage Recessive Creatine X chromosome
19 Image copyright © 2014
Dominance (genetics) Glucose-6-phosphate dehydrogenase Senanayake eta!.; licensee
20
BioMed Central Ltd.
21
22 A is not correct. 12% chose this.
• 23 Cystic f ibrosis is an autosomal-recessive disease resulting from an inherited mutation in the CFTR chloride
• 24 channel. It is associated with symptoms such as difficulty breathing, frequent pneumon ias, and pancreatic
• 25
insufficiency .
Cystic fibrosis Chloride channel Cystic fibrosis transmembrane conductance regulator Mutation Exocrine pancreatic insufficiency Fibrosis
• 26
• 27 C is not correct. 11% chose this .
Hemochromatosis is an autosomal-recessive condition resulting in excessive iron absorption from the digestive
• 28
tract. This resu lts in micronodu lar cirrhosis, pancreatic fibrosis, and bronze skin pigmentation .
• 29 • Tron ovP.rlo~c1 C'irrhoc:ic: C::::lc:trointP.c:tin::ll tr~rt PiomP.nt Hum;:ln c:kin color Fihroc:ic: Tron

6
lock
s
Suspend
0
End Block
Item: 22 of 33 ~. I • M k <:] t> al ~· ~
QIO: 5001 .l. ar Previous Next lab 'lifllues Notes Calculator


9
The correct answer is B. 50°/o chose this.
10
Duchenne muscu lar dystrophy (DMD) is the most common form of muscu lar
11
dystrophy, affecting 1 in 3500 live male births. Symptoms become evident by
12 the age of 5 yea rs, and include delayed wa lking, clumsiness, weakness in the
13 pe lvic girdle muscles, and en largement of the calf muscles (termed
pseudohypertrophy-shown in the image) . Laboratory testing reveals an
14
elevated creatine kinase level. DMD is caused by an abnorma lity of Xp21, the
15 region that codes for dystroph in, a myocyte-anchor ing protein. It is an X-
16 linked recessive disorder, as is glucose-6-phosphate dehydrogenase
17 deficiency.
Dystrophin Creatine kinase Glucose-6-phosphate dehydrogenase deficiency Muscular dystrophy
18
X-linked recessive inheritance Protein Pelvis Sex linkage Recessive Creatine X chromosome
19 Image copyright © 2014
Dominance (genetics) Glucose-6-phosphate dehydrogenase Senanayake eta!.; licensee
20
BioMed Central Ltd.
21
22 A is not correct. 12% chose this.
• 23 Cystic f ibrosis is an autosomal-recessive disease resulting from an inherited mutation in the CFTR chloride
• 24 channel. It is associated with symptoms such as difficulty breathing, frequent pneumon ias, and pancreatic
• 25
insufficiency .
Cystic fibrosis Chloride channel Cystic fibrosis transmembrane conductance regulator Mutation Exocrine pancreatic insufficiency Fibrosis
• 26
• 27 C is not correct. 11% chose this .
Hemochromatosis is an autosomal-recessive condition resulting in excessive iron absorption from the digestive
• 28
tract. This resu lts in micronodu lar cirrhosis, pancreatic fibrosis, and bronze skin pigmentation .
• 29 • Tron ovP.rlo~c1 C'irrhoc:ic: C::::lc:trointP.c:tin::ll tr~rt PiomP.nt Hum;:ln c:kin color Fihroc:ic: Tron

6
lock
s
Suspend
0
End Block
Item: 22 of 33 ~. I • M k <:] t> al ~· ~
QIO: 5001 .l. ar Previous Next lab 'lifllues Notes Calculator
y
9
Cystic fibrosis Chloride channel Cystic fibrosis transmembrane conductance regulator Mutation Exocrine pancreatic insufficiency Fibrosis
10
C is not correct. 11% chose this.
11
Hemochromatosis is an autosomal-recessive condition resulting in excessive iron absorption from t he digestive
12
tract. This resu lts in micronodu lar cirrhosis, pancreatic fibrosis, and bronze skin pigmentation .
13 Iron overload Cirrhosis Gastrointestinal tract Pigment Human skin color Fibrosis Iron

14
D is not correct. 10% chose this.
15
Hereditary spherocytosis is an autosomal-dominant cond it ion resu lting from mutation of the eryth rocyte
16 proteins spectrin or ankyrin. This no rmocytic anemia results from unstable red blood cell membranes, in which
17 eryth rocytes are sma ll and round and without central pa llor.
Red blood cell Hereditary spherocytosis Spectrin Spherocytosis Anemia Dominance (genetics) Ankyrin Genetic disorder Pallor Mutation Cell membrane
18
Normocytic anemia Protein
19
20 E is not correct. 17% chose this.
21 Huntington disease is an autosoma l-dominant disease caused by mutation associated with t rinucleotide repeat
22
expansion .
Trinucleotide repeat expansion Trinucleotide repeat disorder Huntington' s disease Dominance (genetics) Genetic disorder Mutation
• 23
• 24
• 25 Bottom Line:
• 26 Both Duchenne muscu lar dystrophy ( DMD) and g lucose-6-phosphate dehydrogenase (G6PD ) deficiency are
• 27 inhe rited in an X-linked recessive pattern .
Duchenne muscular dystrophy X-linked recessive inheritance Glucose-6-phosphate dehydrogenase Muscular dystrophy
• 28 Glucose-6-phosphate dehydrogenase deficiency Sex linkage Glucose 6-phosphate Recessive
• 29 •

6
lock
s
Suspend
0
End Block
Item: 22 of 33 ~ 1 • M k -<:J 1>- Jil ~· !:';-~
QIO: 5 001 ..L ar Pre v ious Next Lab fli!ltues Not es Calcula t o r
A A

9
FA17 p 57.1
10
Muscular dystrophies
11
Duchenne X-linked disorder lypicall) due to framcshift Duchenne = deleted dyslrophin.
12
or nonsense mutations - truncated or Dystrophin gene (DMD) is the largest
13 absent dystrophin protein - progressive protein-coding human gene - f chance of
14 myofiber damage. Weakness begins in pcl"ic spontaneous mutation. Dystrophin helps
15 girdle muscles and progresses superiorly. anchor muscle fibers, primarily in skeletal and
16 Pseudohypertrophy of calf muscles due to cardiac muscle. It connects the intracellular
fibrofatty replacement of muscle . Waddling crtoskeleton (actin) to the transmembrane
17
gait. Onset before 5 years of age. Dilated proteins a- and ~-dystroglycan, which are
18 D
cardiomyopathy is common cause of death . connected to the extracellular matrix (ECr.. l).
19 Loss of dystrophin results in myonecrosis.
20 f CK and aldolase are seen; genetic testing
21 confirms diagnosis.
22 Becker X-linked disorder typically due to non- Deletions can cause both Duchennc and
• 23
frameshift deletions in dystrophin gene Becker muscular dystrophies. Y, of cases have
{partially functional instead of tnmc:1tcd). Less large deletions spanning one or more exons.
• 24
severe than Duchcnne. Onset in adolescence
• 25
or early adulthood .
• 26
Myotonic type 1 Autosomal dominant. CTG trinucleotide repeal C ataracts, Toupee (early balding in men),
• 27 expansion in the DMPK gene - abnormal Gonadal atrophy.
• 28 expression of myotonin protein kinase
• 29 -+ myotonia, muscle wasting, cataracts,

a
Lock
s
Suspend
8
End Bl ock
Item: 22 of 33 ~ 1 • M k -<:J 1>- Jil ~· !:';-~
QIO: 5 001 ..L ar Pre v ious Next Lab fli!ltues Not es Calcula t o r
A A

9 t CK and aldolase are seen; genetic testing


10 confirms diagnosis.

11 Becker X-I inked disorder typically due to non- Deletions can cause both Duchennc and
&ameshift deletions in d}'strophin gene Becker muscular dystrophies. Y, of cases ha\'e
12
(partially functional instead of truncated). Less large deletions spanning one or more exons.
13 se,·ere than Duchenne. Onset in adolescence
14 or earh- adulthood.
'
15 Myotonic type 1 Autosomal dominant. C T C trinucleotide repeat C ataracts, Toupee (early balding in men),
16 expansion in the DMPK gene - abnormal G onadal atrophy.
17 ex-pression of mrotonin protein kinase
- myotonia, muscle wasting, catamcts,
18
testicular atrophy, frontal balding, arrhythmia.
19
Gower sign-patient uses upper extremities to
20
help stand up.
21 Classically seen in Duchenne muscular
22 dystroph y, but also seen in other muscular
• 23 dystrophies and inAammatory myopathies (eg,
• 24 polymyositis). Lordosis-........

• 25
• 26
• 27
• 28
• 29 • FA17 D 55.1

a
Lock
s
Suspend
8
End Bl ock
Item: 22 of 33 ~ 1 • M k -<:J 1>- Jil ~· !:';-~
QIO: 5 001 ..L ar Pre v ious Next Lab fli!ltues Not es Calcula t o r
A A

9
10 FA17 p 55.1
Modes of inheritance
11
12
Autosomal dominant Often due to defects in structural genes. ~I any Often pleiotropic (multiple apparent!) unrelated
generations, both males and females are effects) and variably expressive (different
13
affected. between indi,•iduals). Family history crucial
14 to diagnosis. With one affected (heteroz} gous)
15 parent, on average, !lz of children affected.
16
17
18
Autosomal recessive Often due to enzyme deficiencies. Usuall) seen Commonlv more se,·ere than dominant disorders;
'
19 in only I generation. patients often present in ch ildhood.
20 f risk in consanguineous families.
21 With 2 carrier (heterozygous) parents, on average:
22
!4 of ch ildren will be affected (homozygous),
!lz of children will be carriers, and \4 of children
• 23
will be neither affected nor carriers .
• 24
• 25 X-linked recessive Sons of heterozygous mothers have a 50% Commonlv more se,·ere in males. Females
'
• 26 chance of being affected. o ma le-to-male usual ly must be homozygous to be affected .
• 27
transmission. Skips generations.
• 28
• 29 •
a
Lock
s
Suspend
8
End Bl ock
Item: 22 of 33 ~ 1 • M k -<:J 1>- Jil ~· !:';-~
QIO: 5 001

9
10
..L

- ar Pre v ious

X-linked dominant
Next Lab fli!ltues Not es

Transmitted through both parents. 'vtothers


transmit to 50% of daughters and sons; fat hers
Calcula t o r

Hypophosphatemic rickets-formerly known as


vitamin 0-resistant rickets. Inherited disorder
11
transmit to all daughters but no sons. resulting in t phosphate wasting at proximal
12
tubule. Results in rickets-like presentation.
13 Other examples: fragile X syndrome, r\lport
14 srndrome.
15
16 Mitochondrial Transmitted only through the mother. All Variable expression in a population or e'en
17 inheritance offspring of affected females may sho" signs of \\ ithin a family due to heteroplasm).
18 disease.
Mitochondrial myopathies-rare disorders;
19 often present with myopathy, lactic acidosis,
20 and C S disease, eg, M ELAS syndrome
21 (mitochondrial encephalopathy, lactic acidosis,
and stroke-like episodes). 2° to failure in
22
oxidative phosphorylation. Muscle biopsy often
• 23
shows "ragged red fibers" (due to accumulation
• 24 of diseased mitochondria) .
• 25
0 = unaffected 1m1le; • = affected male; 0 =unaffected female; e = affected female.
• 26
• 27 FA17 p 56.4
• 28
X-linked recessive O rnithine transcarbamylase deficiency, Fabr) Oblivious Female \Yill O ften G ive ller Boys
• 29 • tti cnrttPrc tl i<:P:I<P \\ 'idrnii-A ltl rif'h •~ ntl romP 0t't.l:~r II Pr ,.J .inkPrl lli<:nrrlPr<:

a
Lock
s
Suspend
8
End Bl ock
Item: 22 of 33 ~ 1 • M k -<:J 1>- Jil ~· !:';-~
QIO: 5 001 ..L ar Pre v ious Next Lab fli!ltues Not es Calcula t o r

9
A
inhe rit ance offspring of affected females may show signs of within a family due to heteroplasmy. A

disease.
10 Mitochondrial myopathies-rare disorders;
11 often present with myopathy, lactic acidosis,
12 and C1 disease, eg, \II ELAS syndrome
(mitochondrial encephalopathy, lactic acidosis,
13
and stroke-like episodes). 2° to failure in
14 oxidati,·e phosphorylation. i\ luscle biopsy often
15 shows "ragged red fibers" (due to accumulation
16 of diseased mitochondria).
17 0 = unaffected male; • = affected male; 0 = unaffected female; e = affected female.
18
19 FA17 p 56.4

20 X-linked recessive Ornithine transcarbamylase deficienC}', Fabry Oblivious Female \\'ill O ften G ive ller Boys
21 disorders disease, \Viskott- ldrich syndrome, Ocular ller x-Linked Disorders
22 albinism, G6PD deficiency, l lunter syndrome,
Bruton agammaglobul in em ia, l lemoph ilia
• 23
A and B, Lcsch-Nyhan syndrome, Duchcnnc
• 24
(and Becker) muscular dystrophy.
• 25 Lyonization -fcmale carriers variably affected
• 26 depending on the pattern of inaet ivai ion of the
• 27 X chromosome carrying the mutant vs normal
gene.
• 28
• 29 •
a
Lock
s
Suspend
8
End Bl ock
Item: 23 of 33 ~. I • M k <:] t> al ~· ~
QIO: 4091 .l. ar Previous Next lab 'lifllues Notes Calculator


9
A 70-yea r-old man complains of a long history of pain in his ankles, toes, and fingers. He reports exqu isite
10 painfu l f lare-ups eve ry few months that are fol lowed by asymptomatic periods. In recent years, he has
11 developed near -constant arth ralgia at basel ine, but what mainly concerns him today is the development of
yellow-white lesions on his ears and Achilles tendon . On examination, the lesions are pa lpable but not tende r to
12
the touch; the ea r lesions do not t ransilluminate. Moreover, you appreciate significant swell ing of the joints,
13 especia lly the toes and knees.
14
15 What is the mechan ism of action of the med ication often used for long-term management that also reduces the risk
16 for developing such lesions?
17 :
18 A. Binds tubu lin
19 B. Blocks fo rmation of prostagland ins and th romboxane f rom arach idonic acid
20
C. Inhibits release of phospholipase A 2
21
22
D. Inhibits xanthine oxidase
• 23 E. Se lective, competitive angiotensin II recepto r inh ibition
• 24
• 25
• 26
• 27
• 28
• 29 •

6
lock
s
Suspend
0
End Block
Item: 23 of 33 ~ 1 • M k -<:J 1>- Jil ~· !:';-~
QIO: 4 0 91 ..L ar Pre v ious Next Lab fli!ltues Not es Calcula t o r
A A

9 The correct a n sw e r is D. 72°/o chose this.


10 The patient has a classic case of chronic gout, with intermittent
11 attacks eventually giving rise to disfiguring tophi (shown in the
images). Gouty arthritis is a common manifestation of
12
hyperuricemia, and tophi form as a result of the accumulation of
13 monosodium urate crystals surrounded by reactive fibroblasts and
14 chronic inflammatory cells in the joints and soft tissues. Common
15 extra-articular sites of tophus formation include the Achilles tendon ;:=:::::::::!!!!!!!!~~!!!!!!!!::::=:;::-' 1;;;=====!!:::==~
and the helix of the external ear. Aspiration of the tophus usually
16
reveals the presence of negatively birefringent needle-shaped
17 crystals characteristic of gout. The best therapy of chronic gouty
18 arthritis aims to lower the levels or uric acid. Allopurinol blocks
xanthine oxidase and thus reduces the generation of uric acid.
19
Therefore it should be used in patients who overproduce uric acid
20 and in those at r isk of tumor lysis syndrome to prevent renal
21 toxicity during therapy for mal ignancies. It is the most effective
22 urate- lowering agent. However, alcohol can interfere with the
effectiveness of al lopurino l. Probenecid increases excretion of uric Images copyright © 2013 Baraf eta/.;
23 licensee BioMed Central Ltd.
acid by the kidneys and can therefore also be used in the
• 24 treatment of chronic gout .
• 25 Tumor lysis syndrome Xanthine oxidase Allopurinol Hyperuricemia Probenecid Uric acid Xanthine Gout Tophus Birefringence Achilles tendon Fibroblast Arthritis

. 26 Neoplasm Kidney Tendon Alcohol Inflammation

. 27 A is not correct. 8 °/o chose this.


. 28 Colch icine binds tubulin, thereby inhibiting microtubu le polymerization/ which blocks mitosis as well as
. 29 • neutroph il m igration. Colchicine is now considered to be second-line treatment of acute gout because of its

a
Lock
s
Suspend
8
End Bl ock
Item: 23 of 33 ~. I • M k <:] t> al ~· ~
QIO: 4091 .l. ar Previous Next lab 'lifllues Notes Calculator


9 A is not correct. 8°/o chose this.
10 Colch icine binds tubul in, thereby inh ibit ing microtubu le polymerization, which blocks mitosis as wel l as
11
neutroph il migration . Colchicine is now considered to be second-l ine treatment of acute gout because of its
narrow therapeutic window and risk of toxicity. Colch icine causes gastrointestinal upset and diarrhea in 80% of
12 people.
13 Microtubule Neutrophil Colchicine Mitosis Tubulin Gout Diarrhea Polymerization Human gastrointestinal tract Gastrointestinal tract Toxicity

14
B is not correct. 10% chose this.
15
Nonsteroidal anti-inflammatory drugs (NSAIDs) block the enzymes cyclooxygenase-1 and/or -2, thereby
16 blocking production of inflammatory mediators. NSAIDs have been used successfu lly in treatment of acute gout.
17 Most NSAIDs can be used in t reatment of gout but care must be taken with giving indomethacin to elderly
patients (central nervous system adverse effects). An agent with a quick onset of action is desired, but aspirin
18
should not be used because it can alter uric acid levels and potentially prolong and intensify an acute attack.
19 Indometacin Uric acid Gout Aspirin PTGSl Anti-inflammatory Nonsteroidal anti-inflammatory drug Inflammation Enzyme Nervous system

20
C is not correct. 9°/o chose this.
21
Corticosteroids are used to t reat episodes of acute gout in patients who cannot use nonsteroida l anti-
22 inflammatory dr ugs. Steroids have specific and nonspecific anti-inflammatory functions. Steroids inhibit release
23 of phospholipase A 2, an enzyme responsib le for the for mation of prostag landins, leukotrienes, and other
• 24 derivatives of the arachidon ic acid pathway . Intra-articula r corticosteroid injections may be particularly useful in
patients with monoarticu lar gout flare-ups .
• 25 Corticosteroid Arachidonic acid Gout Enzyme Prostaglandin leukotriene Anti-inflammatory Nonsteroidal anti-inflammatory drug Steroid Phospholipase A2
• 26
Phospholipase Anabolic steroid
• 27
E is not correct. 1°/o chose this .
• 28
Losartan, an inh ibitor of ang iotensin II receptor, is used t raditiona lly to t reat hypertension and may delay
• 29 • ....... -,.., ...-,...,.. : _ .... -~ ...1:-.J....,....p ;,... ...,,... .... J.... ..,,... .... ....,.pJ...., , T+- J....-.,..-. _...,...,..a- ... -.+-- 1, , .... ,..... _....,..., , , ..,;,..._,..., , .,..;.- -++,...,...+- -. .... .....1 .pJ,..,.,,,... _.. ..... , , J,..,.,... , ,,.._,..a ;...,

6
lock
s
Suspend
0
End Block
Item: 23 of 33 ~. I • M k <:] t> al ~· ~
QIO: 4091 .l. ar Previous Next lab 'lifllues Notes Calculator

• patients (central nervous system adverse effects). An agent with a quick onset of action is desired, but aspirin
9
should not be used because it can alter uric acid levels and potentially prolong and intensify an acute attack.
10
Indometacin Uric acid Gout Aspirin PTGSl Anti-inflammatory Nonsteroidal anti-inflammatory drug Inflammation Enzyme Nervous system
11

12
C is not correct. 9°/o chose this.
Corticosteroids are used to t reat episodes of acute gout in patients who cannot use nonsteroida l anti-
13
inflammatory dr ugs. Steroids have specific and nonspecific anti-inflammatory functions. Steroids inhibit release
14 of phospholipase A 2, an enzyme responsib le for the for mation of prostag landins, leukotrienes, and other
15 derivatives of the arachidon ic acid pathway . Intra-articula r corticosteroid injections may be particularly useful in
16
patients with monoarticu lar gout flare-ups.
Corticosteroid Arachidonic acid Gout Enzyme Prostaglandin leukotriene Anti-inflammatory Nonsteroidal anti-inflammatory drug Steroid Phospholipase A2
17
Phospholipase Anabolic steroid
18
19 E is not correct. 1°/o chose this.
20 Losartan, an inh ibitor of ang iotensin II receptor, is used t raditiona lly to t reat hypertension and may delay
progression of diabetic nephropathy. It has a moderate ly potent uricosuric effect and thus may be used in
21
treatment of chronic gout, but not as fi rst-l ine therapy.
22 losartan Diabetic nephropathy Uricosuric Angiotensin Angiotensin II Gout Hypertension Diabetes mellitus Enzyme inhibitor Kidney disease

23 Receptor (biochemistry)

• 24
• 25
Bottom Line:
• 26
• 27
Uric acid-lower ing agents (a llopu r inol, probenecid) are first-line therapy for chronic gouty arth r itis .
Allopurinol Probenecid Gout Arthritis
• 28
• 29 •

6
lock
s
Suspend
0
End Block
Item: 23 of 33 ~ 1 • M k -<:J 1>- Jil ~· !:';-~
QIO: 4 0 91 ..L ar Pre v ious Next Lab fli!ltues Not es Calcula t o r
A A

9
FA17 p 440.1
10
Gout
11
FINDINGS Acute inflammatory monoarthritis caused by precipitation of monosodium urate crystals in
12
joints fl. Yfore common in males. Associated with hyperuricemia, which can be caused by:
13 Underexcretion of uric acid (90% of patients)- largely idiopathic, potentiated by renal failure;
14 can be exacerbated b) certain medications (eg, thiazide diuretics).
15 o,·erproduction of uric acid (10% of patients)-Lesch-1\yhan syndrome, PRPP excess, f cell
16 tumO\·er (eg, tumor lysis S}11drome}, von Gierke disease.
Crystals are needle shaped and 8 birefringent under polarized light {yellow under parallel light,
17
blue under perpendicular light : ).
18
SYMPIOMS Asrmmetric joint distribution. Joint is swollen, red, and painful. Classic manifestation is painful
19
MTP joint of big toe (podagra). Tophus formation ~ (often on external ear, olecranon bursa,
20 or Achilles tendon). Acute attack tends to occur after a large meal with foods rich in purines
21 {eg, red meat, seafood}, trauma, surgery, dehydration, diuresis, or alcohol consumption (alcohol
22 metabol ites compete for same excretion sites in kidney as uric acid - l uric acid secretion and
23
subsequent buildup in blood).
• 24 IREATMEHI Acute: lSAIDs (cg, indomet hacin), glucocorticoids, colchicine.
Chronic {preventive): xanthine oxidase inhibitors (eg, allopurinol, fe buxostat).
• 25
• 26
• 27
• 28
• 29 •
a
Lock
s
Suspend
8
End Bl ock
Item: 23 of 33 ~ 1 • M k -<:J 1>- Jil ~· !:';-~
QIO: 4 0 91 ..L ar Pre v ious Next Lab fli!ltues Not es Calcula t o r
A A

9 FA17 p457.2
Gout drugs
10
11 Chronic gout drugs (preventive)

12 Allopurinol Competitive inhibitor of xanthine oxidase. D.et - - . Purines - - Nucleic acids


l conversion of hypoxanthine and xanthine to
13
urate. Also used in lymphoma and leukemia
14 Hypoxanthine
to pre,·ent tumor I) sis-associated urate
15 Xanthine
nephropathy. t concentrations of azathioprine
16 and 6-:\ IP (both normally metabolized by Xanth•~xidase ) Allopurinol
xanthine oxidase). - febuxostat
17 Xanthine
Febuxostat Inhibits xanthine oxidase. oxidase
18
Pegloticase Recombinant uricase that catalyzes metabolism Plasma - - Urate crystals - - . Gout
19
uric acid deposited
20 of uric acid to allantoin (a more water-soluble in joints
product).
21
22
Probenecid Inhibits reabsorption of uric acid in proximal Tubular
convoluted tubule (also inhibits secret ion of
~
reabsorption
23
penicillin). Can precipitate uric acid calculi. Probenecid and
• 24 high-dose salicylates
Acute gout drugs Tubular
• 25 -=----- secretion
::;:
~f----~
NSAIOs Anr fu ll-dose 1SAID (eg, naproxcn,
• 26
indomethacin). Avoid sal icylates (may decrease Diuretics and
Urine low-dose saUcylates
• 27 uric acid excretion, particularly at low closes).
• 28 Glucocorticoids OraJ, intra-articular, or parenteral.
• 29 •
a
Lock
s
Suspend
8
End Bl ock
Item: 24 of 33 ~ 1 • M k -<:J 1>- Jil ~· !:';-~
QIO: 5 009 ..L ar Pre v ious Next Lab fli!ltues Not es Calcula t o r

IAA]
A A

9
A 55-year-old woman presents to her physician because of pain and tenderness in the proximal
10 interphalangea l and metacarpophalangeal joints of her hands that is worst in the morning and often lasts
11 until lunchtime. On examination these joints appear inflamed. She also has a low-grade fever. Laboratory
examination demonstrates an elevated erythrocyte sedimentation rate and rheumatoid factor level.
12
13
Which of the following HLA subtypes has been associated with this condition?
14
15 :
A. HLA-B27
16
17 B. HLA-B8
18 C. HLA-BW22
19
D. HLA-DR3
20
E. HLA-DR4
21
22
23
• 24
• 25
. 26
. 27
. 28
. 29 •
a
Lock
s
Suspend
8
End Bl ock
Item: 24 of 33 ~. I • M k <:] t> al ~· ~
QIO: 5009 .l. ar Previous Next lab 'lifllues Notes Calculator


9
The correct answer is E. 60°/o chose this.
10
This woman demonstrates classic symptoms of rheumatoid arthritis, which is associated with HLA-DR4. Other
11
diseases associated with HLA-DR4 include diabetes me llitus.
12 Rheumatoid arthritis Diabetes mellitus HLA-DR4 Arthritis

13
A is not correct. 18% chose this.
14
HLA-B27 is associated with ankylosing spondylitis, postgonococcal arthritis, and acute anterior uveitis. I n the
15 setting of a patient with HLA-B27, dista l interpha langeal synovit is wou ld be consistent with a diagnosis of
16 psoriatic arthrit is.
Ankylosing spondylitis Psoriatic arthritis Uveitis HLA-827 Arthritis Spondylitis Synovitis Anatomical terms of location
17
18 B is not correct. 5°/o chose this.
19 HLA-B8 is associated with Graves disease.
Graves' disease HLA-BS
20
21 C is not correct. 3°/o chose this.
22 HLA-BW22 is associated with Kawasaki disease.
Kawasaki disease Kawasaki Heavy Industries Motorcycle & Engine Kawasaki motorcycles Kawasaki Heavy Industries Kawasaki, Kanagawa
23
24 D is not correct. 14% chose this.
• 25 HLA-DR3 is associated with chronic active hepatitis, Sjogren synd rome, and type 1 diabetes me llitus .
Diabetes mellitus type 1 Diabetes mellitus Hepatitis HLA-DR3
• 26
• 27
• 28 Bottom Line:
• 29 •

6
lock
s
Suspend
0
End Block
Item: 24 of 33 ~. I • M k <:] t> al ~· ~
QIO: 5009 .l. ar Previous Next lab 'lifllues Notes Calculator
• -- - - • - -. l t -- .. - •• - • -.- - - •

9 Rheumatoid arthritis Diabetes mellitus HLA-DR4 Arthritis

10
A is not correct. 18% chose this.
11
HLA-B27 is associated with ankylosing spondylitis, postgonococcal arthritis, and acute anterior uveitis. I n the
12 setting of a patient with HLA-B27, dista l inte rphalangeal synovit is wou ld be consistent with a diagnosis of
13 psoriatic arthrit is.
Ankylosing spondylitis Psoriatic arthritis Uveitis HLA-827 Arthritis Spondylitis Synovitis Anatomical terms of location
14
15 B is not correct. 5°/o chose this.
16 HLA-B8 is associated with Graves disease.
Graves' disease HLA-BS
17
18 C is not correct. 3°/o chose this.
19 HLA-BW22 is associated with Kawasaki disease.
Kawasaki disease Kawasaki Heavy Industries Motorcycle & Engine Kawasaki motorcycles Kawasaki Heavy Industries Kawasaki, Kanagawa
20
21 D is not correct. 14% chose this.
22 HLA-DR3 is associated with chronic active hepatitis, Sjogren synd rome, and type 1 diabetes me llitus.
Diabetes mellitus type 1 Diabetes mellitus Hepatitis HLA-DR3
23
24
• 25 Bottom Line:
• 26
HLA-DR4 is strong ly associated with rheumato id arth r itis. There is also an association between HLA-DR4 and
• 27 type 1 diabetes mellitus .
Rheumatoid arthritis HLA-DR4 Diabetes mellitus type 1 Diabetes mellitus Arthritis
• 28
• 29 •

6
lock
s
Suspend
0
End Block
Item: 24 of 33 ~ 1 • M k -<:J 1>- Jil ~· !:';-~
QIO: 5 009 ..L ar Pre v ious Next Lab fli!ltues Not es Calcula t o r

9
10
11 ljl;fil!11•1
FIRST AID FACTS
for y e ar: 2017 •
12
13 FA17 p 96.2
14 HLA subtypes associated with diseases
15 A3 Hemochromatosis
16 88 Addison disease, myasthenia gra' is, Cra,cs
17 disease
18 827 Psoriatic arthritis, \ nkylosing spondylitis, P\IR. Also known as seronegati,·e arthropathies.
19 IBD-associated arthritis, Reactive arthritis
20 DQ2/ DQ8 Celiac disease I ate (8) too (2) much gluten at Dairy Queen.
21 DR2 l\Iultiplc sclerosis, hay fever, SLE, l\lultiple hay pastures have d irt.
22 Goodpasture syndrome
23 DR3 Diabetes mellitus type I, SLE, Craves disease, 2-3, S-L-E
24 Hashimoto thyroiditis, Addison disease
• 25 DR4 Rheumatoid arthritis, diabetes mellitus type I, There are 4 walls in a "rheum" (room).
. 26
ddison disease
. 27 DRS Pernicious anemia - vitamin B12 deficiency,
Hashimoto thyroiditis
. 28
. 29 •
a
Lock
s
Suspend
8
End Bl ock
Item: 25 of 33 ~ 1 • M k -<:J 1>- Jil ~· !:';-~
QIO: 4695 ..L ar Pre v ious Next Lab fli!ltues Not es Calcula t o r
A A

13
A 32-year-old woman is found to have a mediastinal mass whi le having a plain f ilm X- ray of the chest
14 performed (shown in the image) . She subsequently reports that lately she has been "feeling more fatigued,
15 especially at the end of the day." In the office, she has difficulty rising from her chair to move to the
16
examination table. On physical examination the patient has asymmetric left-sided ptosis, a subjective feel ing of
"blurred vision" when reading the eye chart on the wall, and a facia l droop bilateral ly.
17
18
19
20
21
22
23
24
• 25
• 26
• 27
• 28
• 29
• 30
. 31
. 32
. 33 •

a
Lock
s
Suspend
8
End Bl ock
13
14
15
16
17
18
19
20
21
22
Image courtesy of Dr Hani AI Salam, Radiopaedia.org, riD: 8023
23
24 What test wou ld confirm the most likely diagnosis in this patient?
• 25
:
• 26 A. Acetylcho line receptor ant ibody assay
• 27
B. Complete blood cel l count
• 28
• 29 C. Parathyroid hormone level
• 30 D. Thyroid stimu lating hormone level
. 31
E. Ur ine catecholamine levels
. 32
. 33 •

a
Lock
s
Suspend
8
End Block
Item: 25 of 33 ~. I • M k <:] t> al ~· ~
QIO: 4695 .l. ar Previous Next lab 'lifllues Notes Calculator


13 The correct answer is A. 67°/o chose this.
14 This patient has symptoms and signs consistent with a diagnosis of myasthenia gravis (MG). MG is an
15 autoimmune disorder character ized by the presence of acetylchol ine recepto r antibod ies, which cause impai red
signal transmission at the neuromuscu lar junction . Patients typica lly present with cha racteristic ocu lar (ptosis,
16
diplopia) and musculoskeletal (facia l muscle weakness, proximal muscle weakness) symptoms, which are
17 classica lly worsened by repetitive use of the involved muscles (which is why symptoms are worse by the end of
18 the day) . Thymomas (evidenced by the widened superior mediastinum in the X- ray) occur in app roximately
19 10% of patients with MG. Pure red cel l aplasia is a paraneoplastic syndrome seen in 5% - 15% of thymomas, and
some patients may have the rare t riad of MG, thymoma, and red cell aplasia. The apparent left lower lobe
20
co llapse is likely secondary to compression by th is mass. The diagnosis of MG can be confirmed by measu ring
21 acetylchol ine receptor antibodies and perfor ming electromyog raphy and/or a Tensilon test. Treatment generally
22 consists of an acetylcho lineste rase inhibito r such as pyridostigmine.
Paraneoplastic syndrome Acetylcholinesterase inhibitor Myasthenia gravis Diplopia Thymoma Neuromuscular junction Electromyography Autoimmune disease
23
Mediastinum Ptosis (eyelid) Acetylcholine Pyridostigmine Acetylcholinesterase Autoimmunity Acetylcholine receptor Muscle weakness Edrophonium Antibody
24
Facial muscles Human musculoskeletal system Anatomical terms of location Muscle
25
• 26 B is not correct. 8°/o chose this .
• 27 Lymphoma can manifest with a mediastinal mass and constitutional symptoms, but is not routine ly associated
• 28
with muscu loske letal wea kness. A complete blood cel l count is useful in making the diagnosis in suspected
cases .
• 29 Complete blood count lymphoma Mediastinal tumor Mediastinum Human musculoskeletal system
o30
C is not correct. 11% chose this .
• 31
A parathyroid gland tumor can infrequently manifest with a med iastina l mass and commonly with generalized
• 32
weakness secondary to high serum ca lcium levels. Measurements of parathyroid hor mone level are usefu l in
• 33 ~ ma kinq the diaqnosis in suspected cases. ~

6
lock
s
Suspend
0
End Block
Item: 25 of 33 ~. I • M k <:] t> al ~· ~
QIO: 4695 .l. ar Previous Next lab 'lifllues Notes Calculator

13
cases.
Complete blood count lymphoma Mediastinal tumor Mediastinum Human musculoskeletal system
14
15 C is not correct. 11% chose this.
16 A parathyroid gland tu mor can infrequently man ifest wit h a med iastina l mass and commonly with generalized
weakness secondary to high serum ca lcium levels. Measurements of parathyroid hor mone level are usefu l in
17
ma king the diagnosis in suspected cases.
18 Parathyroid gland Parathyroid hormone Neoplasm Mediastinum Blood plasma Hormone Serum calcium Gland Calcium

19
D is not correct. 8°/o chose this.
20
Hypothyroidism can manifest with gene ral ized fatigue and muscu loskeletal weakness, but is not routinely
21 associated with a mediastinal mass. Measu rements of the thyroid-stimulating hormone leve l are usefu l in
22 ma king the diagnosis in suspected cases.
Thyroid-stimulating hormone Hypothyroidism Mediastinum Fatigue (medical) Hormone Human musculoskeletal system
23
24 E is not correct. 6°/o chose this.
25 Neu roblastoma can man ifest with a mediastinal mass and constitutiona l symptoms, although it is typica lly a
• 26
pediatric ma lignancy. Testing of ur ine catecholamine levels is useful in making the diagnosis in suspected cases .
Catecholamine Neuroblastoma Cancer Malignancy Urine Pediatrics
• 27

• 28
• 29 Bottom Line:
o30 Myasthenia gravis ( MG ) is an autoimmune disorder characterized by the presence of acetylcho line receptor
• 31 antibodies . Thymomas occur in app roximately 10% of patients with MG .
Myasthenia gravis Autoimmune disease Acetylcholine Antibody Thymoma Autoimmunity Acetylcholine receptor Muscle weakness Receptor (biochemistry)
• 32

• 33

6
lock
s
Suspend
0
End Block
Item: 25 of 33 ~. I • M k <:] t> al ~· ~
QIO: 4695 .l. ar Previous Next lab 'lifllues Notes Calculator

• •
13
FA17 p445.2
14
Neuromuscular junction diseases
15
Myasthe nia gravis l a mbe rt-Eaton myasthe nic syndrome
16
FREQUENCY tvlost common 1 MJ disorder Uncommon
17
PATHOPHYSIOLOGY Autoantibodies to postsynaptic AC h receptor Autoantibodies to presynaptic Ca 2+ channel
18
-+ ! ACh release
19
CLINICAL Ptosis, diplopia, weakness Proximal muscle weakness, autonomic
20
\~orsen s with muscle use symptoms (dry mouth, impotence)
21 Improvement after edrophonium (tensilon) test Improves with muscle use
22 ASSOCIATED WITH Thymoma, thymic hyperplasia Small cell lung cancer
23
AChE INHIBITOR ADMINISTRATION Reverses symptoms (edrophonium to diagnose, Minimal effect
24 pyridostigmine to treat)
25
• 26
FA11 p217.1
• 27 Paraneoplastic syndromes
• 28 MANIFESTATION DESCRIPTION/MECHANISM MOST COMMONLY ASSOCIATED CANCER(S)
• 29 Cutaneous
o30 Acanthosis nigricans I Iyperpigmcnted velvety plaques in axilla and Gastric adenocarcinoma and other visceral
• 31 neck malignancies (but more commonly associated
• 32
with obesity and insulin resistance)

• 33
Sign of Leser-Trelat Sudden onset of multiple seborrheic keratoses Gl adenocarcinomas and other visceral •

6
lock
s
Suspend
0
End Block
Item: 25 of 33 ~. I • M k <:] t> al ~· ~
QIO: 4695 .l. ar Previous Next lab 'lifllues Notes Calculator

• Paraneoplastic syndromes •
13
MANIFESTATION DESCRIPTION/MECHANISM MOST COMMONLY ASSOCIATED CANCER(S)
14
15
Cutaneous

16
Acanthosis nigricans I lyperpigmcnted velvety plaques in axilla and Gastric adenocarcinoma and other visceral
neck malignancies (but more commonly associated
17
with obesity and insulin resistance)
18
Sign of Leser-Trelat Sudden onset of multiple seborrheic keratoses C l adenocarcinomas and other visceral
19
malignancies
20
Endocrine
21
Hypercalcemia PTHrP Squamous cell carcinomas of lung, head, and
22 neck; renal, bladder, breast, and ovarian
23 carcmomas
24 t 1,25-(0Hh vitamin 0 3 (calcitriol) Lymphoma
25 Cushing syndrome t ACTH
Small cell lung cancer
• 26 Hyponatremia (SIADH) t AD11
• 27 Hematologic
• 28 Polycythemia t Erythropoietin Renal cell carcinoma, hepatocellular
• 29 carcinoma, hemangioblastoma,
o30 pheochromocytoma, leiomyoma
• 31 Pure red cell aplasia Anemia with low rcticulocytes
Thymoma
• 32 Good syndrome Hypogammaglobulinemia
• 33 • Trousseau svndrome li!:!ratorv suocrficial thromboohlebitis •

6
lock
s
Suspend
0
End Block
Item: 25 of 33 ~ 1 • M k -<:J 1>- Jil ~· !:';-~
QIO: 4695 ..L ar Pre v ious Next Lab fli!ltues Not es Calcula t o r

13 Good syndrome Hypogammaglobul•nem•a
14 Trousseau syndrome r-. Iigratory superficial th romboph lebit is
15 Nonbacterial Deposition of sterile platelet thrombi on heart Adenocarcinomas, especially pancreatic
16 thrombotic (marantic) valves
17
endocarditis

18 Neuromuscular

19 Anti-NMDA receptor Psychiatric disturbance, memory deficit , Ovarian teratoma


20
encephalitis seizures, dyskinesias, autonomic instability,
language dysfunction
21
Opsoclonus- "Dancing eyes, chmcing feet" 'euroblastoma (children), small cell lung
22
myoclonus ataxia cancer (adults)
23 syndrome
24
Paraneoplastic Antibodies against antigens in Purkinje cells Small cell lung cancer (anti-Hu), gynecologic
25 cerebellar and breast cancers (anti-Yo), and Hodgkin
• 26 degeneration lymphoma (anti-Tr)
• 27 Paraneoplastic Antibodies against Hu antigens in neurons Small cell lung cancer
• 28 encephalomyelitis
• 29 Lambert-Eaton Antibodies against presynaptic (P/Q-type) Ca 2+ Sma II cell hmg cancer
• 30 myasthenic syndrome channels at \llj
. 31 Myasthenia gravis Antibodies against postS)'naptic ACh receptors Thymoma
. 32 at N:VIJ
. 33 •

a
Lock
s
Suspend
8
End Bl ock
Item: 26 of 33 ~. I • M k <:] t> al ~· ~
QIO: 3189 .l. ar Previous Next Lab 'lifllues Notes Calculator


13
Late one even ing, a 57-year-o ld man has sudden-onset, severe pain in his left great toe. He presents to his
14 primary ca re physician the next day, and reports no history of t rauma . On physical examination there is
15 edema with erythema and pain on movement of the left first metatarsopha langeal joint, but there is no
overlying skin ulceration. A joint aspi rate is performed and microscopy reveals numerous neutrophi ls. Over the
16
next 4 weeks, he has two simi lar episodes. On physica l examination between these attacks, there is minima l loss
17 of joint mobil ity.
18
19 Wh ich of the fo llowing laboratory find ings is most characteristic for his under lying disease process'
20
:
21 A. Hypercalcemia
22
B. Hyperglycemia
23
24
C. Negatively birefringent crysta ls in joint aspirate
25 D. Positive rheumatoid factor
• 26 E. Positively birefringent crystals in joint aspi rate
• 27
• 28
• 29
o30
• 31
• 32
• 33 ~

6
lock
s
Suspend
0
End Block
Item: 26 of 33 ~ 1 • M k -<:J 1>- Jil ~· !:';-~
QIO: 3189 ..L ar Prev ious Next Labfli!ll ues Not es Calculat o r

A A

13
14 The correct answer is C. 79°/o chose this.
15 This man is suffering f rom gout. Gout causes painfu l swelling of joints,
commonly the fi rst metata rsophalangeal joint, caused by precipitation of
16
monosodium urate crystals (MSU) . It can be caused by any condition that
17 predisposes to hyperuricemia, such as Lesch-Nyhan syndrome or the use of
18 thiazide diuretics. Gout is diagnosed by viewing negatively birefringent
needle-shaped crystals in a joint aspirate of synovial fluid. Birefringence is an
19
optical property of the crystals observable under polarized filters and negative
20 birefringence means MSU crystals appear yellow when parallel and blue when
21 perpendicular to the light source (as demonstrated in this image). The
22 presence of neutrophils in the joint aspirate demonstrates the inflammatory
nature of the immune system's reaction to the crystal.
23
Lesch·Nyhan syndrome Hyperuricemia Metatarsophalangeal articulations Gout Birefringence Synovial fluid Image courtesy of Ed Uthman,
24 MD
Uric acid Immune system Diuretic Neutrophil Thiazide Crystal
25
26 A is not correct. 2°/o chose this.
• 27 Hypercalcemia can be caused by hy perparath yroid ism , wh ich may result in bone lesions (osteit is fibrosa
cystica). Excess parathyroid horm one sti m ulates osteoblast s t o increased product ion of recept or activator of
• 28
nuclear factor-KB ligand ( RANK-L) and M-CSF, which subsequently stimulat e osteoclasts to resorb bone . The
• 29 resu lt is cystic bone lesions and hypercalcem ia. This would be an inflammatory process, as is the case presented
• 30 in the vignett e. However, it does not typica lly affect joints, nor does it cause swelling or redness .
Osteitis fibrosa cystica Hyperparathyroidism Parathyroid hormone Hypercalcaemia Osteoclast Osteoblast Parathyroid gland Ligand Hormone Ligand (biochemistry)
. 31
Receptor (biochemistry) Bone Inflammation
. 32
. 33 •
B is not correct. 1 °/o chose this .

a
Lock
s
Suspend
8
End Block
Item: 26 of 33 ~ 1 • M k -<:J 1>- Jil ~· !:';-~
QIO: 3189 ..L ar Prev ious Next Labfli!ll ues Not es Calculat o r
+ " I I T •• • - ••
A A

13
14 B is not correct. 1 °/o chose this.
15 Hyperglycemia is a feature of diabetes mellitus/ a systemic endocrine disorder. Through nonenzymatic
glycosylation of arterioles/ hyperglycemia causes peripheral vascula r disease/ thus increasing the risk of foot
16
ulcerations and osteomyelitis.
17 Osteomyel t s Diabetes mel ;tus Endocrine disease Hyperglycemia Peripheral artery disease Endocrine system Vascular disease Peptic ulcer Glycosylation

18 Ulcer 'dermatology\ Blood vessel Arteriole

19
D is not correct. 3 °/o chose this.
20
Positive rheumatoid factor is present in 80% of patients with rheumatoid arthritis. This pain is typically
21 persistent for weeks to months and, if untreated, is associated with significant joint deformities.
22 Rheumatoid factor Rheumatoid arthritis Arthritis Contracture

23 E is not correct. 15°/o chose this.


24 Pseudogout manifests w ith symptoms that m im ic those of gout but is caused
25 by the precipitation of ca lcium pyrophosphate crystals (CPPD) w ith in the joint
space, which often have a rhomboidal or rectangular shape, as shown in this
26
image. These crystals are positively birefringent/ wh ich means the CPPD
• 27 crysta ls appear blue when parallel to the light source. This disease classica lly
• 28 affects large joints, most common ly t he knee, in men over 50 years of age .
Chondrocalcinosis Gout Birefringence Calcium pyrophosphate Calcium Pyrophosphate
• 29
• 30
. 31
. 32 Image courtesy of David Iberri
. 33 •

a
Lock
s
Suspend
8
End Block
Item: 26 of 33 ~. I • M k <:] t> al ~· ~
QIO: 3189 .l. ar Previous Next Lab 'lifllues Notes Calculator


13
D is not correct. 3°/o chose this.
14 Positive rheumatoid factor is present in 80% of patients with rheumatoid arthritis. This pain is typical ly
15 persistent for weeks to months and, if untreated, is associated with significant joint deformities.
Rheumatoid factor Rheumatoid arthritis Arthritis Contracture
16
17 E is not correct. 15% chose this.
18 Pseudogout manifests with symptoms that mimic those of gout but is caused
19 by the precipitation of calcium pyrophosphate crystals (CPPD) within the joint
space, which often have a rhomboidal or rectangular shape, as shown in th is
20
image. These crysta ls are positively birefringent, which means the CPPD
21 crystals appear blue when paral lel to the light source. This disease classically
22 affects large joints, most commonly the knee, in men over 50 years of age.
Chondrocalcinosis Gout Birefringence Calcium pyrophosphate Calcium Pyrophosphate
23
24
25
26 Image courtesy of David Iberri
• 27
• 28
Bottom Line:
• 29
o30
Gout is caused by urate crystals and resu lts in painfu l swollen joints (often the first metatarsophalangea l).
Negatively birefringent crysta ls are diagnostic .
• 31 Gout Birefringence Uric acid Metatarsophalangeal articulations
• 32
• 33 ~

6
lock
s
Suspend
0
End Block
Item: 26 of 33 ~ 1 • M k -<:J 1>- Jil ~· !:';-~
QIO: 3189 ..L ar Pre v ious Next Labfli!llues Not es Calcula t o r
A A

13
FA17 p 440.1
14
Gout
15
FINDINGS Acute inflammatory monoarthritis caused by precipitation of monosodium urate crystals in
16
joints · . More common in males. Associated with hrperuricemia, which can be caused by:
17 Underexcretion of uric acid (90% of patients)-largely idiopathic, potentiated by renal failure;
18 can be exacerbated br certain medications (eg, thiazide diuretics).
19 0\·erproduction of uric acid ( 10% of patients)- Lesch-1\yhan syndrome, PRPP excess. f cell
20 turnover (eg, tumor lysis S} ndrome), von Gierke disease.
21
e
CrystaJs are needle shaped and birefringent under polarized light (yellow under parallel light,
blue under perpendicular light ).
22
SYMPTOMS Asymmetric joint distribution. Joint is swollen, red, and painful Classic manifestation is painful
23
MTP joint of big toe (podagra). Tophus formation (often on external ear, olecranon bursa,
24 or Achilles tendon). Acute attack tends to occur after a large meal with foods rich in purines
25 (eg, red meat, seafood), trauma, surgery, dehydration, diuresis, or alcohol consumption (alcohol
26 metabol ites compete for same excretion sites in kidney as uric acid -+ l uric acid secretion and
• 27
subsequent buildup in blood).

• 28
TREATMENT Acute: SA!Ds (eg, indomethacin), glucocorticoids, colchicine.
Chronic (preventive): xa nth ine oxidase inhibitors (eg, allopurinol, febuxostat).
• 29
• 30
• 31
• 32

• 33

a
Lock
s
Suspend
8
End Bl ock
Item: 27 of 33 ~ 1 • M k -<:J 1>- Jil ~· !:';-~
QIO: 3188 ..L ar Pre v ious Next Labfli!llues Not es Calcula t o r

IAA]
A A

13
A 64-year-old woman with no prior med ica l history has had increasing back pa in and right hip pain fo r the
14 past decade. On physical examination, she has bony, cool to the touch enlargement of both the proxima l and
15 distal interphalangeal joints, as shown in the image .
16
17
18
19
20

21
22

23
24
25
26
• 27
• 28
• 29 I mage courtesy of Wikimedia Commons
• 30
• 31 Wh ich of the following diseases is most likely the cause of this patient's symptoms?
• 32 :
• 33 A. Gout

a
Lock
s
Suspend
8
End Bl ock
Item: 27 of 33 ~. I • M k <:] t> al ~· ~
QIO: 3188 .l. ar Previous Next Lab 'lifllues Notes Calculator


13
14
The correct answer is B. 77°/o chose this.
Osteoarthritis (OA) is a disease of wea r and tear, leading to destruction of articu lar carti lage, subchondral bone
15
for mation, osteophytes, sclerosis, and other degenerative changes. It is common and progressive and becomes
16 more so with age. It classica lly man ifests in weight-bearing joints as pain after use, improving with rest. It
17 commonly affects the distal interphalangeal joints as well. Common imaging findings include narrowing of the
18
joint space, sclerosis, and the presence of osteophytes. The image reveals the presence of Bouchard and
Heberden nodes, representing bony enlargement of the proximal and distal interpha langeal joints, respectively.
19
Of note, in contrast to rheumatoid arthritis, OA may manifest with invo lvement of both joints.
20 Rheumatoid arthritis Osteoarthritis Epiphysis Articular cartilage Osteophyte Cartilage Interphalangeal articulations of hand Arthritis Ossification

21 Anatomical terms of location Bone

22
A is not correct. 4°/o chose this.
23
Gout is a painfu l swel ling of a joint, most common ly the metatarsopha langeal joint, caused by precipitation of
24 monosodium urate crystals. It is diagnosed by viewing the crysta ls in the joint's synovial fluid, which are
25 negative ly birefringent. It is not associated with osteophytes or sclerosis with nar rowing
Metatarsophalangeal articulations Gout Birefringence Synovial fluid Uric acid Osteophyte Synovial membrane Synovial joint
26
27 C is not correct. 2°/o chose this.
• 28 Osteomye litis is an infection in the bone. It presents most commonly with tenderness, warmth, swelling, and
• 29
more acute pa in, rather than joint narrowing . The pa in typically is present with and without movement .
Osteomyelitis Bone Infection
o30
• 31 D is not correct. 3°/o chose this .
• 32
Pseudogout causes symptoms that mimic those of gout, but is caused by the precipitation of calcium
pyrophosphate crystals within the joint space. These crystals are positive ly birefringent. This disease classical ly
• 33 ~ ;::offprt"c: l;::ornP inint-c: mnc:t" rnmmnnlv t-hP k- nPP i n mPn nvPr c;n vP;::orc: nlrl Tt" ic: nnt" ;::oc:c:nri;::ot-Prl w ith nc:t"Pnnhvt"Pc: nr

6
lock
s
Suspend
0
End Block
Item: 27 of 33 ~. I • M k <:] t> al ~· ~
QIO: 3188 .l. ar Previous Next Lab 'lifllues Notes Calculator


13
C is not correct. 2°/o chose this.
14
Osteomye litis is an infection in the bone. It presents most commonly with tenderness, warmth, swelling, and
15 mo re acute pa in, rather than joint narrowing . The pa in typically is present with and without movement.
16 Osteomyelitis Bone Infection

17 D is not correct. 3°/o chose this.


18 Pseudogout causes symptoms that mimic those of gout, but is caused by the precipitation of calcium
19 pyrophosphate crystals within the joint space. These crystals are positive ly bi refringent. This disease classical ly
affects large joints, most commonly the knee, in men over 50 years old. It is not associated with osteophytes or
20
sclerosis with narrowing .
21 Chondrocalcinosis Gout Birefringence Calcium pyrophosphate Calcium Pyrophosphate Osteophyte

22
E is not correct. 14% chose this.
23
Rheumato id arth r itis is an autoimmune arth ritis caused by inflammato ry destruction of synovia l joints. It is
24 associated with pain that is worst in the morning, imp roving with use, and classica lly affects the proxima l
25 interphalangeal joints and spa res the distal interphalangeal joints. It is not associated with osteophytes or
26 sclerosis with narrowing .
Rheumatoid arthritis Arthritis Osteophyte Synovial joint Interphalangeal articulations of hand Synovial membrane Autoimmunity Synovial fluid
27
Autoimmune disease Anatomical terms of location Inflammation
• 28
• 29
o30 Bottom Line:
• 31 Bouchard and Hebe rden nodes (PI P and DI P nodu les, respectively) are found in osteoarthrit is .
Osteoarthritis Interphalangeal articulations of hand
• 32
• 33 ~

6
lock
s
Suspend
0
End Block
Item: 27 of 33 ~ 1 • M k -<:J 1>- Jil ~· !:';-~
QIO: 3188 ..L ar Pre v ious Next Labfli!llues Not es Calcula t o r
A A

13
14 FA17 p439.1
Osteoarthritis and rheumatoid arthritis
15
Osteoarthritis Rheumatoid arthritis
16
17
PATHOGENESIS t\lechanical-\\ear and tear destroys articular Autoimmune-inflammation induces formation
cartilage (degenerati,·e joint disorder) of pannus (prol iferati,·e granulation tissue ),
18
.... inflammation with inadequate repair. which erodes articular cartilage and bone.
19 Chondrocytes mediate degradation and
20 inadequate repair.
21 PREDISPOSING FACTORS Age, female, obesity, joint trauma. Female, 1-1 LA-DR4, smoking, silica exposure.
22 <±> rheumatoid factor (lgM antibody that
23 targets lgG Fe region; in 80%), anti-cyclic
citrullinated peptide antibody (more specific).
24
25
PRESENTATION Pain in weight-bearing joints after use (eg, Pain, swelling, and morning stiffness lasting
at the end of the day), imprm·ing with rest. > I hour, impro,·ing with use. Symmetric
26
Asymmetric joint involvement. Knee carti lage joint involvement. Systemic symptoms
27 loss begins media lly ("bowlegged"). o (fever, fatigue, weight loss). Extraarticular
• 28 systemic symptoms. manifestations common."
• 29 JOINT FINDINGS Osteophytes (bone spurs), joint space narrowi ng, Erosions, juxta-articular osteopenia, soft tissue
• 30 subchondra I sclerosis and cysts. Sp1ovia I swelling, subchondra I cysts, joint space
• 31 Auid non-inflammatory (WBC < 2000/mml). narrowing. Deformities: cervical subluxation,
bl\'olves DIP (lleberdcn nodes : ) and PIP ulnar finger deviation, swan neck • ,
• 32
(Bouchard nodes ), and 1st C MC; not \1CP. boutonniere (]. lm·okes t\ICP, PIP, wrist; not
• 33 nlP nr ld- r' l r ' Urll'\\'; ... 1 A.. ;,.) ;.,A.,n"\n'Vl f nr\

a
Lock
s
Suspend
8
End Bl ock
Item: 27 of 33 ~ 1 • M k -<:J 1>- Jil ~· !:';-~
QIO: 3188 ..L ar Pre v ious Next Labfli!llues Not es Calcula t o r
A A

13 JOINT FINDINGS Osteophytes (bone spurs), joint space narrowi ng, Erosions, juxta-articular osteopenia, soft tissue
14 subchondral sclerosis and cysts. Synovia I swelling, subchondral cysts, joint space
15 Auid non-inflammatory (WBC < 2000/mm ~). narrowing. Deformities: cer\'ical subluxation,
16
ln\'olves DIP (Ileberden nodes ) and PIP ulnar finger deviation, swan neck • ,
(Bouchard nodes }, and 1st C.\IIC; not \1C P. boutonniere ll Im·oh-es ICP, PIP, wrist; not
17
DIP or 1st C~ IC. Svno,·ial Auid inAammalon .
J •

18
TREATMENT Acetaminophen, NSA IDs, intra-articular AIDs, glucocorticoids, disease-modifying
19
glucocorticoids. agents (methotrexate, sulfasala1ine,
20 h}dro.>.} chloroquine, leAunomide}, biologic
21 agents (eg, T F'- a inhibitors).
22 Extraarticular manifestations include rheumatoid nodules (fibrin oid necrosis" ith palisading histiocytes) in subcutaneous
23 tissue and lung (+ pneumoconiosis .... Caplan syndrome), interstitia l lung disease, pleuritis, pericarditis, anemia of chronic
24 disease, neutropenia +splenomegaly (Felty syndrome), AA amyloidosis, Sjogren syndrome, scleritis, carpal tunnel syndrome.
25 Normal Osteoarthritis Normal Rheumatoid
arthritis
26 - - ---ThiCkened
capsule Bone and
27 cartilage
Joint capsule ~ Joint capsule ~ eros1on
• 28
and synovial '\ and synovial '\
lining lining /
• 29 . - - - - - Increased
Synovial _ / SynoVIal _ / synovial fluid
• 30
cavity/ cav1ty ./' _......- Pannus
• 31 Cartilage Cartilage.../ formation
• 32

• 33

a
Lock
s
Suspend
8
End Bl ock
Item: 27 of 33 ~. I • M k <:] t> al ~· ~
QIO: 3188 .l. ar Previous Next Lab 'lifllues Notes Calculator


13
14
15
16
17
18
19
20
FA17 p440.1
21
Gout
22
FINDINGS Ac ute inflammatory monoarthritis caused by precipitation of monosodium ura te crystals in
23 joints f.J. More common in males. Associated with hyperuricemia, which can be caused by:
24 Underexcretion of uric acid (90% of patie nts)- largely idiopathic, pote ntiated by re nal failure;
25 can be exacerbated by certain med ications (eg, th iazide diu retics).
26 • O verproduction of uric acid (10% of patie nts)- Lesch-Nyhan syndrome, PRPP excess, t cell
turnover (eg, tumor lysis syndrome), von G ie rke d isease.
27
Crystals are needle shaped and 8 bi refringent under polarized light (yellow under parallel light,
• 28 blue under perpendicular light [i)).
• 29
SYMPTOMS Asymmetric joint distribution. Joint is swolle n, red, and painful. Classic ma nifestation is painful
o30 MTP joint of big toe (podagra). Tophus formation B (often on exte rnal ear, olecranon bursa,
• 31 or Achilles tendon). Acute attack tends to occur after a large meal with foods rich in purines
• 32 (eg, red meat, seafood), trauma, surgery, de hydration, diuresis, or alcohol consumptio n (alcohol
meta bolites compete for same excretion sites in kidney as uric acid - l uric acid secre tion and
• 33 ~ • I • t • • t I , ._

lock
6 s
Suspend
0
End Block
Item: 27 of 33 ~ 1 • M k -<:J 1>- Jil ~· !:';-~
QIO: 3188 ..L ar Pre v ious Next Labfli!llues Not es Calcula t o r

13
A
• Overproduction of uric acid (I0% of pal ienls)- Lesch-t'\yhan syndrome, PRPP excess, t cell A

turnover (eg, tumor lysis syndrome), von Gierke disease.


14
Crystals are needle shaped and 9 birefringent under polarited light (yellow under parallel light,
15
blue under perpendicular light : ).
16
SYMPTOMS Asymmetric joint distribution. Joint is swollen, red, and painful. Classic manifestation is painful
17 MTP joint of big toe (podagra). Tophus formation 9 (often on external ear, olecranon bursa,
18 or Achilles tendon). Acute attack tends to occur after a large meal with foods rich in purines
19 (eg, red meat, seafood), trauma, surgery, dehydration, diuresis, or alcohol consumption (alcohol
20 metabolites compete for same e:-.crelion sites in kidney as uric acid - l uric acid secretion and
subsequent buildup in blood).
21
TREATMENT Acute: 1 SAlDs (cg, indomethacin), glucocorticoids, colchicine.
22
Chronic (preventive): xanthine oxidase inhibitors (eg. allopurinol, febuxostat).
23
24
25
26
27
• 28
• 29
• 30
. 31
. 32
. 33 •

a
Lock
s
Suspend
8
End Bl ock
Item: 28 of 33 ~. I • M k <:] t> al ~· ~
QIO: 5092 .l. ar Previous Next lab 'lifllues Notes Calculator


13
A mother br ings her 2-month-old infant to the emergency department because of lethargy, fa ilure to thrive,
14 and a fever of 39 .2°C ( 102 .6°F). Physica l examination revea ls increased head ci rcumference and prominent
15 hepatosplenomegaly. Screen ing laboratory tests revea l a profound anem ia and leukopen ia. Mastoid and
para nasal sinus malformations are also noted during the cou rse of the patient workup. Despite fluid resuscitation
16
and init iation of antibiotic therapy, the ch ild dies . On autopsy, histolog ic ana lysis of the bone marrow reveals a lack
17 of a medu llary canal, with persistence of the primary spong iosa, and deposition of dense, sclerotic bone .
18
19 Which malfunction ing cel ls are the cause of th is patient's disease process?
20
:
21 A. Hepatocytes
22
B. Ly mphoid progen itor cells
23
24
C. Osteoblasts
25 D. Osteoclasts
26 E. Reticu locytes
27
• 28
• 29
o30
• 31
• 32
• 33 ~

6
lock
s
Suspend
0
End Block
Item: 28 of 33 ~. I • M k <:] t> al ~· ~
QIO: 5092 .l. ar Previous Next lab 'lifllues Notes Calculator


13
The correct answer is D. 54°/o chose this.
14
The most like ly diagnosis for this child is osteopetrosis. This rare hereditary disorder occu rs from a failure of the
15 resorption and remodeling of bone due to ma lfunctioning osteoclasts. The skeleton becomes diffuse ly sclerotic
16 and dense as new bony matrix is laid into the medul lary cana l, replacing the hematopoietic tissue . Calcified
17 remnants of the prima ry spongiosa, wh ich is norma lly removed du r ing growth, are seen within the mature
bone. Patients compensate with extramedul lary hematopoiesis, leading to hepatosp lenomegaly. Despite the
18
increased density, the bone is brittle and pred isposed to fracture. There are two main types of the disease,
19 character ized by their inheritance patterns. The autosomal-recessive form is more aggressive and is often fatal
20 in utero or in the neonatal period. The autosomal-dominant form is usua lly benign and may be discovered
incidentally on X-ray.
21
Osteopetrosis Hepatosplenomegaly Haematopoiesis Genetic disorder Osteoclast Calcification Medullary cavity Extramedullary hematopoiesis Dominance (genetics)
22
Bone fracture Bone X-ray Sclerosis (medicine) In utero
23
24 A is not correct. 2°/o chose this.
25
Osteopetrosis presents with hepatomegaly due to the need for extramedul lary hematopoiesis, not a defect of
the intrinsic cellular makeup of the liver.
26 Osteopetrosis Extramedullary hematopoiesis Hepatomegaly Haematopoiesis liver
27
B is not correct. 12% chose this.
28
Despite find ing leukopenia in osteopetrosis, the root cause is not a defect in the lymphoid progenitor cells. Their
• 29
norma l proliferation is prevented by the filling of the intramedu llary space with bony tissue.
o30 Osteopetrosis leukopenia lymphatic system lymphoblast

• 31
C is not correct. 27% chose this .
• 32 I n osteopetrosis, the function of the osteoblasts goes unchecked by ma lfunctioning osteoclasts. Though there is
• 33 ~
an excess of bony material, osteopetrosis is not a neop lastic process. ~

6
lock
s
Suspend
0
End Block
Item: 28 of 33 ~. I • M k <:] t> al ~· ~
QIO: 5092 .l. ar Previous Next lab 'lifllues Notes Calculator


13
A is not correct. 2°/o chose this.
14
Osteopet rosis presents with hepat omega ly due to the need for ext ramedu llary hematopoiesis, not a defect of
15 the intrinsic cel lular makeup of the liver.
16 Osteopetrosis Extramedullary hematopoiesis Hepatomegaly Haematopoiesis liver

17 B is not correct. 12% chose this.


18 Despite find ing leukopenia in osteopet rosis, the root cause is not a defect in the lymphoid progen itor cel ls. Their
19 norma l proliferation is prevented by the filling of t he intramedu lla ry space with bony t issue.
Osteopetrosis leukopenia lymphatic system lymphoblast
20
21 C is not correct . 27% chose this.
22 I n osteopetrosis, the function of the osteoblasts goes unchecked by ma lfunct ioning osteoclast s. Though there is
23 an excess of bony material, osteopetrosis is not a neop last ic process .
Osteopetrosis Osteoclast Osteoblast Neoplasm
24
25 E is not correct. 5°/o chose this.
26 Patients with ost eopetrosis may often present with anemia. However, th is is not due to a defect in RBC
maturation . It is due to the crowding effect of limit ed intramedullary space.
27
Osteopetrosis Anemia Medullary cavity Red blood cell Intramedullary rod
28
• 29
o30 Bottom Line:
• 31 Osteopet rosis is due to a fa ilure of bone resorption and remodel ing secondary to ma lfunction ing osteoclasts .
Osteopetrosis Bone resorption Osteoclast Bone
• 32
• 33 ~

6
lock
s
Suspend
0
End Block
Item: 28 of 33 ~. I • M k <:] t> al ~· ~
QIO: 5092 .l. ar Previous Next lab 'lifllues Notes Calculator


13 Patients with osteopetrosis may often present with anemia. However, th is is not due to a defect in RBC
14 maturation. It is due to the crowding effect of limited intramedu llary space.
Osteopetrosis Anemia Medullary cavity Red blood cell Intramedullary rod
15
16
17 Bottom Line:
18 Osteopetrosis is due to a fai lure of bone resorption and remodeling secondary to ma lfunction ing osteoclasts.
19 Osteopetrosis Bone resorption Osteoclast Bone

20
21
22 lijl;fiiJI•J for year:[ 2017 ..
FI RST AID FAC T S
23
24 FA17 p435.3
25 Osteopetrosis Fa ilure of normal bone resorption due to defective osteoclasts --+ thickened, dense bones that are
26 prone to fracture. Defective osteoclasts cause overgrowth and sclerosis of cortical bone. Bone fills
27 marrow space --+ pancytopenia, extramedullary hematopoiesis. Mutations (eg, carbon ic anhydrase
28 II) impair abil ity of osteoclast to generate acidic environment necessary for bone resorption . X-rays
show diffuse symmetric skeletal sclerosis (bone-in-bone, "stone bone" fl.l). Can result in cranial
• 29
nerve impingement and palsies as a result of narrowed foramina. Bone marrow transplant is
o30 potentially curative as osteoclasts are derived from monocytes .
• 31
• 32
• 33 ~

6
lock
s
Suspend
0
End Block
Item: 29 of 33 ~. I • M k <:] t> al ~· ~
QIO: 4435 .l. ar Previous Next lab 'lifllues Notes Calculator


13
A 66-year-old nonsmoking man presents to his primary care physician with 2 months of back pa in. He denies I"' Aj
14 a history of t rauma and cannot recal l straining his back at any point. Further history reveals no pain on A
15 walking or stand ing for long periods . The pain does not wake him from sleep at night. He den ies recent
weight loss, night sweats, and fevers. He has no hesitancy, frequency, or pain on urination. Physical examination
16
revea ls a negative straight leg raise test and age- appropriate range of motion . Of note, the patient reports
17 recently diminished auditory acuity. The X-ray of the pelv is, shown in the image, high lights enlarged bone with
18 th ick coarsened cortices and cancellous bone .
19
20
21
22
23
24
25
26
27
28
• 29
o30
• 31
• 32
• 33 ~

6
lock
s
Suspend
0
End Block
13
14
15
16
17
18
19
20
21
22
I mage courtesy of William Scott, MD
23
24 Which of the following sets of laboratory values wou ld be expected in this patient?
25
:
26 A. Decreased calcium, increased phosphate, elevated alkaline phosphatase
27
B. Elevated calcium, decreased phosphate, elevated alka line phosphatase
28
• 29 C. Elevated ca lcium, elevated phosphate, no rma l alkaline phosphatase
• 30 D. Norma l calcium, normal phosphate, elevated al kaline phosphatase
. 31
E. Norm al calcium, normal phosphate, norm al al kaline phosphatase
. 32
. 33 •

a
Lock
s
Suspend
8
End Block
Item: 29 of 33 ~. I • M k <:] t> al ~· ~
QIO: 4435 .l. ar Previous Next lab 'lifllues Notes Calculator


13
The correct answer is D. 47°/o chose this.
14
This patient displays many of the symptoms of Paget disease of bone. Often
15 idiopathic, th is disease is characterized by extensive remode ling of bone that
16 resu lts in drastica lly increased bone mass. In fact, this patient's diminished
17
hearing like ly results from th is patho log ic remodeling in his ea r. Despite
increased bone mass, these patients are at increased r isk of fractures and
18 bone cancers, often sarcomas . Histologic examination revea ls th ickened
19 trabecu lae with a "mosaic" pattern, as shown in the image. The classic lab
20 finding for Paget disease is an iso lated elevation in alkal ine phosphatase.
Calcium levels are sometimes elevated, but more commonly wi ll be with in
21
norma l limits. I t is important to note that Paget disease of the bone and
22 osteopetrosis are two diseases that can man ifest in a very similar fashion: Image courtesy of Dr. Michael
23 both affect predominantly bones in the spine, pelvis, sku ll, or legs. Lab Bonert
24
findings in osteopetrosis may be similar but are more var iab le, with
normal/low calcium; normal/elevated parathyroid hormone, vitamin D, and alkaline phosphatase; and elevated
25 osteoclast-derived tartrate-resistant acid phosphatase (TRAP). The key differentiating character istic is age of
26 onset. Osteopetrosis generally will occur in chi ldren and adolescents or, rarely, infants. Paget disease usua lly
27 occurs in midd le-aged or older patients .
Osteopetrosis Paget' s disease of bone Parathyroid hormone Alkaline phosphatase Vitamin D Acid phosphatase Pelvis Idiopathy Parathyroid gland Alkalinity
28
Histology Sarcoma Hormone Trabecula Calcium Pathology Bone Phosphatase
29
o30 A is not correct. 17% chose this.
• 31 This set of laboratory values is consistent with rena l insufficiency resulting in decreased vitamin D production,
which leads to less intestinal absorption of calcium . This produces a state of secondary hyperparathyroidism as
• 32
the body promotes reabsorption of bone in an attempt to increase serum calcium levels to normal. Even though
• 33 ~ n;1r;1thvroirl hnrmnnP. IP.vP.Ic; nromntP. nhnc;nh;1tP. P.X<.rP.tinn hv thP. k irlnP.vc;. hvnF>rnhnc;nh;1tP.mi;1 nr:<.11rc; rl11P. tn
6
lock
s
Suspend
0
End Block
Item: 29 of 33 ~. I • M k <:] t> al ~· ~
QIO: 4435 .l. ar Previous Next lab 'lifllues Notes Calculator


13 A is not correct. 17% chose this.
14 This set of laboratory values is consistent with rena l insufficiency resulting in decreased vitamin D production,
15 which leads to less intestinal absorption of calcium . This produces a state of secondary hyperparathyroidism as
16
the body promotes reabsorption of bone in an attempt to increase serum calcium levels to normal. Even though
parathyroid hormone levels promote phosphate excretion by the kidneys, hyperphosphatemia occu rs due to
17 decreased renal function. Bone-specific alkaline phosphatase (BSAP) leve ls are elevated because of increased
18 osteob lastic activity and bone formation.
Hyperphosphatemia Parathyroid hormone Alkaline phosphatase Hyperparathyroidism Vitamin 0 Calcium Secondary hyperparathyroidism Renal function Osteoblast
19
Phosphate Vitamin Kidney Blood plasma Alkalinity Ossification Hormone Parathyroid gland Serum (blood) Bone Serum calcium Excretion
20
21 B is not correct. 17% chose this.
22 Elevated calcium, low phosphate, and elevated alkaline phosphatase would be typical of primary
23 hyperparathyroidism. Elevated levels of the parathyroid hor mone (PTH) lead to increased bone breakdown,
which increases serum levels of calcium and phosphate . However, PTH activity at the level of the kidneys leads
24
to ca lcium reabsorption and phosphate secretion . Bone-specific alkal ine phosphatase (BSAP) levels are elevated
25 because of increased osteoblastic activity and bone formation .
26 Parathyroid hormone Alkaline phosphatase Hyperparathyroidism Primary hyperparathyroidism Calcium Phosphate Parathyroid gland Osteoblast Alkalinity

27 Blood plasma Ossification Hormone Phosphatase Kidney Bone Serum (blood) Secretion

28 C is not correct. 8°/o chose this.


29 Vitamin D intoxication can result in elevated calcium and phosphate levels but norma l alka line phosphatase.
o30 Vitamin D promotes renal and intestina l absorption of ca lcium but also decreases renal excretion of
• 31 phosphate. Indirectly, by raising calcium and phosphate levels, bone for mation is favored. I n ch ron ic cases of
hypervitaminosis D, however, the di rect effects of the vitamin on bone actual ly promote bone reabsoprtion,
• 32
which may lead to an elevated alkaline phosphate level.
• 33 ~
Alkaline phosphatase Vitamin 0 Hypervitaminosis 0 Calcium Vitamin Phosphate Bone Kidney Alkalinity Excretion

6
lock
s
Suspend
0
End Block
Item: 29 of 33 ~. I • M k <:] t> al ~· ~
QIO: 4435 .l. ar Previous Next lab 'lifllues Notes Calculator
- - - - - - -- -
- - - - - - - --
13 to ca lcium reabsorption and phosphate secretion . Bone-specific alkal ine phosphatase (BSAP) levels are elevated
14 because of increased osteoblastic activity and bone formation .
Parathyroid hormone Alkaline phosphatase Hyperparathyroidism Primary hyperparathyroidism Calcium Phosphate Parathyroid gland Osteoblast Alkalinity
15
Blood plasma Ossification Hormone Phosphatase Kidney Bone Serum (blood) Secretion
16
17 C is not correct. 8°/o chose this.
18 Vitamin D intoxication can result in elevated calcium and phosphate levels but norma l alka line phosphatase.
19
Vitamin D promotes renal and intestina l absorption of ca lcium but also decreases renal excretion of
phosphate. Indirectly, by raising calcium and phosphate levels, bone for mation is favored. I n ch ron ic cases of
20 hypervitaminosis D, however, the di rect eff ects of the vitamin on bone actual ly promote bone reabsoprtion,
21 which may lead to an elevated alkal ine phosphate level.
Alkaline phosphatase Vitamin 0 Hypervitaminosis 0 Calcium Vitamin Phosphate Bone Kidney Alkalinity Excretion
22
23 E is not correct. 11% chose this.
24 A patient with primary osteoporosis can have norma l laboratory f indings. Decreased bone mass occu rs not
25 because of a deficiency in nutr ients, minerals, or hormones, but mainly because of an imbalance in the activity
of osteob lasts and osteoclasts. This is often a resu lt of age-related changes, genetic factors, and, in women,
26
postmenopausal states.
27 Osteoporosis Osteoclast Osteoblast Postmenopausal Menopause Hormone Bone

28
29
Bottom Line:
o30
Paget disease of the bone has both osteolytic and osteosclerotic stages, resulting in increased bone mass and
• 31
a mosaic pattern of lamellar bone. Paradoxical ly, th is heavy bone is more prone to fractu re .
• 32 Osteosclerosis Bone Paget' s disease of bone Osteolysis Bone fracture

• 33 ~

6
lock
s
Suspend
0
End Block
Item: 29 of 33 ~ 1 • M k -<:J 1>- Jil ~· !:';-~
QIO: 4435 ..L ar Pre v ious Next Lab fli!ltues Not es Calcula t o r
A A

13
FA17 p436.2
14
15 Paget disease of bone Common, localized d isorder of hone Hat size can be increased due to skull
(osteitis deformans) remodeling caused b) t osteoclast ie act i\ it y thickening fJ; hearing loss is common due to
16
followed by t osteoblastic activity that forms aud itory foramen narrowing.
17
poor-quality bone. eru m Ca 2+, phosphorus, Stages of Paget disease:
18 and PTH b ·els are normal. t A I.P. \ losaic L} tic-osteoclasts
19 pattern of wO\en and lamellar bone (ost eoc~ tes \1 ixed -osteoclasts + osteoblasts
20 within lacu nae in chaotic juxtapositions); long Sclerotic-osteoblasts
bone chalk-stick frac tures. t blood Aow from Q uiescent-minimal osteoclast/osteoblast
21
t arterio\·enous shunts may cause high-out put act i\·it y
22
heart failure. t risk of osteogenic sarcoma. Treatment: bisphosphonates.
23
24 FA17 p437.1
25 Lab values in bone disorders
26 DISORDER SERUM Cal+ ALP PTH COMMENTS

27 Osteoporosis l bone mass


28 Osteopetrosis - I! Dense, brittle bones. Ca 2+ l in severe,
29 malignant disease
• 30 Paget disease of bone t Abnormal "mosaic" bone architecture
• 31 Osteitis fibrosa cystica "Brown tumors" due to fibrous replacement of
• 32 bone, subperiosteal thinning
Primary t t t Idiopathic or parath) roid h) perplasia, adenoma,
• 33 L.- ·-----*'---· -: ~: --
a
Lock
s
Suspend
8
End Bl ock
Item: 29 of 33 ~ 1 • M k -<:J 1>- Jil ~· !:';-~
QIO: 4435 ..L ar Pre v ious Next Lab fli!ltues Not es Calcula t o r
A
--.- heart fa.Iure. f n sk of ostcogentc sa rcoma. I reatment: btsphosphonates. A

13
14
FA17 p 437.1
15
lab values in b o ne disorders
16 DISORDER SERUMCal~ PO J- ALP PTH COMMENTS
17 Osteoporosis ! bone mass
18
Osteopetrosis - II Dense, brittle bones. Cal• l in severe,
19 malignant disease
20 Paget disease of bone t Abnormal "mosaic" bone architecture
21
Osteitis fibrosa cystica ''Brown tumors" due to fibrous replacement of
22 bone, subperiosteal thinning
23 Primary t t t Idiopathic or paralh) roid h) perplasia, adenoma,
24 hyperparathyroidism carcmoma
25 Secondary t t t Orten as compensation for CKD (! P0 4 1
26
hyperparathyroidism excretio n and production of activated
vitamin D)
27
Osteomalacia/rickets t t Soft bones; vitamin D deficiency also causes 2°
28
hyperparathyroidism
29
Hypervitaminosis D t t Caused by oversupplementation or
• 30
granulomatous disease (eg, sarcoidosis)
. 31
. 32
. 33 •

a
Lock
s
Suspend
8
End Bl ock
Item: 30 of 33 ~ 1 • M k -<:J 1>- Jil ~· !:';-~
QIO: 4053 ..L ar Pre v ious Next Lab fli!ltues Notes Calcula t o r

IAA]
A A

13
A 32-year-old homeless man presents to the emergency department w ith multiple fractures after falling
14 down a flight of stai rs. Rad iographs indicate that some of these fractures actually occurred before the fall,
15 and that some of the fractures may be pathologic . Laboratory studies show low levels of vitamin D and
16
phosphate, a low-norma l level of calcium, and an elevated level of alkaline phosphatase .

17
What type of histologic finding would corroborate this patient's most likely diagnosis?
18
19 :
A . Areas of increased unmineralized osteoid adjacent to normal trabeculae
20
21 B. Hypoproliferation of osteocytes in cortical bone
22 C. Lack of a medullary canal and decreased numbers of osteoclasts
23
D. Neutrophi lic infiltration and necrotic changes in trabeculae
24
25
E. Sheets of bone matrix displacing normal bone structure, w ith bizarre large cells
26
27
28
29
o30
. 31
. 32
. 33 •

a
Lock
s
Suspend
8
End Bl ock
Item: 30 of 33 ~. I • M k <:] t> al ~· ~
QIO: 4053 .l. ar Previous Next lab 'lifllues Notes Calculator


13 The correct answer is A. 62°/o chose this.
14 Osteomalacia is a disorder of bone minera lization in adults (image indicates
15 chondroid-like mater ial). New matrix is synthesized, but there is an inabi lity
16
to minera lize it . This is in contrast to osteoporosis, in which there is reduced
synthesis of new bone matrix. Osteoma lacia patients often have severely
17 P.la&ma cell
reduced bone densities and are prone to patho log ic fractures. The most .¥
18 common cause of osteoma lacia is vitamin D deficiency, which resu lts in
19 hypocalcemia . Worldwide, inadequate dietary intake is the most common
cause of vitamin D deficiency. The lack of calcium and phosphate causes
Chonooid-4ike ~s
20
21
osteocytes to produce less mineral ized bone surrounding the ce lls. The
common lab profi le in vitamin D deficiency is low leve ls of vitamin D and •
\
22 phos~hate, with a low or ~o rmal level of ca l.cium, depending on compensation Image copyright 2009 Multimed
23 from 1ncreased parathyroid hormone secret1on, and an elevated level of Inc
alka line phosphatase. Other causes of vitamin D deficiency include insufficient ·
24
sunlight exposure (especial ly in dark-skinned people), renal tubu lar acidosis,
25 ma labsorption in pregnancy, and chronic renal fai lure. Of note, vitamin D deficiency in ch ildren instead produces
26 r ickets.
Osteoporosis Hypocalcaemia Osteomalacia Rickets Parathyroid hormone Alkaline phosphatase Renal tubular acidosis Chronic kidney disease Malabsorption
27
Vitamin 0 Calcium Bone Parathyroid gland Hypovitaminosis 0 Acidosis Osteocyte Hormone Alkalinity Mineralization (biology) Pregnancy Phosphate Kidney
28
Vitamin Osteon
29
30 B is not correct. 10% chose this.
• 31 The pro liferative status of osteocytes is not greatly decreased in osteomalacia. Mineral ization deficiencies, in this
• 32
case, occur as a resu lt of the decreased ava ilability in necessary vitamins, not deficiencies in the quantitative or
qual itative status of cel ls invo lved the functiona l maintenance of bone, name ly osteocytes .
• 33 ~ Oc:tP.om;ll~c-i~ MinP.r~li7::ltion (hiolonv) RonP. Oc:tP.oc-vtP.

6
lock
s
Suspend
0
End Block
t>
Item: 30 of 33
QIO: 4053

13
~. I
.l.
:
• M
ar
k <:]
..
Previous Next
al
lab 'lifllues

Notes
~
Calculator

The pro liferative status of osteocytes is not greatly decreased in osteomalacia. Mineral ization deficiencies, in this
14
case, occur as a resu lt of the decreased ava ilability in necessary vitamins, not deficiencies in the quantitative or
15 qual itative status of cel ls involved the functiona l maintenance of bone, name ly osteocytes.
16 Osteomalacia Mineralization (biology) Bone Osteocyte

17 C is not correct. 10% chose this.


18 I n osteopetrosis, a col lection of rare genetic diseases, osteoclast function is deficient, leading to th ickened bones
19 lacking medul lary cavities; these bones are brittle. The number of osteoclasts may be reduced or increased
depend ing on the variant. Vitamin D levels wou ld not be decreased.
20
Osteoclast Osteopetrosis Medullary cavity Vitamin D Disease
21
22
D is not correct. 5°/o chose this.
Osteomye litis is an infection of the bone. I t is most common ly caused by Staphylococcus aureus and displays
23
neutroph ils and bone necrosis on microscopic examination. Infection with Mycobacterium tuberculosis can cause
24 a monocytic predominance . Vitamin D levels wou ld not be decreased .
25 Osteomyelitis Mycobacterium tuberculosis Vitamin D Staphylococcus aureus Tuberculosis Neutrophil Staphylococcus Necrosis Bone Mycobacterium Monocyte

26 Vitamin Infection

27
E is not correct. 13% chose this.
28 Osteosarcoma is the most common primary mal ignancy of bone. It is characterized by the for mation of new
29 bone matrix. I t most common ly arises in the metaphysis of long bones . Vitamin D levels would not be
30 decreased .
Osteosarcoma Metaphysis Vitamin D Malignancy Cancer Vitamin Bone long bone
• 31
• 32

• 33 Bottom Line:

6
lock
s
Suspend
0
End Block
Item: 30 of 33 ~. I • M k <:] t> al ~· ~
QIO: 4053 .l. ar Previous Next lab 'lifllues Notes Calculator


13 C is not correct. 10% chose this .
14 I n osteopetrosis, a col lection of rare genetic diseases, osteoclast function is deficient, leading t o t hickened bones
15
lacking medul lary cavit ies; these bones are brittle. The number of osteoclast s may be reduced or increased
depend ing on the variant . Vit am in D levels wou ld not be decreased.
16 Osteoclast Osteopetrosis Medullary cavity Vitamin D Disease
17
D is not correct. 5°/o chose this.
18
Osteomye lit is is an infect ion of the bone. I t is most common ly caused by Staphylococcus aureus and displays
19 neutroph ils and bone necrosis on microscopic examinat ion. Infection with Mycobacterium tuberculosis can cause
20 a monocyt ic predominance . Vitamin D levels wou ld not be decreased .
Osteomyelitis Mycobacterium tuberculosis Vitamin D Staphylococcus aureus Tuberculosis Neutrophil Staphylococcus Necrosis Bone Mycobacterium Monocyte
21
Vitamin Infection
22
23 E is not correct. 13% chose this.
24 Osteosa rcoma is the most common primary ma lignancy of bone. It is characterized by the for mation of new
25 bone mat rix. I t most common ly arises in the metaphysis of long bones . Vitamin D levels would not be
decreased .
26
Osteosarcoma Metaphysis Vitamin D Malignancy Cancer Vitamin Bone long bone
27
28
29 Bottom Line:
30 Osteoma lacia is found in ma lnourished adu lts with vitamin D and calcium deficiencies. It may present with
• 31
pathologic fract ures or bone pa ins and a waddl ing gait .
Osteomalacia Vitamin D Vitamin Malnutrition Bone Calcium Hypocalcaemia
• 32
• 33 ~

6
lock
s
Suspend
0
End Block
Item: 30 of 33 ~. I • M k <:] t> al ~· ~
QIO: 4053 .l. ar Previous Next lab 'lifllues Notes Calculator


13 Osteomalacia is found in malnourished adults with vitamin D and calcium deficiencies. It may present with
14 pathologic fractu res or bone pains and a waddling gait .
Osteomalacia Vitamin D Vitamin Malnutrition Bone Calcium Hypocalcaemia
15
16
17
lijl;fiiJI•J for year:[ 2017 "
18 FI RST AID FAC T S

19
FA11 p436.1
20
21 Osteomalacia/rickets Defective mineralization of osteoid
22
(osteomalacia) or cartilaginous growth plates
(rickets, only in children). Most commonly clue
23
to vitamin D deficiency.
24 X-rays show osteopen ia and "Looser zones"
25 (pseuclofractures) in osteomalacia, epiphyseal
26 widening and metaphyseal cupping/fraying in
27
rickets. Children with rickets have pathologic
bow legs (genu va rum rJ), bead-like
28
costochondral junctions (rachitic rosary I]J),
29 craniotabes (soft skull).
30 l vitamin D - l serum CaZ+ - t PTH
• 31 secretion - l serum P0 4 3- .
• 32
Hyperactivity of osteoblasts - t ALP.
• 33 ~

6
lock
s
Suspend
0
End Block
Item: 31 of 3 3 ~ 1 • M k -<:J 1>- Jil ~· !:';-~
QIO: 4 0 54 ..L ar Pre v ious Next Lab fli!ltues Notes Calcula t o r

IAA]
A A

13
A 32-year-old man recently underwent a liver transplant and was prescribed cyclosporine to reduce the
14 chance of immune rejection. After a few weeks of taking the drug, he started experiencing acute pain,
15 tenderness, warmth, and swel ling of his right great toe. His physician takes a fluid sample from the affected
area. Exam ination of the contents reveals the presence of intracel lular crystals.
16
17
Wh ich of the following med ications could prevent future recurrence of this problem?
18
19
A. Allopurinol
20
21 B. Calcitonin
22 C. Colchicine
23
D. Furosemide
24
25
E. Warfarin

26
27
28
29
30
. 31
. 32
. 33 •

a
Lock
s
Suspend
8
End Bl ock
Item:31of33 ~. , . M k <:] t> al ~· ~
QIO: 4054 .l. ar Previous Next lab 'lifllues Notes Calculator

13
14 The correct answer is A. 81°/o chose this.
15
The main adverse reactions to cyclosporine therapy are rena l dysfunction, tremo r, hi rsutism, hypertension, and
gum hyperplasia. Cyclospo rine can cause urate nephropathy, which damages rena l tubules irreversibly, thus
16 decreasing the kidney's ability to excrete potentially toxic metabo lites. Th is patient is suffering f rom acute gouty
17 arthritis secondary to impaired renal excretion of uric acid and thus increased serum leve ls of urate, which can
18 precipitate as monosod ium urate crystals in joints. Gout classically presents with a warm, red, pa inful joint. The
most common joint affected in gout is the great toe. Other potential adverse effects of cyclosporine that occur
19
secondary to renal failure include hyperkalem ia, hypophosphatemia, hypomagnesemia, hypercalciuria, and
20 metabolic acidosis. Allopurino l is used to prevent gouty arthrit is by inhibit ing xanth ine oxidase, an enzyme
21 involved in uric acid synthesis. The mainstay of therapy for acute gout is nonsteroida l anti-inflammatory drugs
22
and corticosteroids; allopurinol is not effective in the treatment of acute gout because it does not add ress the
primary issues of pain and inflammation .
23 Xanthine oxidase Hyperkalemia Hypercalciuria Allopurinol Hypomagnesemia Metabolic acidosis Xanthine Enzyme Hirsutism Uric acid Ciclosporin Gout
24 Corticosteroid Hypertension Hypophosphatemia Nonsteroidal anti -inflammatory drug Kidney disease Kidney Tremor Blood plasma Acidosis
25 Acute uric acid nephropathy Inflammation Arthritis Anti-inflammatory Serum (blood) Precipitation (chemistry) Toe Metabolite Hyperplasia Metabolism
26
B is not correct. 2°/o chose this.
27
Calcitonin counteracts the action of parathyroid hormone, thus reducing serum calcium levels. Whereas
28
cyclosporine is known to cause hyperca lciuria, it is not associated with the formation and precipitation of
29 ca lcium pyrophosphate crystals in joints and connective t issues, also known as pseudogout. The development of
30 pseudogout is associated with joint t rauma, familial chondroca lcinosis, hemochromatosis, and certain other
metabolic and endocr ine disorders. Thus, calcitonin would not prevent acute gouty arthritis.
31
Chondrocalcinosis Parathyroid hormone Iron overload Ciclosporin Gout Calcitonin Parathyroid gland Hormone Endocrine disease Arthritis Calcium Serum (blood)
• 32
Endocrine system Metabolism Blood plasma
• 33

6
lock
s
Suspend
0
End Block
Item:31of33 ~. , . M k <:] t> al ~· ~
QIO: 4054 .l. ar Previous Next lab 'lifllues Notes Calculator


13 C is not correct. 15% chose this.
14 Colch icine inh ibits microtubu le polymerization by binding to tubul in; thus it has found use in a variety of
15 different medical conditions. Whereas it is utilized sometimes as an off-label treatment for gout, it wou ld not be
indicated in th is situation . Furthermore, colch icine is associated with significant toxicity, includ ing fever, nausea,
16
vomit ing, and kidney fai lure.
17 Microtubule Colchicine Gout Tubulin Polymerization Nausea Kidney Off-label use Vomiting Fever Toxicity

18
Dis not correct. 1°/o chose this.
19
Fu rosemide is a loop diuretic used in the treatment of congestive heart fa ilure and acute rena l fai lure. Loop
20 diuretics may be used to increase calcium diuresis in hypercalcemic states. However, loop diuretics are known
21 to cause hyper ur icemia, which may precipitate gout in some patients and should be avoided in this patient
22
specifical ly .
loop diuretic Furosemide Hyperuricemia Diuretic Heart failure Gout Congestive heart failure Acute kidney injury Forced diuresis Hypercalcaemia Polyuria Calcium
23
Precipitation (chemistry) Kidney
24
25 E is not correct. 1°/o chose this.
26 Warfarin is an anticoagu lant that inh ibits vitamin K-dependent clotting factors II, VII, IX, and X. Neither
elevated ur ic acid levels nor cyclospor ine itself predisposes patients to thrombus formation; therefore,
27
antith rombotic therapy is unnecessary.
28 Anticoagulant Warfarin Uric acid Ciclosporin Thrombus Coagulation Vitamin Antithrombotic

29
30
Bottom Line:
31
Cyclospor ine and loop diuretics may predispose a patient to acute gouty arthrit is caused by uric acid crystal
• 32
deposition in a joint space, commonly the big toe, an kle, or knee. Gout is treated acutely with nonsteroida l
• 33 ~ anti-inflammatorv druas or corticosteroids and oroohvlactical lv with alloourinol. ~

6
lock
s
Suspend
0
End Block
Item:31of33 ~. , . M k <:] t> al ~· ~
QIO: 4054 .l. ar Previous Next lab 'lifllues Notes Calculator
• - -. ·- • • - • - • - - • - -. l t • - • 'f ••• -..,- -· --

13 vomit ing, and kidney fa ilure.


14 Microtubule Colchicine Gout Tubulin Polymerization Nausea Kidney Off-label use Vomiting Fever Toxicity

15
Dis not correct. 1°/o chose this.
16 Fu rosem ide is a loop diuretic used in the treatment of congestive heart fa ilure and acute rena l fai lure. Loop
17 diuretics may be used to increase ca lcium diuresis in hypercalcemic stat es. Howeve r, loop diuretics are known
18 to cause hyper ur icemia, which may preci pitate gout in some patients and should be avoided in this patient
specifica lly .
19 loop diuretic Furosemide Hyperuricemia Diuretic Heart failure Gout Congestive heart failure Acute kidney injury Forced diuresis Hypercalcaemia Polyuria Calcium
20
Precipitation (chemistry) Kidney
21
E is not correct. 1°/o chose this.
22
Warfa rin is an ant icoagu lant that inh ibits vitamin K- dependent clotting factors II, VII, IX, and X. Neither
23
elevat ed ur ic acid levels no r cyclospor ine it self pred isposes patients to th rombus fo rmat ion; therefore,
24 antit hrombotic therapy is unnecessa ry .
25 Anticoagulant Warfarin Uric acid Ciclosporin Thrombus Coagulation Vitamin Antithrombotic

26
27
Bottom Line:
28
Cyclospor ine and loop diuretics may pred ispose a patient to acut e gouty arthrit is caused by uric acid crystal
29 deposit ion in a jo int space, commonly t he big t oe, an kle, or knee. Gout is treat ed acutely wit h nonsteroida l
30 anti-inflammato ry drugs or corticosteroids and prophylactical ly wit h allopurinol.
Allopurinol Uric acid Ciclosporin Gout Diuretic Corticosteroid Nonsteroidal anti-inflammatory drug loop diuretic Anti-inflammatory Preventive healthcare Arthritis
31
Hallux Toe
• 32
• 33 ~

6
lock
s
Suspend
0
End Block
Item: 31 of 3 3 ~ 1 • M k -<:J 1>- Jil ~· !:';-~
QIO: 4 0 54 ..L ar Pre v ious Next Lab fli!ltues Notes Calcula t o r
A A

13
FA17 p440.1
14
Gout
15
FINDINGS Acute inflammatory monoarthrit is caused by precipitation of monosodium urate crystals in
16 joints · . More common in males. Associated with hypennicemia, which can be caused by:
17 Underexcretion of uric acid (90% of patients)- largely idiopathic, potentiated by renal failure;
18 can be exacerbated by certain medications (eg, thiazide diuretics).
19
0\·erproduction of uric acid ( 10% of patients)- Lesch-1\yhan syndrome, PRPP e'l:cess, t cell
turnover (eg, tumor lysis S) ndrome), von Gierke disease.
20
e
CrystaJs are needle shaped and birefr ingent under polarized light (yellow under parallel light,
21 blue under perpendicular light ).
22
SYMPTOMS Asymmetric joint distribution. Joint is swollen, red, and pa infuJ. Classic manifestation is painful
23 MTP joint of big toe (podagra). Tophus formation (often on external ear, olecranon bursa,
24 or Achilles tendon). Acute attack tends to occur after a large meal with foods rich in purines
25 (eg, red meat, seafood), trauma, surgery, dehydration, diuresis, or alcohol consumption (alcohol
metabol ites compete for same excretion sites in kidney as uric acid -+ l uric acid secretion and
26
subsequent buildup in blood).
27
TREATMENT Acute: SA!Ds (eg, indomethacin), glucocort icoids, colchicine.
28
Chronic (preventive): xa nthine oxidase inhibitors (eg, allopurinol, febuxostal).
29
30
31
• 32

. 33

a
Lock
s
Suspend
8
End Bl ock
Item: 32 of 33 ~ 1 • M k -<:J 1>- Jil ~· !:';-~
QIO: 4691 ..L ar Pre v ious Next Lab fli!ltues Not es Calcula t o r
A A

13
A 15-year-old boy presents to the neurologist because of increased difficu lty with maintaining his balance
14 and normal gait. He states that he has been "feeling weaker" than normal, especial ly when trying to climb
15 stairs, rise from a chair, or brush his teeth. His mother reports that one of his maternal uncles also had
16
similar symptoms that developed during the teenage years. Physica l examination reveals the patient has d ifficulty
rising to a standing position from sitting in a chair. Laboratory studies reveal a serum creatine kinase level of
17 13,021 IU/ L.
18
19 What type of genetic mutation is most likely responsible for the patient's symptoms?
20
:
21 A. In-frame deletion
22
B. Maternal imprinting
23
24
C. Out-of-frame deletion
25 D. Paterna l imprinting
26 E. Trinucleotide repeat
27
28
29
30
31
. 32
. 33 •

a
Lock
s
Suspend
8
End Bl ock
Item: 32 of 33 ~. I • M k <:] t> al ~· ~
QIO: 4691 .l. ar Previous Next lab 'lifllues Notes Calculator


13
The correct answer is A. 49°/o chose this.
14
This patient suffers from Becker muscular dystrophy (BMD), which is a mi lder form of Duchenne muscu lar
15 dystrophy (DMD). Patients with BMD frequently present with muscle weakness and/or card iac disease beginning
16 in mid-ado lescence, but do not routinely have the intellectua l impair ments associated with DMD . Patients with
17 BMD also survive longer, usua lly th rough the th ird decade. Although both BMD and DMD are a consequence of
abnorma l or absent dystrophin protein, respectively, the genetics of each are unique. In BMD, in-frame
18
de letions (wh ich maintain the reading frame du r ing the translation process) in the dystroph in gene result in
19 abnorma l dystrophin protein that retains some level of function. In DMD, out-of-frame de letions of the
20 dystrophin gene resu lt in severely truncated or absent dystroph in protein, resulting in a much more severe
phenotype.
21
Dystrophin Duchenne muscular dystrophy Becker' s muscular dystrophy Gene Muscular dystrophy Phenotype Protein Muscle weakness Muscle Reading frame
22
Genetics
23
24 B is not correct. 7°/o chose this.
25
Maternal imprinting resu lts from the inactivation (methylation) of the imprinted al lele; an example is Angel man
synd rome . Clinical features include severe intellectua l disability with limited verba l communication, motor
26 reta rdation, ataxia, seizures, and unusual facies with a large mandible and open-mouthed expression of the
27 tongue . Patients often have an easily excitable persona lity, often laughing or smiling, and typical ly have
28 tremulous movements of the limbs, as if they were a real-l ife marionette.
Angelman syndrome Allele Ataxia Intellectual disability Methylation Genomic imprinting Epileptic seizure Mandible Imprinting (psychology)
29
30 C is not correct. 22% chose this.
31 Out-of-frame de letions often result in prematu re termination of protein translation, resulting in nonfunctiona l or
absent protein product. A notable examp le is Duchenne muscu lar dystrophy, wh ich is character ized by muscular
32
dystrophy with a much earl ier onset with a much more rapid progression than Becker dystor phy . Moreover,
• 33 ~ oatients often do not live to verv advanced aaes . ~

6
lock
s
Suspend
0
End Block
Item: 32 of 33 ~. I • M k <:] t> al ~· ~
QIO: 4691 .l. ar Previous Next lab 'lifllues Notes Calculator


13 C is not correct. 22% chose this.
14 Out-of-frame de letions often result in prematu re termination of protein translation, resulting in nonfunctiona l or
15 absent protein product. A notable examp le is Duchenne muscu lar dystrophy, wh ich is character ized by muscular
dystrophy with a much earl ier onset with a much more rapid progression than Becker dystor phy . Moreover,
16
patients often do not live to very advanced ages.
17 Duchenne muscular dystrophy Protein Protein biosynthesis Muscular dystrophy Translation (biology) Deletion (genetics)

18
D is not correct. 3°/o chose this.
19
Paterna l imprinting resu lts from the inactivation (methylation) of the imprinted al lele; an example is Prader-W illi
20 synd rome, which is cha racterized by inte llectual disability, hyperphag ia, obesity, hypogonadotropic
21 hypogonadism, hypotonia, and smal l stature with sma ll hands and feet.
Prader-Willi syndrome Allele Polyphagia Hypogonadism Hypotonia Intellectual disability Methylation Genomic imprinting Obesity
22
23 E is not correct. 19% chose this.
24 Diseases with trinucleotide repeats may demonstrate anticipation (increased severity of disease with subsequent
generations) . One exampleis Huntington disease, cha racterized by motor difficu lt ies, notably chorea, and
25
cognitive decl ine; another is fragile X syndrome, character ized by intellectua l disabi lity; macroorchidism; and
26 distinct facies, notably a long face, large ears, and prominent jaw.
27 Fragile X syndrome Huntington' s disease Macroorchidism Intellectual disability Chorea Trinucleotide repeat disorder

28
29
Bottom Line:
30
Becker muscula r dystrophy (BMD) manifests with muscle weakness or ca rdiac disease or both beginning in
31 ado lescence, but patients do not routinely have the intellectua l impai r ments. Both BMD and Duchenne
32 muscu lar dystrophy are a consequence of abnorma l (in-frame de letion) or absent (out-of-frame deletion)
• 33 ~
dystrophin protein .

6
lock
s
Suspend
0
End Block
Item: 32 of 33 ~ 1 • M k -<:J 1>- Jil ~· !:';-~
QIO: 4691 ..L ar Pre v ious Next Lab fli!ltues Not es Calcula t o r
A A

13 FA17 p 57. 1
14 Muscular dystrophies
15 Duchenne X-linked disorder typically due to framcshift Duchenne = deleted dystrophin.
16 or nonsense mutations - truncated or Oystrophin gene (DMD) is the largest
17
absent dystrophin protein - progressive protein-coding human gene - f chance of
myofiber damage. Weakness begins in pelvic ~ponlaneous mutation. Dystrophin helps
18
girdle muscles and progresses superiorly. anchor muscle fibers, primarily in skeletal and
19 cardiac muscle. It connects the intracellular
Pseudohypertrophy of calf muscles due to
20 •·..-rl• fibrofatty replacement of muscle . \ addling crtoskeleton (actin) to the transmembrane
21 . D
gait. Onset before ; years of age. Dilated proteins a- and ~-dystroglycan, which are
22
cardiomyopathy is common cause of death . connected to the extracellular matrix (ECr.. l).
l.oss of dystrophin results in myonecrosis.
23
f CK and aldolase are seen; genetic testing
24
confirms diagnosis.
25
Becker X-linked disorder typically due to 110 11- Deletions can cause both Duchennc and
26 frameshift deletions in dystrophin gene Becker muscular dystrophies. Y, of cases have
27 (partially functional instead of trun ct~ ted ). Less large deletions spanning one or more exons.
28 severe than Duchcnnc. Onset in adolescence
29
or early adulthood.

30
Myotonic type 1 Autosomal dominant. C'fG trinucleotide repeat C ataracts, Toupee (early balding in men),
expansion in the DMPK gene - abnormal Gonadal atroph)'.
31
expression of myotonin protein kinase
32 - myotonia, muscle wasting, cataracts,
. 33 testicular atroplw. frontal balding, arrhvthmia .
a
Lock
s
Suspend
8
End Bl ock
Item: 32 of 33 ~ 1 • M k -<:J 1>- Jil ~· !:';-~
QIO: 4691 ..L ar Pre v ious Next Lab fli!ltues Not es Calcula t o r
A
,
A

13 Myotonic type 1 Autosomal dominant. CTG trinucleotide repeal Cataracts, Toupee (early balding in men),
14 expansion in the DMPK gene ... abnormal Gonadal atrophy.
15 expression of myotonin protein kinase
16 ... myotonia, muscle wasting, cataracts,
testicular atrophy, frontal balding, arrhythmia.
17
Gower sign-patient uses upper e>.tremit ies to
18
help stand up.
19
Classically seen in Duchenne muscular
20 dystrophy, but also seen in other muscular
21 dystrophies and inAammatory lll}Opathies (eg,
22 polymyositis). lonlcM~ )
23
24
25
26

27 FA17 p 54 .1
28
Imprinting At some loci, only one allele is act ive; the Both Prader-\Villi and Angelman syndromes
29 other is inactive (imprinted/inactivated b) are due to mutation or deletion of genes on
30 methylation). With one allele inactivat·ed, chromosome I 5.
31 deletion of the acti,·e allele ... disease.
32 Prader-Willi syndrome latemal imprinting: gene from mom is normally 25% of cases due to maternal uniparental
silent and Paternal gene is deleted/mutated. disomy (two maternally imprinted genes arc
. 33 . .. .
a
Lock
s
Suspend
8
End Bl ock
Item: 32 of 33 ~ 1 • M k -<:J 1>- Jil ~· !:';-~
QIO: 4691 ..L ar Pre v ious Next Lab fli!ltues Not es Calcula t o r

13
A
•. - - • •

methylation). With one allele inactivated,


- ~

chromosome I 5.
- + - --
- · ~
A

14 deletion of the active allele - disease.


15 Prader-Willi syndrome latemal imprinting: gene from mom is normally 25% of cases due to maternal uniparental
16 silent and Paternal gene is deleted/mutated. disomy (two maternally imprinted genes arc
Results in hyperphagia, obesity, intellectual received; no paternal gene received).
17
disability, hypogonadism, and hypotonia.
18
AngeiMan syndrome Paternal imprinting: gene from dad is normally 5% of cases due to paternal uniparental disomy
19
silent and \1aternal gene is deleted/mutated. (two paternally imprinted genes are received; no
20 Results in inappropriate laughter ("happ) maternal gene received).
21 puppet~), seizures, ataxia, and severe
22 intellectual disability.
23
24 FA17 p 58.2

25 Trinucleotide repeat Huntington disease, myotonic dystrophy, Try (trinucleotide) hunting for my fragile cage-
26 expansion diseases fragile X syndrome, and Friedrcieh ataxia. free eggs (X).
27 ~ lay show genetic anticipation (disease severity
t and age of onset l in successive generations).
28
Huntington disease = (CAG)11 Cmtdate has l ACh and GABA
29 Iyotonic dystrophy = (CTG)11 Cataracts, Toupee (early balding in men),
30 G onadal atrophy
31 Fragile X syndrome = (CCC)11 C hin (protmding), Giant Gonads
32 Friedreich ataxia = (GAA)11 Ataxic G \\it
. 33
-
a
Lock
s
Suspend
8
End Bl ock
Item: 33 of 33 ~. I • M k <:] t> al ~· ~
QIO: 3483 .l. ar Previous Next lab 'lifllues Notes Calculator


13
A 52-year-old man presents to his physician for an annual examination . Wh ile discussing blood work that was I"' Aj
14 done the week before the appointment, the physician notes a low- density lipoprotein cholesterol level that is A
15 above the normal limit . Additional ly, the patient's blood pressure is in the hypertensive range. The patient
says that his father died from a heart attack at the age of 55 years. The physician prescribes a med ication to the
16
patient to reduce his r isk of having a myocard ial infarction. 3 months later, the patient presents to the emergency
17 department with complaints of severe muscle aches and a darken ing of his urine. Laboratory stud ies subsequently
18 show the patient's blood urea nit rogen level is 20 mg/dl and creatinine level is 1.4 mg/dl. Dipstick is positive for
19 blood, but microscopy reveals no RBCs .
20
21
What is the most likely cause of the patient's creatinine level?
22 :

23
A. Acute tubular necrosis
24 B. Ben ign prostatic hyperplasia
25 C. I nterstitial nephritis
26
D . Rena l artery obstruction
27
28 E. Uretera l obstruction
29
30
31
32
• 33 ~

6
lock
s
Suspend
0
End Block
Item: 33 of 33 ~ 1 • M k -<:J 1>- Jil ~· !:';-~
QIO: 3483 ..L ar Pre v ious Next Lab fli!ltues Notes Calcula t o r

13
A
The correct answer is A. 60°/o chose this. A

14 The patient has an elevat ed low- density lipoprotein level, which increases his
risk of developing atherosclerosis and subsequent myocardial infarction. The
15
patient was prescribed a statin, wh ich acts by inh ibiting 3-hydroxy-3-methyl- .,
16 glutaryi-CoA reductase. Although rare, one adverse effect of statins is • __ - .• ~ ~~! r- • ~ Q .
17 rhabdomyolysis, which results in myalgias and the presence of elevated levels - ~S ~~ ~ ~- ~ ·
of myoglobin in the blood. Myog lobin is then filtered by the kidney, where it ~ ·~~~o •
18
19
acts as a nephrotoxin and can cause acute tubular necrosis (ATN ), explaining a~ ~~ ·
the patient's elevated BUN and creatinine. Routine urinalysis dipsticks can
20 report the presence of "blood " in the urine, but what these dipsticks actually
21 measure is heme. Heme groups may come from hemoglobin in RBCs, but
false positives for "blood" in urine can result from myoglobinuria. One reason
22
for m icroscopy after a positive dipst ick test for blood is to confirm that RBCs, Image copyright © Indian
23 and not myoglobin, are responsible for the abnormal result . (The image Journal of Nephrology
24 shown here demonstrates intrat ubu lar casts.) The presence of blood in the
25
urine with no RBCs visua lized on microscopy should raise concern for myoglobinuria.
Statin Nephrotoxicity Rhabdomyolysis Myoglobin Atherosclerosis Myoglobinuria Myocardial infarction Acute tubular necrosis Hemoglobin Low-density lipoprotein
26
Creatinine Urinalysis Heme Hematuria Urine Kidney Blood urea nitrogen Necrosis Lipoprotein Microscopy Adverse effect
27
28 B is not correct. 3°/o chose this.
29 Ben ign prostatic hyperplasia is a post renal cause of acute renal fai lure. I t increases in incidence w ith age, but it
is not associated with the initiat ion of st atin therapy.
30 Statin Benign prostatic hyperplasia Acute kidney injury Hyperplasia Benignity Kidney
31
C is not correct. 25°/o chose this.
32
I nterstitial nephritis is an intrinsic cause of acute renal failure, and it is most often associated w ith al lerg ic
33 ~ ,...O"::I,...~i,•u"'\r> .. " ,.l,..,,,r '"""l"'t.C"'f-ol'"'l"'\i,..,~l "!ll"'\ .. i _inFI"!l~~"!l rn,...,, ,..,,.., ,,r- rlit ,,..o.. irr ":lt"'\,..1 ~t''lrof-.ihi"r i,...r\ i nFor .. i nt""\r nr- "::ot"'\ inFilf•,..-:. ta i\to

a
Lock
s
Suspend
8
End Bl ock
Item: 33 of 33 ~. I • M k <:] t> al ~· ~
QIO: 3483 .l. ar Previous Next lab 'lifllues Notes Calculator
..........., •• _.._ ... '::1'' ,.. .............., ... ''11"" ... '1""' ...... ' ... , , ........... ·~ .......... , '''J""'I ''11"" ... '1""' ...... ' ... _. ...... '::1'"""1 ............. ,
• •
13
C is not correct. 25% chose this.
14
Interstitial nephritis is an intrinsic cause of acute renal failure, and it is most often associated with al lerg ic
15
reactions to drugs (nonsteroidal anti-inflammatory drugs, diuretics, and antibiotics), infections, or an infiltrative
16 process. Statin use is more commonly associated with r habdomyolysis than interstitial nephritis as an adverse
17 effect. The presence of ur inary eosinophi ls is associated with interstitia l nephritis and is commonly tested.
Statin Rhabdomyolysis Interstitial nephritis Nephritis Acute kidney injury Eosinophil Nonsteroidal anti-inflammatory drug Diuretic Anti-inflammatory Antibiotics
18
Allergy Interstitial fluid Kidney Adverse effect
19
20 D is not correct. 8°/o chose this.
21 Rena l artery obstruction is a prerenal cause of acute renal failure, wh ich will man ifest with simila r blood urea
22
nitrogen and creatinine findings as those of this patient. However, myalgia is not a common symptom
associated with renal artery obstruction, and renal artery obstruction is not associated with the initiation of
23 statin therapy.
24 Statin Renal artery stenosis Blood urea nitrogen Myalgia Urea Creatinine Renal artery Acute kidney injury Nitrogen Symptom Kidney

25
E is not correct. 4°/o chose this.
26
Ureteral obstruction is a rare cause of postrenal acute rena l failure. In norma l individua ls, un ilateral ureteral
27 obstruction does not result in renal failure since the other kidney is able to compensate adequately . However, in
28 the rare situation of bilateral ureteral obstruction or where a person with one function ing kidney has a ureteral
obstruction, rena l failure may occur. Causes of ureteral obstr uction include calculi, abdominopelvic tumors, scar
29
tissue/strictures, and retroperitoneal fibrosis.
30 Acute kidney injury Retroperitoneal fibrosis Retroperitoneal space Fibrosis Kidney Ureter Neoplasm Scar Calculus (medicine)

31
32
Bottom Line:
33

6
lock
s
Suspend
0
End Block
Item: 33 of 33 ~. I • M k <:] t> al ~· ~
QIO: 3483 .l. ar Previous Next lab 'lifllues Notes Calculator
- - - - --- - - -- - - - - - --- - - - - - - - - - -- - - - - - - -

13 process. Stat in use is more common ly associated wit h r habdomyolysis t han interst itial neph ritis as an adverse
14 effect. The presence of ur inary eosinophi ls is associated with interst itial nephritis and is common ly test ed .
Statin Rhabdomyolysis Interstitial nephritis Nephritis Acute kidney injury Eosinophil Nonsteroidal anti-inflammatory drug Diuretic Anti-inflammatory Antibiotics
15
Allergy Interstitial fluid Kidney Adverse effect
16
17 D is not correct. 8°/o chose this.
18 Rena l artery obstruction is a prerena l cause of acut e rena l failure, wh ich will man ifest with simi lar blood urea
nitrogen and creatinine find ings as those of t his patient . However, myalgia is not a common symptom
19
associated wit h rena l artery obst ruction, and renal artery obstruction is not associated with the init iation of
20 statin t herapy.
21 Statin Renal artery stenosis Blood urea nitrogen Myalgia Urea Creatinine Renal artery Acute kidney injury Nitrogen Symptom Kidney

22 E is not correct. 4°/o chose this.


23 Uret era l obst ruction is a rare cause of postrenal acut e rena l failure . I n norma l ind iv idua ls, un ilat era l uret eral
24 obst ruct ion does not result in renal fai lure since the other kidney is able to compensat e adequately . However, in
25 the rare situation of bilateral ureteral obstruction or where a person with one function ing kidney has a ureteral
obst ruct ion, renal failure may occur. Causes of ureteral obstr uction include calcu li, abdom inopelvic t umors, sca r
26
t issue/strictu res, and retroperit onea l fibrosis.
27 Acute kidney injury Retroperitoneal fibrosis Retroperitoneal space Fibrosis Kidney Ureter Neoplasm Scar Calculus (medicine)

28
29
Bottom Line:
30
Statin use can lead t o rhabdomyolysis and myoglobinuria, resulting in acut e tubula r necrosis.
31 Statin Rhabdomyolysis Myoglobinuria Acute tubular necrosis Necrosis
32
33 ~

6
lock
s
Suspend
0
End Block
Item: 33 of 33 ~. I • M k <:] t> al ~· ~
QIO: 3483 .l. ar Previous Next lab 'lifllues Notes Calculator

13
FA11 p 572.2
14
Acute tubular necrosis Most common cause of acute kidney injury in hospitalized patients. Spontaneously resolves in
15
many cases. Can be fatal, especially during initial oliguric phase. f FE Ia.
16 Key findin g: granular ("muddy brown") casts r.J.
17 3 stages:
18 L Inciting event
19
2. Maintenance phasc-oliguric; lasts 1-3 weeks; risk of hyperkalemia, metabolic acidosis,
uremta
20
3. Recovery phase- polyuric; BU and serum creatinine fall; risk of hypokalemia
21 Can be caused by ischemic or nephrotoxic injury:
22 Ischemic-2° to ! renal blood Row (eg, hypotension, shock, sepsis, hemorrhage, HF). Results
23 in death of tubular cells that may slough into tubular lumen [lJ (PCT and th ick ascending limb
24
are highly susceptible to injury).
• Nephrotoxic-r to injury resulting from toxic substances (eg, aminoglycosides, radiocontrast
25
agents, lead, cisplatin, ethylene glycol), crush injury (myoglobinuria), hemoglobinuria. PCT is
26 particularly susceptible to injury.
27
28
29
FA11 p306.1
30 Lipid-lowering agents
31 DRUG LDL HDL TRIGLYCERIDES MECHANISMS Of ACTION ADVERSE EFFECTS/PROBLEMS
32 HMG-CoA reduct ase f Inhibit conversion of HMG- Hepatotoxicity {f LFTs),
33 inhibitors CoA to meva lonate, a myopathy (esp. when used

6
lock
s
Suspend
0
End Block
Item: 33 of 33 ~. I • M k <:] t> al ~· ~
QIO: 3483 .l. ar Previous Next lab 'lifllues Notes Calculator

• •
13
FA17 p306.1
14
Lipid-lowe ring agents
15 DRUG LDL HDL TRIGLYCERIDES MECHANISMS Of ACTION ADVERSE EFFECTS/PROBLEMS
16 HMG-CoA reduct ase Ul f l Inhibit conversion of HMG- Hepatotoxicity (f LFTs),
17 inhibitors CoA to meva lonate, a myopathy (esp. when used
18 (eg, lovastatin, cholesterol precursor; with fibrates or niacin)
pravastatin) l mortal ity in CAD patients
19
20
Bile acid resins u Slightly t Slightly f Pre,·ent intestinal reabsorption GI upset, l absorption of
Cholestyramine, of bile acids; liver must use other drugs and fat-soluble
21 cholesterol to make more vitamins
colestipol,
22 colesevelam
23 Ezetimibe u f /- l /- Prevent cholesterol absorption Rare f LF1S, diarrhea
24 at small intestine brush
25 border
26 Fib rates f lU Upregulate LPL - f TG Myopathy (f risk with
27 Ge mfibrozil, clearance statins), cholesterol
bezafibrate, Activates PPAR-a to induce gallstones
28 HDL synthesis
fenofibrate
29
Niacin (vitamin 8 3 ) u ff Inhibits lipolysis (hormone- Red, Aushed face, which is
30 sensitive lipase) in adipose l by NSAIDs or long-term
31 tissue; reduces hepatic VLDL use
32 synthesis Hyperglycemia
Hyperuricemia
33 • •

6
lock
s
Suspend
0
End Block
Item: 33 of 33 ~ 1 • M k -<:J 1>- Jil ~· !:';-~
QIO: 3483 ..L ar Pre v ious Next Lab fli!ltues Notes Calcula t o r
A A

13 PCSK9 inhibitors t Inactivation of LDI ,-receptor \tlyalgias, delirium,


14 Alirocumab, dcgradalion, increasing dementia, other
evolocumab amount of LDL remo,ed neurocognili,·e effects
15
from bloodstream
16
17
Liver Blood Enterocyte Intestinal lumen
AcetytCoA
18
HMG·CoA ApoE Lymphaocs

f
19 receptor CHY CHY
H CHY
20 red
Ezebmobe )
~ FFA
21
22
Cholesterol VLDL
pool

VLDL
IJ_
FFA
Cholesterol
FFA
23
~
~ileacids Bile acods
24
Noacin
25
Statins \ HDL
26 Lovastabn LPL·
Pravastatin r . : \eceptor UPREGULATION
27 Stmvastatin ~ +----11 Bile acid resins
Atorvast.Jtin Fibrates cholestyramlne
28 Rosuvast.Jtin colestipol
gemfibrozol
bezafibrate colesevelam
29 AdipoSe tiSSUe
fenohbrate
30
31
IPOLYSIS
32
33 )

a
Lock
s
Suspend
8
End Bl ock
Item: 33 of 33 ~ 1 • M k -<:J 1>- Jil ~· !:';-~
QIO: 3483 ..L ar Pre v ious Next Lab fli!ltues Notes Calcula t o r
A A

13
14 FA17 p 571 .2

15 Acute kidney injury Acute kidney injury is defined as an abrupt decline in renal function as measured by t creatinine
16 (acute renal failure) and t BUN or by oliguria/anuria.
17 Prerenal azotemia Due to l RBF (eg, hr potension) - l CFR. l\a+/H20 and BUN retained by kidney in an attempt to
18 consen·e volume - t BUN/creatinine ratio (BU is reabsorbed, creatinine is not) and l FE'-Ja·
19 Intrinsic renal failure Cenerallr due to acute tubular necrosis or ischemia/toxins; less commonly due to acute
20
glomerulonephritis (eg, RPC l, hemol} tic uremic syndrome) or acute interstitial nephritis.
In AT~. patchy necrosis - debris obstructing tubule and Auid backAow across necrotic tubule
21
- l GFR. Urine has epithelial/granular casts. BUN reabsorption is impaired - l BUI\/creatinine
22 ratio and t FENa·
23 Postrenal azotemia Due to outAow obstruction (stones, BPI I, neoplasia, congenital anomalies). Develops only with
24 bilateral obstruction.
25
Pre renal Intrinsic renal Postrenal
26
Urine osmolality > 500 < 350 < 350
27
(mOsm/kg)
28
Urine Na+ (mEq/L) < 20 > 40 > 40
29
FENa < I% >2% < 1% (mild)
30
> 2% (severe)
31
Serum BUN/Cr > 20 < 15 Varies
32
33 •

a
Lock
s
Suspend
8
End Bl ock

Das könnte Ihnen auch gefallen